Assessment of Musculoskeletal System; Musculoskeletal Trauma and Orthopedic Surgery

Ace your homework & exams now with Quizwiz!

The patient asks, "What does the doctor mean when he says that I have an avulsion fracture in my leg? I thought I had a sprain!" What is the best response by the nurse? A. Increase muscle strength with daily isometric exercise. B. Avoid exercising on concrete or hard pavement surfaces. C. Perform stretching and warm-up exercises before exercise. D. Wrap susceptible joints with elastic bandages or adhesive tape before exercise.

"It means that a ligament pulled a bone fragment loose." An avulsion fracture occurs when a ligament pulls a bone fragment loose, with pain similar to a sprain. A fracture with two or more fragments is a comminuted fracture. It is a spiral fracture when it is twisted around a bone shaft. It is a transverse fracture when the line of fracture is at right angles to the longitudinal axis.

38. Which joint surgery is used to arthroscopically remove degenerative tissue in joints? a. Osteotomy b. Arthrodesis c. Debridement d. Synovectomy

38. c. Debridement removes degenerative tissue from joints. Osteotomy corrects bone deformity by removal of a wedge or slice of bone. Arthrodesis surgically fuses a joint to relieve pain. Synovectomy removes tissue involved in joint destruction from rheumatoid arthritis (RA).

The patient had a pelvis fracture and open reduction surgery. What is a chief disadvantage of this form of fracture management? A. Increased pain B. Decreased mobility C. Infection D. Deformity

Answer: C Rationale: The chief disadvantages of open reduction (e.g. bone alignment through surgical incision) are infection, anesthesia complications, and effects of preexisting medical conditions.

A patient with carpal tunnel syndrome (CTS) reports being awakened at night due to tingling and numbness in the hands. Which activity should the nurse instruct the patient to perform to relieve the symptoms? 1 "Shake your hands." 2 "Wash your hands with water." 3 "Apply cold packs to your hands." 4 "Apply warm, moist heat to your hands."

1 Shaking the hands often relieves CTS symptoms. Washing hands with water will not relieve the symptoms. Cold application is performed to promote healing of sprains. Warm, moist heat is applied 24 to 48 hours after a sprain injury. Text Reference - p. 1509

Which athlete is least likely to experience a meniscus injury, according to the nurse who is assisting in conducting sports physicals? 1 A swimmer 2 A soccer player 3 A football player 4 A basketball player

1 A meniscus injury may be manifested as a popping, clicking, tearing sensation with effusion and swelling. Swimmers have repeated overhead arm movements and are not likely to suffer from this condition. Athletes who play basketball, soccer, and football have a higher risk of knee injury than swimmers. Test-Taking Tip: You have at least a 25 percent chance of selecting the correct response in multiple-choice items. Text Reference - p. 1510

The patient had frostbite on the distal areas of the toes on both feet. The patient is scheduled for amputation of the damaged tissue. Which assessment finding or diagnostic study is the most objective indicator for locating the level of the patient's injury? 1 Arteriography showing blood vessels 2 Peripheral pulse assessment bilaterally 3 Patches of black, indurated, and cold tissue 4 Bilateral pale and cool skin below the ankles

1 Arteriography is the most objective study to determine viable tissue for salvage based on perfusion, because actual blood flow through the tissues is observed in real time. It is considered the gold standard for evaluating arterial perfusion. Bilateral peripheral pulse assessment, areas of black, indurated, and cold tissue, and bilateral pale and cool skin all identify the lack of tissue perfusion, but not the specific area where tissue perfusion stops and amputation needs to occur. Text Reference - p. 1530

Which primary manifestation is associated with bursitis? 1 Swelling 2 Painless joints 3 Tingling sensation 4 Altered neurovascular status.

1 Bursitis is the inflammation in the closed sacs lined with synovial membrane that contain synovial fluid and are located between tendons and bones near the joints. The swelling is the primary manifestation of bursitis. Bursitis is characterized by painful joints and a warm sensation, but not a tingling sensation. The manifestation of altered neurovascular status may appear later in acute soft tissue injury. Text Reference - p. 1511

A coal miner with a history of rheumatoid arthritis reports pain, swelling, and a limited range of motion in the knee joints. What diagnosis does the nurse suspect? 1 Bursitis 2 Shin splints 3 Rotator cuff tear 4 Impingement syndrome

1 Bursitis is the inflammation of the bursae located near the joints. Rheumatoid arthritis causes inflammation in the joints, resulting in friction between joint surfaces. The repetitive kneeling involved in occupations such as coal mining may result in bursitis. A shin splint are is inflammation along the anterior aspect of the calf due to periostitis. A rotator cuff tear is a muscle tear around the shoulder joint. Impingement syndrome is also a shoulder injury. Text Reference - p. 1511

When teaching a group of young adults, what general measures should a nurse teach to prevent injuries? Select all that apply. 1 Use seat belts regularly. 2 Avoid drunken driving. 3 Drive slower than the posted speed limit. 4 Use safety equipment at work. 5 Use protective athletic equipment. 6 While driving, talk on the phone only if the call is important.

1, 2 4, 5 A nurse plays a pivotal role in teaching patients to take appropriate measures to prevent injuries. Some of these measures include regularly using seat belts; avoiding drunken driving, using protective athletic equipment (helmets and knee, wrist, and elbow pads), and using safety equipment at work. Obeying the speed limit is sufficient; the nurse does not need to teach that people should always drive slower than the speed limit. The teaching should encourage people not to use the phone at all while driving; if an important call must be made, the driver should find a safe place to stop before making it. Test-Taking Tip: Answer the question that is asked. Read the situation and the question carefully, looking for key words or phrases. Do not read anything into the question or apply what you did in a similar situation during one of your clinical experiences. Think of each question as being an ideal, yet realistic, situation. Text Reference - p. 1518

The nurse reviews the test results of a patient that reports shoulder weakness with decreased range-of-motion (ROM). What activities may have contributed to the patient's condition? Select all that apply. 1 Lifting heavy weights 2 A blow to the upper arm 3 Falling onto an outstretched arm 4 Working with vibrating power tools 5 Using a computer keyboard frequently

1, 2, 3 The positive results of the drop arm test and a tear seen in the MRI scan may indicate a rotator cuff injury. A tear in the rotator cuff may occur due to repeated overhead motions and can include heavy weight lifting, a blow to the upper arm, or falling onto an outstretched arm. The other activities such as working with vibrating power tools or using a computer keyboard and mouse frequently are not likely to cause a rotator cuff injury. Text Reference - p. 1510

The nurse provides education to an athlete about how to avoid sprains and strains. Which suggestion is appropriate for the nurse to include? Select all that apply. 1 "Perform balancing exercises." 2 "Use a cane while walking." 3 "Perform strengthening exercises." 4 "Take an analgesic before exercising." 5 "Perform muscle warming up exercises before vigorous activities."

1, 3, 5 Balance exercises help in preventing falls. Strengthening exercises help in building up muscle strength and bone density. Performing warm-up exercises before any vigorous activity reduces the risk of sprains and strains. A cane can assist in walking but does not reduce sprains or strains. Taking analgesics before exercise in the absence of injury or pain may be unnecessary. Text Reference - p. 1507

When performing the physical examination of a patient, how should the nurse assess the function of the peroneal nerve? Select all that apply. 1 Assess dorsiflexion of the foot. 2 Assess plantar flexion of the foot. 3 Assess sensation in the sole of the foot between the first and second toes. 4 Assess sensation in the sole of the foot between the third and fourth toes. 5 Assess sensation in the dorsal part of the foot between the first and second toes.

1, 5 The peroneal nerve has motor and sensory functions. The motor function is assessed by looking for dorsiflexion of the foot, whereas the sensory function is assessed by looking for sensation on the dorsal aspect of the foot between the first and second toes. Plantar flexion of the foot and sensory supplies to the sole of the foot are the functions of the tibial nerve. The function of the peroneal nerve is not assessed by sensation in the sole of the foot between the third and fourth toes or by sensation in the dorsal part of the foot between the first and second toes, because the peroneal nerve does not supply these areas. Test-Taking Tip: Being emotionally prepared for an examination is key to your success. Proper use of this text over an extended period of time ensures your understanding of the mechanics of the examination and increases your confidence about your nursing knowledge. Your lifelong dream of becoming a nurse is now within your reach! You are excited, yet anxious. This feeling is normal. A little anxiety can be good because it increases awareness of reality; but excessive anxiety has the opposite effect, acting as a barrier and keeping you from reaching your goal. Your attitude about yourself and your goals will help keep you focused, adding to your strength and inner conviction to achieve success. Text Reference - p. 1518

10. The x-ray shows that the patient's fracture is at the remodeling stage. What characteristics of the fracture healing process are happening at this stage (select all that apply)? a. Radiologic union b. Absorption of excess cells c. Return to preinjury strength and shape d. Semisolid blood clot at the ends of fragments e. Deposition and absorption of bone in response to stress f. Unorganized network of bone woven around fracture parts

10. a, b, c, e. When the remodeling stage of healing occurs, radiologic union is present. Excess callus is reabsorbed, trabecular bone is laid, and the bone returns to its preinjury structure strength and shape. The osteoblasts and osteoclasts function normally in response to stress. The fracture hematoma stage is when the hematoma at the ends of the fragments becomes a semisolid blood clot. There is an unorganized network of bone composed of cartilage, osteoblasts, calcium, and phosphorus woven around fracture parts in the callus formation stage.

11. A patient is brought to the emergency department with an injured lower left leg following a fall while rock climbing. The nurse identifies the presence of a fracture based on what cardinal sign of fracture? a. Muscle spasms b. Obvious deformity c. Edema and swelling d. Pain and tenderness

11. b. Deformity is the cardinal sign of fracture but may not be apparent in all fractures. Other supporting signs include edema and swelling, localized pain and tenderness, muscle spasm, ecchymosis, loss of function, crepitation, and an inability to bear weight.

12. A patient with a fractured femur experiences the complication of malunion. The nurse recognizes that what happens with this complication? a. The fracture heals in an unsatisfactory position. b. The fracture fails to heal properly despite treatment. c. Fracture healing progresses more slowly than expected. d. Loss of bone substances occurs as a result of immobilization.

12. a. A malunion occurs when the bone heals in the expected time but in an unsatisfactory position, possibly resulting in deformity or dysfunction. Nonunion occurs when the fracture fails to heal properly despite treatment and delayed union is healing of the fracture at a slower rate than expected. In posttraumatic osteoporosis, the loss of bone substances occurs as a result of immobilization.

13. What is a disadvantage of open reduction and internal fixation of a fracture compared to closed reduction? a. Infection b. Skin irritation c. Nerve impairment d. Complications of immobility

13. a. Open reduction uses a surgical incision to correct bone alignment but infection is the main disadvantage, as well as anesthesia complications or the effect of preexisting medical conditions. Skin irritation and nerve impairment is most likely with skin traction. Prolonged immobility is possible with skeletal traction.

14. A young patient with a fractured femur has a hip spica cast applied. While the cast is drying, what should the nurse do? a. Elevate the legs above the level of the heart for 24 hours. b. Turn the patient to both sides and prone to supine every 2 hours. c. Cover the cast with a light blanket to avoid chilling from evaporation. d. Assess the patient frequently for abdominal pain, nausea, and vomiting.

14. d. Complaints of abdominal pain or pressure, nausea, and vomiting are signs of cast syndrome that occur when hip spica casts or body jacket braces are applied too tightly, causing compression of the superior mesenteric artery against the duodenum. The cast may need to be split or removed and the health care provider should be notified. Elevation is not indicated for a spica cast and the patient with a spica cast should not be placed in the prone position during the initial drying stage because the cast is so large and heavy it may break. A cast should never be covered with a blanket because heat builds up in the cast and may increase edema.

15. A patient is admitted with an open fracture of the tibia following a bicycle accident. During assessment of the patient, what specifically should the nurse question the patient about? a. Any previous injuries to the leg b. The status of tetanus immunization c. The use of antibiotics in the last month d. Whether the injury was exposed to dirt or gravel

15. b. Infection is the greatest risk with an open fracture and all open fractures are considered contaminated. Tetanus prevention is always indicated if the patient has not been immunized or does not have current boosters. Prophylactic antibiotics are often used in management of open fractures but recent antibiotic therapy is not relevant, nor is previous injury to the site.

Priority Decision: A patient has fallen in the bathroom of the hospital room and reports pain in the upper right arm and elbow. Before splinting the injury, the nurse knows that the priority management of a possible fracture should include which action? a. Elevation of the arm b. Application of ice to the site c. Notification of the health care provider d. Neurovascular checks below the site of the injur

16. d. Pulses, sensation, and motor function distal to the injury should be checked before and after splinting to assess for nerve or vascular damage and documented to avoid doubts about whether a problem discovered later was missed during the original examination or was caused by the treatment. Elevation of the limb and application of ice should be instituted after the extremity is splinted.

17. To assess for neurologic status in a patient with a fractured humerus, what should the nurse ask the patient to do? a. Evert, invert, dorsiflex, and plantar flex the foot. b. Abduct, adduct, and oppose the fingers and pronate and supinate the hand. c. Assess the location, quality, and intensity of pain below the site of the injury. d. Assess the color, temperature, capillary refill, peripheral pulses, and presence of edema in the extremity.

17. b. Neurologic assessment includes evaluation of motor and sensory function and, in the upper extremity, includes abduction and adduction of the fingers, opposition of the fingers, and supination and pronation of the hands. It also includes sensory perception in the fingers. Evaluation of the feet would occur in lower extremity injuries. Assessment of color, temperature, capillary refill, peripheral pulses, and edema evaluates vascular status.

18. A patient is discharged from the outpatient clinic following application of a synthetic fiberglass long arm cast for a fractured ulna. Before discharge, the nurse instructs the patient to do what? a. Never get the cast wet. b. Move the shoulder and fingers frequently. c. Place tape petals around the edges of the cast when it is dry. d. Use a sling to support the arm at waist level for the first 48 hours.

18. b. A patient with any type of cast should exercise the joints above and below the cast frequently and moving the fingers frequently will improve circulation and help to prevent edema. Unlike plaster casts, thermoplastic resin or fiberglass casts are relatively waterproof and, if they become wet, can be dried with a hair dryer on low setting. Tape petals are used on plaster casts to protect the edges from breaking and crumbling but are not necessary for synthetic casts. After the cast is applied, the extremity should be elevated at about the level of the heart to promote venous return and ice may be used to prevent edema.

19. A patient with a fractured tibia accompanied by extensive soft tissue damage initially has a splint applied and held in place with an elastic bandage. What early sign should alert the nurse that the patient is developing compartment syndrome? a. Paralysis of the toes b. Absence of peripheral pulses c. Distal pain unrelieved by opioid analgesics d. Skin over the injury site is blanched when the bandage is removed

19. c. Pain that is distal to the injury and is unrelieved by opioid analgesics is the earliest sign of compartment syndrome. Paralysis and absence of peripheral pulses will eventually occur if it is not treated but these are late signs that often appear after permanent damage has occurred. The overlying skin may appear normal because the surface vessels are not occluded.

What measures should a nurse take to prevent muscle spasms in cases of musculoskeletal injuries? Select all that apply. 1 Massage the muscle spasms. 2 Align the affected extremity appropriately. 3 Use thermotherapy on the affected extremity. 4 Place the affected extremity in a comfortable position. 5 Provide isometric muscle strengthening exercises.

2, 3, 4 Muscle spasms are caused by involuntary muscle contraction after fracture, strain, or nerve injury. These may displace a nondisplaced fracture or prevent it from healing spontaneously. Therefore it is important to take measures to prevent muscle spasms. The affected extremity should be aligned appropriately. Thermotherapy may reduce muscle spasm. The extremity should be placed in a comfortable position. Massaging a muscle spasm may stimulate muscle tissue contraction, further increasing pain and spasm. Therefore it is not advisable to massage spasms. An isometric muscle-strengthening exercise regimen will not prevent muscle spasms in this injured patient.

The nurse provides discharge education to a patient who had recently underwent closed reduction therapy for realigning a dislocated hip joint. What instruction is appropriate for the nurse to include in the care plan? Select all that apply. 1 "Take a prophylactic antibiotic." 2 "Restrict strenuous activities." 3 "Return to normal activities gradually." 4 "Apply warm, moist heat to the affected area." 5 "Perform gentle range-of-motion exercises."

2, 3, 5 The patient who recently underwent closed reduction therapy for realigning the hip dislocation is advised to restrict strenuous activities that strain the joint. The patient should gradually return to the normal activities. The patient should perform gentle ROM exercises to prevent motion limitations of the joint. Antibiotic prophylaxis is appropriate for patients with open fractures and large tissue defects. The application of warm and moist heat is useful in a strain injury. Text Reference - p. 1508

2. The nurse teaches individuals that one of the best ways to prevent musculoskeletal injuries during physical exercise is by doing what? a. Increase muscle strength with daily isometric exercise. b. Avoid exercising on concrete or hard pavement surfaces. c. Perform stretching and warm-up exercises before exercise. d. Wrap susceptible joints with elastic bandages or adhesive tape before exercise.

2. c. Warm-up exercises "prelengthen" potentially strained tissues by avoiding the quick stretch often encountered in sports and also increase the temperature of muscle, resulting in increased speed of cell metabolism, increased speed of nerve impulses, and improved oxygenation of muscle fibers. Stretching is also thought to improve kinesthetic awareness, lessening the chance of uncoordinated movement. Muscle strength is not a key factor in soft tissue injuries and taping or wrapping joints may actually predispose a person to injury by weakening the joint, unless a previous injury is being treated.

20. What surgical treatment will the nurse prepare the patient for in the presence of compartment syndrome? a. Fasciotomy b. Amputation c. Internal fixation d. Release of tendons

20. a. Soft tissue edema in the area of the injury may cause an increase of pressure within the closed spaces of the tissue compartments formed by the nonelastic fascia, creating compartment syndrome. If symptoms occur, it may be necessary to incise the fascia surgically, a procedure known as a fasciotomy. Amputation is usually necessary only if the limb becomes septic because of untreated compartment syndrome.

21. Which type of fracture occurred when there is radial nerve and brachial artery damage and the fracture is reduced with a hanging arm cast? a. Fractured tibia b. Colles' fracture c. Fractured humerus d. Femoral shaft fracture

21. c. The fractured humerus may cause radial nerve and brachial artery damage and it may be reduced nonsurgically with a hanging arm cast. A fractured tibia and femoral shaft are in the leg. The Colles' fracture is in the wrist and manifests with pronounced swelling and obvious deformity of the wrist; it is treated with closed manipulation and immobilization.

22. The woman with osteoporosis slipped on the ice and now her wrist hurts. If there is a fracture, what type of fracture is expected? a. Dislocation b. Open fracture c. Colles' fracture d. Incomplete fracture

22. c. A Colles' fracture most often occurs in patients over 50 years of age with osteoporosis and frequently when the patient attempts to break a fall with an outstretched arm and hand. Dislocation is the complete separation of articular surfaces of the joint caused by a ligament injury. Open fracture is when there is communication with the external environment. A fracture is incomplete if only part of the bone shaft is fractured and the bone is still in one piece.

24. In a patient with a stable vertebral fracture, what should the nurse teach the patient to do? a. Remain on bed rest until the pain is gone. b. Logroll to keep the spine straight when turning. c. How to use bone cement to correct the problem. d. Take as much analgesic as needed to relieve the pain.

24. b. The spine should be kept straight by turning the shoulders and hips together (logrolling). This keeps the spine in good alignment until union has been accomplished. Bed rest may be required for a short time but not until the pain is gone. Analgesics should be taken only as ordered. If they do not relieve the pain, the health care provider should be notified. Bone cement is used by the surgeon to stabilize vertebral compression fractures.

When is a fat embolism most likely to occur? a. 24 to 48 hours following a fractured tibia b. 36 to 72 hours following a skull fracture c. 4 to 5 days following a fractured femur d. 5 to 6 days following a pelvic fracture

25. a. Initial manifestations of fat embolism usually occur 24 to 48 hours after injury and are associated with fractures of long bones and multiple fractures related to pelvic injuries, including fractures of the femur, tibia, ribs, and pelvis.

26. The nurse suspects a fat embolism rather than a pulmonary embolism from a venous thrombosis when the patient with a fracture develops what? a. Tachycardia and dyspnea b. A sudden onset of chest pain c. Petechiae around the neck and upper chest d. Electrocardiographic (ECG) changes and decreased PaO2

26. c. Patients with fractures are at risk for both fat embolism and pulmonary embolism from venous thromboembolism but there is a difference in the time of occurrence, with fat embolism occurring shortly after the injury and thrombotic embolism occurring several days after immobilization. They both may cause pulmonary symptoms of chest pain, tachypnea, dyspnea, apprehension, tachycardia, and cyanosis. However, fat embolism may cause petechiae located around the neck, anterior chest wall, axilla, buccal membrane of the mouth, and conjunctiva of the eye, which differentiates it from thrombotic embolism.

27. Which kind of hip fracture is usually repaired with a hip prosthesis? a. Intracapsular b. Extracapsular c. Subtrochanteric d. Intertrochanteric

27. a. A hip prosthesis is usually used for intracapsular fractures. The other options are all extracapsular fractures.

28. An older adult woman is admitted to the emergency department after falling at home. The nurse cautions her not to put weight on the leg after finding what in the patient assessment? a. Inability to move the toes and ankle b. Edema of the thigh extending to the knee c. Internal rotation of the leg with groin pain d. Shortening and external rotation of the leg

28. d. The classic signs of a hip fracture are shortening of the leg and external rotation accompanied by severe pain at the fracture site and additional injury could be caused by weight bearing on the extremity. The patient may not be able to move the hip or the knee but movement in the ankle and toes is not affected.

29. A patient with an extracapsular hip fracture is admitted to the orthopedic unit and placed in Buck's traction. The nurse explains to the patient that the purpose of the traction is to do what? a. Pull bone fragments back into alignment b. Immobilize the leg until healing is complete c. Reduce pain and muscle spasms before surgery d. Prevent damage to the blood vessels at the fracture site

29. c. Although surgical repair is the preferred method of managing intracapsular and extracapsular hip fractures, initially patients frequently may be treated with skin traction, such as Buck's traction or Russell's traction, to immobilize the limb temporarily and to relieve the painful muscle spasms before surgery is performed. Prolonged traction would be required to reduce the fracture or immobilize it for healing, creating a very high risk for complications of immobility.

A patient reports a clicking, popping, and locking sensation with knee instability and tests positive when performing McMurray's test. Which condition can occur in the patient, if this is left untreated? 1 Bursitis 2 Avascular necrosis 3 Quadriceps atrophy 4 Pathologic fracture

3 A clicking, popping, and locking sensation with knee instability along with a positive McMurray's test indicate a meniscus injury. If untreated for a prolonged period, the meniscus injury may result in quadriceps atrophy due to disuse of the muscle. Bursitis is inflammation of the bursae and is not due to a meniscus injury. Avascular necrosis results from a lack of blood flow to the bones and does not occur due to untreated meniscus injury. A pathologic fracture is not associated with the prolonged lack of treatment of meniscus injury. Text Reference - p. 1511

The nurse cares for a patient who experiences a severe hip injury. What condition places the patient at risk for avascular necrosis? 1 Open joint injury 2 Intraarticular injury 3 Inadequate blood supply to the bone 4 Damaged adjacent neurovascular tissue

3 A posterior hip dislocation due to severe injury may result in avascular necrosis if the joints remain unreduced for a long time. The dislocated parts may block the blood supply to the bone, resulting in bone death and necrosis. The open joint injuries are susceptible to infection but are not associated with avascular necrosis. Intraarticular injuries are another complication of dislocation and may not be related to avascular necrosis. The adjacent neurovascular tissue will be manifested by altered neurovascular status. Text Reference - p. 1511

Which is a common site of bursitis? 1 Back 2 Thighs 3 Elbows 4 Abdomen

3 Bursitis is the inflammation of the bursae. These are located at sites of friction such as between tendons and bones and near the joints. The most common sites of bursitis occurrence include the elbows, shoulders, and greater trochanters of the hip. The back, thighs, and abdomen are less likely to be affected. Text Reference - p. 1511

What is carpel tunnel syndrome (CTS)? 1 The tear within the muscle or tendinoligamentous structures around the shoulder 2 The entrapment of soft tissue structures under the coracoacromial arch of the shoulder 3 The compression of the median nerve that enters the hand through the narrow confines of the carpal tunnel 4 The injury to fibrocartilage of knee, characterized by popping, clicking, tearing sensation, effusion, and swelling

3 CTS involves the compression of the median nerve, which enters the hand through the narrow confines of the carpal tunnel. A rotator cuff tear is a tear within the muscle or tendinoligamentous structures around shoulder. The entrapment of soft tissue structures under the coracoacromial arch of the shoulder is impingement syndrome. The injury to fibrocartilage of the knee, characterized by popping, clicking, tearing sensation, effusion, and swelling, is a meniscus injury. Text Reference - p. 1509

What is the most useful initial nursing action for a patient who sustains a joint sprain? 1 Seeking a prescription for a prophylactic antibiotic 2 Administering an over-the-counter analgesic 3 Applying ice compresses to the injured area 4 Providing the patient with instructions about weight bearing

3 Ice application is the most useful intervention after a sprain. Cold compression produces hypothermia of the affected area, facilitating vasoconstriction and reducing the perception and transmission of nerve pain impulses. Antibiotic prophylaxis is administered for an open fracture or external extremity injury. Analgesics such as aspirin can be administered after the cold compress. Patient instructions should be provided after the extent of the injury is determined and initial interventions performed. Text Reference - p. 1507

A nurse provides postoperative instructions to a patient who underwent reconstructive surgery for an anterior crucial ligament (ACL) injury. Which statement made by the patient indicates the need for further teaching? 1 "I should do range-of-motion (ROM) exercises." 2 "My knees should be kept immobilized while healing." 3 "It is not likely that my physical functioning will ever return to normal." 4 "Progressive weight-bearing may be critical for healing."

3 Reconstructive therapy involves the removal of a torn anterior cruciate ligament tissue and its replacement with an allograft tissue. The patient's physical functioning may return to normal in six to eight months. ROM exercises are encouraged soon after the surgery to prevent complications related to prolonged immobilization. The knee is placed in a brace or immobilized after surgery. Progressive weight-bearing is determined by the degree of surgical repair. Text Reference - p. 1511

Which condition does the nurse anticipate if a patient experiences a severe injury to the ligament around the humerus bone? 1 Strain 2 Sprain 3 Dislocation 4 Subluxation

3 Severe injury of the ligamentous structures surrounding the humerus is a dislocation. A strain is an excessive stretching of the muscle, its fascial sheath, or a tendon. A sprain generally represents an injury to the ligament structures surrounding a joint. Subluxation is a partial displacement of the joint surface. Text Reference - p. 1508

A nurse applies an elastic bandage to a patient's knee and provides discharge instructions after teaching the patient how to reapply the bandage. Which statement made by the patient indicates the need for further teaching? 1 "I will wrap it tightly but ensure that there is no numbness." 2 "I will wrap it starting from distal to proximal end." 3 "I will leave it in place for prolonged periods." 4 "I will leave it in place for 30 minutes and then remove it for 15 minutes."

3 The elastic bandage should not be wrapped for prolonged periods, because it may irritate the area and cut off circulation. The bandage should not be wrapped too tightly, because it may interfere with the blood supply and cause numbness. The ideal way of wrapping the bandage is to start from the distal end and progress to the proximal area. The bandage should be wrapped for 30 minutes and removed for 15 minutes. Test-Taking Tip: Identifying content and what is being asked about that content is critical to your choosing the correct response. Text Reference - p. 1507

The nurse is caring for a patient with a dislocated hip. The nurse knows that this type of injury is associated most commonly with: 1 Fall from a high place 2 Osteoporosis of the hip 3 Motor vehicle collisions 4 Pathologic fractures of the hip

3 The hip is the most common dislocation of the lower extremity and generally is associated with motor vehicle collisions. Falls from a high place generally result in a fracture. Osteoporosis of the hip joint is more likely to result in a fracture and not dislocation. Pathologic fractures secondary to a history of cancer are not associated with hip dislocations. Text Reference - p. 1586

31. What should the nurse include in discharge instructions for the patient following a hip prosthesis with a posterior approach? a. Restrict walking for 2 to 3 months. b. Take a bath rather than a shower to prevent falling. c. Keep the leg internally rotated while sitting and standing. d. Have a family member put on the patient's shoes and socks.

31. d. Patients with hip prostheses with a posterior approach must avoid extreme flexion, adduction, or internal rotation for at least 6 weeks to prevent dislocation of the prosthesis. Gradual weight bearing on the limb is allowed and ambulation should be encouraged. The leg should be not be externally rotated.

A patient has an injury to the eye. On examination, the nurse finds a brown tissue on the surface of the ocular globe and a teardrop-shaped pupil. What should the nurse do next? Select all that apply. 1 Assess for the function of cranial nerves. 2 Send the patient for a computed tomography (CT) scan. 3 Stop further examination of the eye. 4 Place a protective shield over the eye. 5 Put antibiotic eyedrops in the eye.

3, 4 The presence of brown tissue (iris or ciliary body) on the surface of the globe, extrusion of vitreous humor, and an eccentric or teardrop-shaped pupil indicate rupture of the globe. When such an injury is suspected, it is important to stop the examination and place a protective shield over the involved eye. Assessment of the function of the cranial nerves and sending the patient for a CT scan are done in the case of any facial fractures, but they aren't directly related to the eye assessment. Because this is a case of globe rupture, antibiotic eyedrops should be given only per the prescription of an ophthalmologist. Test-Taking Tip: Practicing a few relaxation techniques may prove helpful on the day of an examination. Relaxation techniques such as deep breathing, imagery, head rolling, shoulder shrugging, rotating and stretching of the neck, leg lifts, and heel lifts with feet flat on the floor can effectively reduce tension while causing little or no distraction to those around you. It is recommended that you practice one or two of these techniques intermittently to avoid becoming tense. The more anxious and tense you become, the longer it will take you to relax. Text Reference - p. 1529

3. The patient asks, "What does the doctor mean when he says that I have an avulsion fracture in my leg? I thought I had a sprain!" What is the best response by the nurse? a. "It is a fracture with more than two fragments." b. "It means that a ligament pulled a bone fragment loose." c. "The line of the fracture is twisted along the shaft of the bone." d. "The line of the fracture is at right angles to the longitudinal axis of the bone."

3. b. An avulsion fracture occurs when a ligament pulls a bone fragment loose, with pain similar to a sprain. A fracture with two or more fragments is a comminuted fracture. It is a spiral fracture when it is twisted around a bone shaft. It is a transverse fracture when the line of fracture is at right angles to the longitudinal axis.

30. A patient with a fractured right hip has an anterior open reduction and internal fixation of the fracture. What should the nurse plan to do postoperatively? a. Get the patient up to the chair on the first postoperative day. b. Position the patient only on the back and the unoperative side. c. Keep the leg abductor pillow on the patient even when bathing. d. Ambulate the patient with partial weight bearing by discharge.

30. a. Because the fracture site is internally fixed with pins or plates, the fracture site is stable and the patient is moved from the bed to the chair on the first postoperative day. Ambulation begins on the first or second postoperative day without weight bearing on the affected leg. Weight bearing on the affected extremity is usually restricted for 6 to 12 weeks until adequate healing is evident on x-ray. The patient may be positioned on the operative side following internal fixation and abductor pillows are used for patients who have total hip replacements.

32. When preparing a patient for discharge following fixation of a mandibular fracture, the nurse determines that teaching has been successful when the patient says what? a. "I can keep my mouth moist by sucking on hard candy." b. "I should cut the wires with scissors if I begin to vomit." c. "I may use a bulk-forming laxative if my liquid diet causes constipation." d. "I should use a moist swab to clean my mouth every time I eat something."

32. c. The low-bulk, high-carbohydrate liquid diet and intake of air through a straw required during mandibular fixation often lead to constipation and flatus, which may be relieved with bulk-forming laxatives, prune juice, or ambulation. Wires or rubber bands should be cut only in the case of cardiac or respiratory arrest and patients should be taught to clear their mouth of vomitus or secretions. The mouth should be thoroughly cleaned with water, saline, or alkaline mouthwashes or using a Water Pik as necessary to remove food debris. Hard candy should not be held in the mouth.

33. Priority Decision: Twenty-four hours after a below-the-knee amputation, a patient uses the call system to tell the nurse that his dressing (a compression bandage) has fallen off. What is the first action that the nurse should take? a. Apply ice to the site. b. Cover the incision with dry gauze. c. Reapply the compression dressing. d. Elevate the extremity on a couple of pillows.

33. c. The compression dressing or bandage supports the soft tissues, reduces edema, hastens healing, minimizes pain, and promotes residual limb shrinkage. If the dressing is left off, edema will form quickly and may delay rehabilitation. Elevation and ice will not be as effective at preventing the edema that will form. Dressing the incision with dry gauze will not provide the benefits of a compression dressing.

A patient complains of pain in the foot of a leg that was recently amputated. What should the nurse recognize about this pain? a. It is caused by swelling at the incision. b. It should be treated with ordered analgesics. c. It will become worse with the use of a prosthesis. d. It can be managed with diversion because it is psychologic.

34. b. Phantom sensation or phantom pain may occur following amputation, especially if pain was present in the affected limb preoperatively. The pain is a real sensation to the patient and will first be treated with analgesics and other pain interventions (i.e., tricyclic antidepressants, antiseizure drugs, transcutaneous electrical nerve stimulation [TENS], mirror therapy, acupuncture). As recovery and ambulation progress, phantom limb sensation usually subsides.

35. Priority Decision: An immediate prosthetic fitting during surgery is used for a patient with a traumatic below-theknee amputation. During the immediate postoperative period, what is a priority nursing intervention? a. Monitor the patient's vital signs. b. Assess the incision for hemorrhage. c. Elevate the residual limb on pillows. d. Have the patient flex and extend the knee every hour.

35. a. Because the device covers the residual limb, the surgical site cannot be directly seen and postoperative hemorrhage is not apparent on dressings, requiring vigilant assessment of vital signs for signs of bleeding. Elevation of the residual limb with an immediate prosthetic fitting is not necessary because the device itself prevents edema formation. Exercises to the leg are not performed in the immediate postoperative period to avoid disruption of ligatures and the suture line.

36. Why does a nurse position a patient with an above-the-knee amputation with a delayed prosthetic fitting prone several times a day? a. To prevent flexion contractures b. To assess the posterior skin flap c. To reduce edema in the residual limb d. To relieve pressure on the incision site

36. a. Flexion contractures, especially of the hip, may be debilitating and delay rehabilitation of the patient with a leg amputation. To prevent hip flexion, the patient should avoid sitting in a chair with the hips flexed or having pillows under the surgical extremity for prolonged periods and the patient should lie on the abdomen for 30 minutes three to four times a day to extend the hip.

37. A patient who had a below-the-knee amputation is to be fitted with a temporary prosthesis. It is most important for the nurse to teach the patient to do what? a. Inspect the residual limb daily for irritation. b. Apply an elastic shrinker before applying the prosthesis. c. Perform range-of-motion (ROM) exercises to the affected leg four times a day. d. Apply alcohol to the residual limb every morning and evening to toughen the skin.

37. a. Skin breakdown on the residual limb can prevent the use of a prosthesis so the limb should be inspected every day for signs of irritation or pressure areas. No substances except water and mild soap should be used on the residual limb and range-of-motion (ROM) exercises are not necessary when the patient is using a prosthesis. A residual limb shrinker is an elastic stocking that is used to mold the limb in preparation for prosthesis use but a cotton residual limb sock is worn with the prosthesis.

39. When the nursing student asks the RN what an arthroplasty is, what is the best description the RN can give the student? a. Surgical fusion of a joint to relieve pain b. Correction of bone deformity by removal of a wedge or slice of bone c. Reconstruction or replacement of a joint to relieve pain and correct deformity d. Used in rheumatoid arthritis to remove the tissue involved in joint destruction

39. c. An arthroplasty is reconstruction or replacement of a joint to relieve pain and correct deformity, especially with osteoarthritis, RA, avascular necrosis, congenital deformities, or dislocations. Arthrodesis is the surgical fusion of a joint to relieve pain. An osteotomy removes a wedge of bone to correct a bone deformity. Synovectomy is used in RA to remove the tissue involved in joint destruction.

A patient states, "I twisted my ankle while walking." Which assessment finding make the nurse suspect that the injury is a third degree sprain? 1 Swelling around the ankle 2 Redness of the skin around the ankle 3 Pain on movement of the ankle 4 Muscle gap on palpation of the skin around the ankle

4 A third degree sprain involves complete tearing of the ligament, and a gap in the muscle may be apparent or be palpated through the skin. The swelling may occur due to the inflammatory responses by the released mediators but is not specific to the degree of sprain. Redness may or may not be present. The patient may feel pain due to the injury irrespective of the degree of the sprain, but the pain becomes severe in a third degree sprain. Test-Taking Tip: Sometimes the reading of a question in the middle or toward the end of an exam may trigger your mind with the answer. Text Reference - p. 1506

A patient sustains a severe hip injury with dislocation in the posterior direction. Joint aspiration reveals hemarthrosis. What condition does the nurse suspect? 1 Bursitis 2 Arthrofibrosis 3 Avascular necrosis 4 Intraarticular fracture

4 Severe injury may result from posterior hip dislocation due to damage to the ligament structure around the joint. The presence of hemarthrosis indicates an intraarticular fracture and bleeding into the joint space. Bursitis is the inflammation of closed sae bursae. Arthrofibrosis is the "freezing" of the shoulder after prolonged immobilization after surgery. Avascular necrosis is associated with inadequate blood supply, resulting in bone cell death. Text Reference - p. 1508

What is the best intervention for a patient with an anterior cruciate ligament (ACL) injury who evidences tight and painful effusion? 1 Applying ice 2 Elevating the knee 3 Administering aspirin 4 Preparing for aspiration

4 The anterior cruciate ligament injury may involve a tear from the bone attachments that form the knee. The patient may report a tight and painful effusion, and a joint aspiration may be needed. Application of ice interferes with transmission of pain impulses and may not help in joint effusion. Elevation of the knee relieves edema. Nonsteroidal antiinflammatory drugs (NSAIDs) such as aspirin may relieve the pain at the injury site, but may not relieve effusion. Text Reference - p. 1511

The nurse reviews the plan of care for the initial management of a patient with an injured ankle ligament. Which item listed on the care plan requires attention? 1 Apply ice to the ankle. 2 Limit movement of the ankle. 3 Keep the affected ankle elevated. 4 Apply warm, moist heat to the ankle.

4 The application of warm compresses should be applied after the acute phase of injury as the risk of internal bleeding subsides. Ice should be applied to the ankle to reduce inflammation and pain. The ankle should be elevated to prevent swelling and encourage fluid drainage. The movement of the ankle should be restricted to relive pain and provide rest. Test-Taking Tip: Look for answers that focus on the patient or that are directed toward the patient's feelings. Text Reference - p. 1507

A patient reports pain and inflammation of the knee. The nurse flexes the knee by 15 to 30 degrees and also pulls the tibia forward while keeping the femur stable. The nurse feels a soft forward motion of the tibia with an indistinct endpoint. What diagnosis does the nurse anticipate? 1 Bursitis 2 Rotator cuff injury 3 Carpal tunnel syndrome 4 Anterior cruciate ligament injury

4 The knee with an anterior cruciate ligament tear may produce a positive Lachman's test. This test involves flexing the knee by 15 to 30 degrees and pulling the tibia forward, keeping the femur stable. It is considered positive with forward motion of the tibia with a soft feeling and indistinct endpoint. This test is not performed to assess bursitis. A rotator cuff injury may be examined by a drop arm test. Carpel tunnel syndrome may be examined by positive results for Tinel's sign and Phalen's sign. Text Reference - p. 1511

The nurse reviews the results of a magnetic resonance image (MRI) study that was performed on a patient with a severe shoulder injury. Which MRI finding has the potential risk of dislocation? 1 Tearing of the ligaments around the shoulder joint 2 Mild tears within the muscles around the shoulder joint 3 Severe tearing within the muscles around the shoulder joint 4 Deformation injury to the humerus around the shoulder joint

4 The patient with a dislocation may have deformation injury to the humerus around the shoulder joint. Tearing of ligaments in the shoulder may not indicate a dislocation. Mild tears in the shoulder muscles may not indicate dislocation. Severe tearing in the shoulder muscles may not indicate dislocation. Text Reference - p. 1508

A patient with an extensive tear of the muscles around the shoulder underwent acromioplasty. Which postoperative activity may result in arthrofibrosis? 1 Lifting weights after six months 2 Beginning physical therapy on the first postoperative day 3 Performing pendulum exercises on the first postoperative day 4 Keeping the shoulder immobilized for prolonged periods after surgery

4 The shoulder of a patient who underwent acromioplasty should not be immobilized for a prolonged time, because it may cause arthrofibrosis. Weight lifting is recommended if the patient has fully recovered. Physical therapy and pendulum exercises begin from the first postoperative day to promote healing and prevent complications. Test-Taking Tip: Multiple-choice questions can be challenging, because students think that they will recognize the right answer when they see it or that the right answer will somehow stand out from the other choices. This is a dangerous misconception. The more carefully the question is constructed, the more each of the choices will seem like the correct response. Text Reference - p. 1510

4. The patient with osteoporosis had a spontaneous hip fracture. How should the nurse document this before the x-ray results return? a. Open fracture b. Oblique fracture c. Pathologic fracture d. Greenstick fracture

4. c. A pathologic fracture is a spontaneous fracture at the site of bone disease, such as osteoporosis. An open fracture is when there is communication with the external environment. The oblique fracture has a slanted fracture line. A greenstick fracture is splintered on one side and the other side is bent.

40. A 65-year-old patient has undergone a right total hip arthroplasty with a cemented prosthesis for treatment of severe osteoarthritis of the hip. What is included in the activity the nurse anticipates for the patient on the patient's first or second postoperative day? a. Transfer from the bed to the chair twice a day only b. Turning from the back to the unaffected side q2hr only c. Crutch walking with non-weight bearing on the operative leg d. Ambulation and weight bearing on the right leg with a walker

40. d. Physical therapy is initiated on the first postoperative day with ambulation and weight bearing using a walker for a patient with a cemented prosthesis and non-weight bearing on the operative side for an uncemented prosthesis. In addition, the patient sits in the chair at least twice a day and is turned to both sides and back with the operative leg supported.

41. When positioning the patient after a total hip arthroplasty with a posterior approach, it is important that the nurse maintain the affected extremity in what position? a. Adduction and flexion b. Abduction and extension c. Abduction and internal rotation d. Adduction and external rotation

41. b. Following a total hip arthroplasty with a posterior approach, during hospitalization an abduction pillow is placed between the legs to maintain abduction and the leg is extended. Extremes of internal rotation, adduction, and 90-degree flexion of the hip must be avoided for 4 to 6 weeks postoperatively to prevent dislocation of the prosthesis.

42. Following a knee arthroplasty, a patient has a continuous passive motion machine for the affected joint. The nurse explains to the patient that this device is used for what purpose? a. To relieve edema and pain at the incision site b. To promote early joint mobility and increase knee flexion c. To prevent venous stasis and the formation of a deep venous thrombosis d. To improve arterial circulation to the affected extremity to promote healing

42. b. Continuous passive motion machines are frequently used following knee surgery to promote earlier joint mobility. Because joint dislocation is not a problem with knee replacements, early exercise with straight leg raises and gentle ROM is also encouraged postoperatively.

Priority Decision: A patient with severe ulnar deviation of the hands undergoes an arthroplasty with reconstruction and replacement of finger joints. Postoperatively, what is it most important for the nurse to do? a. Position the fingers lower than the elbow. b. Perform neurovascular assessments of the fingers q2-4hr. c. Encourage the patient to gently flex, extend, abduct, and adduct the fingers q4hr. d. Remind the patient that function of the hands is more important than their cosmetic appearance.

43. b. Neurovascular checks of the fingers following surgery of the hands are essential to detect compromised vascular and neurologic function caused by trauma or edema. Postoperatively, the hands are elevated with a bulky dressing in place and when the dressing is removed, a guided splinting program is started. Exercises are performed three to four times a day when the splints are removed and the patient is discharged. Before surgery, it must be made clear to the patient that the goal of the surgery is to restore function related to grasp, pinch, stability, and strength and the hands will not necessarily have good cosmetic appearance.

44. Priority Decision: Following change-of-shift handoff, which patient should the nurse assess first? a. A 58-year-old male experiencing phantom pain and requesting analgesic b. A 72-year-old male being transferred to a skilled nursing unit following repair of a hip fracture c. A 25-year-old female in left leg skeletal traction asking for the weights to be lifted for a few minutes d. A 68-year-old male with a new lower leg cast complaining that the cast is too tight and he cannot feel his toes

44. d. The patient with a tight cast may be at risk for neurovascular compromise (impaired circulation and peripheral nerve damage) and should be assessed first. The other patients should be seen as soon as possible. Providing analgesia for the patient with phantom pain would be the next priority. The patient in skeletal traction needs explanation of the purpose and functioning of the traction. She may need analgesia or muscle relaxants to help tolerate the traction.

The patient works on a computer 8 hours each day. What kind of repetitive strain injury would be expected in this patient? a. Meniscus injury b. Rotator cuff injury c. Radial-ulnar fracture d. Carpal tunnel syndrome

5. d. Carpal tunnel syndrome would be expected related to the continuous wrist movements. Injuries of the menisci, which are fibrocartilage in the knee, are common with athletes. Radial-ulnar fractures are seen with great force such as a car accident or a fall. Rotator cuff injuries occur with sudden adduction forces applied to the cuff while the arm is held in abduction. They are commonly seen with repetitive overhead motions.

6. The athlete comes to the clinic with bursitis. What does the nurse know happens to the tissue to cause pain when bursitis occurs? a. Tearing of a ligament b. Stretching of muscle and fascia sheath c. Inflammation of synovial membrane sac at friction sites d. Incomplete separation of articular surfaces of joint caused by ligament injury

6. c. Bursitis is inflammation of synovial membrane sac at friction sites. Tearing of a ligament is a sprain. Stretching of muscle and fascia sheath is a strain. Incomplete separation of articular surfaces of joints caused by ligament injury is subluxation.

7. Application of RICE (rest, ice, compression, and elevation) is indicated for initial management of which type of injury? a. Muscle spasms b. Sprains and strains c. Repetitive strain injury d. Dislocations and subluxations

7. b. Application of cold, compression, and elevation are indicated to prevent edema resulting from sprain and some strain injuries. Muscle spasms are usually treated with heat applications and massage and repetitive strain injuries require cessation of the precipitating activity and physical therapy. Dislocations or subluxations require immediate reduction and immobilization to prevent vascular impairment and bone cell death.

What should be included in the management during the first 48 hours after an acute soft tissue injury of the ankle (select all that apply)? a. Use of elastic wrap b. Initial immobilization and rest c. Elevation of ankle above the heart d. Alternating the use of heat and cold e. Administration of antiinflammatory drugs

8. a, b, c, e. Consider the principle of RICE. Rest: movement should be restricted. Ice: cold should be used to promote vasoconstriction and to reduce edema. C: compression helps to decrease swelling. E: elevate the extremity above the level of the heart. Mild nonsteroidal antiinflammatory drugs (NSAIDs) may be needed to manage pain. Warm, moist compresses may be used after 48 hours for 20 to 30 minutes at a time to reduce swelling and provide comfort.

9. The patient had a fracture. At 3 weeks to 6 months there is clinical union, and this is the first stage of healing that is sufficient to prevent movement of the fracture site when the bones are gently stressed. How is this stage of fracture healing documented? a. Ossification b. Remodeling c. Consolidation d. Callus formation

9. a. Ossification is the stage of fracture healing when there is clinical union and enough strength to prevent movement at the fracture site. Remodeling is the normal function of the bone. Consolidation is when the distance between bone fragments eventually closes and radiologic union first occurs. The callus formation stage appears by the end of the second week of injury when minerals and new bone matrix are deposited in the osteoid that is produced in the granulation tissue stage.

The nurse is caring for a 76-year-old man who has undergone left knee arthroplasty with prosthetic replacement of the knee joint to relieve the pain of severe osteoarthritis. Postoperatively the nurse expects what to be included in the care of the affected leg? a. Progressive leg exercises to obtain 90-degree flexion b. Early ambulation with full weight bearing on the left leg c. Bed rest for 3 days with the left leg immobilized in extension d. Immobilization of the left knee in 30-degree flexion for 2 weeks to prevent dislocation

A Although early ambulation is not done, the patient is encouraged to engage in progressive leg exercises until 90-degree flexion is possible. Because this is painful after surgery, the patient requires good pain management and often the use of a CPM machine. The patient's knee is unlikely to dislocate.

The patient had frostbite on the distal areas of the toes on both feet. The patient is scheduled for amputation of the damaged tissue. Which assessment finding or diagnostic study is the most objective indicator for locating the level of the patient's injury? a. Arteriography showing blood vessels b. Peripheral pulse assessment bilaterally c. Patches of black, indurated, and cold tissue d. Bilateral pale and cool skin below the ankles

A Arteriography is the most objective study to determine viable tissue for salvage based on perfusion because actual blood flow through the tissues is observed in real time. It is considered the gold standard for evaluating arterial perfusion. Bilateral peripheral pulse assessment, areas of black, indurated, and cold tissue, and bilateral pale and cool skin all identify the lack of tissue perfusion, but not the specific area where tissue perfusion stops and amputation needs to occur.

A 28-year-old woman with a fracture of the proximal left tibia in a long leg cast complains of severe pain and a prickling sensation in the left foot. The toes on the left foot are pale and cool. Which action should the nurse take? a. Notify the health care provider immediately. b. Elevate the left leg above the level of the heart. c. Administer prescribed morphine sulfate intravenously. d. Apply ice packs to the left proximal tibia over the cast.

A Clinical manifestations of compartment syndrome include (1) paresthesia, (2) pain distal to the injury that is not relieved by opioid analgesics and pain on passive stretch of muscle traveling through the compartment, (3) pressure increases in the compartment, (4) pallor, coolness, and loss of normal color of the extremity, (5) paralysis or loss of function, and (6) pulselessness or diminished/absent peripheral pulses. Pain unrelieved by drugs and out of proportion to the level of injury is one of the first indications of impending compartment syndrome. Pulselessness and paralysis (in particular) are later signs of compartment syndrome. Notify the health care provider immediately of a patient's changing condition. Because elevation of the extremity may lower venous pressure and slow arterial perfusion, the extremity should not be elevated above heart level. Similarly, the application of cold compresses may result in vasoconstriction and exacerbate compartment syndrome.

An injured soldier had an amputation of his left leg and is reporting shooting pain and heaviness in the area of his missing leg. What would be the best response by the nurse for this patient? a. Use mirror therapy. b. Give opioid analgesics. c. Rebandage the residual limb. d. Show the patient the leg is gone.

A Mirror therapy has been shown to reduce phantom limb pain in some patients. Opioid analgesics, rebandaging the residual limb, and showing the patient that the leg is gone will not decrease phantom limb pain.

The patient had frostbite on the distal areas of the toes on both feet. The patient is scheduled for amputation of the damaged tissue. Which assessment finding or diagnostic study is the most objective indicator for locating the level of the patient's injury? A Arteriography showing blood vessels B Peripheral pulse assessment bilaterally C Patches of black, indurated, and cold tissue D Bilateral pale and cool skin below the ankles

A Arteriography showing blood vessels Arteriography is the most objective study to determine viable tissue for salvage based on perfusion because actual blood flow through the tissues is observed in real time. It is considered the gold standard for evaluating arterial perfusion. Bilateral peripheral pulse assessment, areas of black, indurated, and cold tissue, and bilateral pale and cool skin all identify the lack of tissue perfusion, but not the specific area where tissue perfusion stops and amputation needs to occur.

The patient had a lumbar spine arthrodesis. What should the nurse include in discharge teaching (select all that apply)? A Do not smoke cigarettes. B You should not walk for 3 weeks. C You must wear your brace at all times. D You may drive as soon as you feel like it. E Do not bend your spine until your follow-up appointment.

A Do not smoke cigarettes. E Do not bend your spine until your follow-up appointment. After a spinal fusion, the patient should not smoke cigarettes as nonunion tends to occur more often with smokers. Preventing pressure by not bending or twisting the spine or lifting more than 10 pounds will facilitate healing. The amount of time that is needed will be determined by the surgeon at follow-up appointments, but healing usually takes 6 to 9 months. An important aspect of healing is progressively increasing walking, which increases circulation of nutrients and oxygen for healing. If a brace is ordered to protect the surgical area, the surgeon will order how often the patient should wear it. Driving is not done until the surgeon allows it and the patient is no longer taking opioids for pain.

The nurse is caring for a 76-year-old man who has undergone left knee arthroplasty with prosthetic replacement of the knee joint to relieve the pain of severe osteoarthritis. Postoperatively the nurse expects what to be included in the care of the affected leg? A Progressive leg exercises to obtain 90-degree flexion B Early ambulation with full weight bearing on the left leg C Bed rest for 3 days with the left leg immobilized in extension D Immobilization of the left knee in 30-degree flexion for 2 weeks to prevent dislocation

A Progressive leg exercises to obtain 90-degree flexion Although early ambulation is not done, the patient is encouraged to engage in progressive leg exercises until 90-degree flexion is possible. Because this is painful after surgery, the patient requires good pain management and often the use of a CPM machine. The patient's knee is unlikely to dislocate.

1 Ligament injury is the tearing or stretching of a ligament as a result of inversion, eversion, shearing, or torque applied to a joint. Overuse syndrome results from prolonged force or repetitive movements and awkward postures. A tear within the muscle or tendinoligamentous structures around the shoulder is a rotator cuff tear. Impingement syndrome is the entrapment of soft tissue structures under the coracoacromial arch of the shoulder. Text Reference - p. 1509

A patient experiences a tearing of a ligament as a result of shearing stresses applied to a joint. What diagnosis does the nurse anticipate? 1 Ligament injury 2 Rotator cuff tear 3 Overuse syndrome 4 Impingement syndrome

3 Subluxation, also known as dislocation, may be assessed by means of palpation of the space between the head of the bone and the cavity where it is normally located. Subluxation results in partial loss of function and intense pain. A subluxation is not a fracture—there is no break in bone integrity—but a subluxation is treated similarly to a fracture. With subluxation, there is no rupture in the integrity of the bone. Subluxation may be described as a misalignment, but this is not an accurate term. Text Reference - p. 1507

A patient is found to have a partially dislocated shoulder. How should the nurse document this finding? 1 Fracture 2 Rupture 3 Subluxation 4 Misalignment

The nurse admits a 55-year-old female with multiple sclerosis to a long-term care facility. Which finding is of most immediate concern to the nurse? A. Ataxic gait B. Radicular pain C. Severe fatigue D. Urinary retention

A. Ataxic gait An ataxic gait is a staggering, uncoordinated gait. Fall risk is the highest in individuals with gait instability or visual or cognitive impairments. The other signs and symptoms (e.g., fatigue, urinary retention, radicular pain) may also occur in the patient with multiple sclerosis and need to be managed, but are not the priority.

The nurse admits a 55-year-old female with multiple sclerosis to a long-term care facility. Which finding is of most immediate concern to the nurse? A. Ataxic gait B. Radicular pain C. Severe fatigue D. Urinary retention

A. Ataxis gait An ataxic gait is a staggering, uncoordinated gait. Fall risk is the highest in individuals with gait instability or visual or cognitive impairments. The other signs and symptoms (e.g., fatigue, urinary retention, radicular pain) may also occur in the patient with multiple sclerosis and need to be managed, but are not the priority.

A 50-year-old patient is reporting a sore shoulder after raking the yard. The nurse should suspect which problem? A. Bursitis B. Fasciitis C. Sprained ligament D. Achilles tendonitis

A. Bursitis Bursitis is common in adults over age 40 and with repetitive motion, such as raking. Plantar fasciitis frequently occurs as a stabbing pain at the heel caused by straining the ligament that supports the arch. Achilles tendonitis is an inflammation of the tendon that attaches the calf muscle to the heel bone, not the shoulder, and causes pain with walking or running. A sprained ligament occurs when a ligament is stretched or torn from a direct injury or sudden twisting of the joint, not repetitive motion.

Which finding from a patient's right knee arthrocentesis will be of concern to the nurse? a. Cloudy fluid b. Scant thin fluid c. Pale yellow fluid d. Straw-colored fluid

A. Cloudy fluid

When working with patients, the nurse knows that patients have the most difficulties with diarthrodial joints. Which joints are included in this group of joints? (Select all that apply.) A. Hinge joint of the knee B. Ligaments joining the vertebrae C. Fibrous connective tissue of the skull D. Ball and socket joint of the shoulder or hip E. Cartilaginous connective tissue of the pubis joint

A. Hinge joint of the knee D. Ball and socket joint of the shoulder or hip The diarthrodial joints include the hinge joint of the knee and elbow, the ball and socket joint of the shoulder and hip, the pivot joint of the radioulnar joint, and the condyloid, saddle, and gliding joints of the wrist and hand. The ligaments and cartilaginous connective tissue joining the vertebrae and pubis joint and the fibrous connective tissue of the skull are synarthrotic joints.

The nurse is performing a musculoskeletal assessment of an 81-year-old female patient whose mobility has been progressively decreasing in recent months. How should the nurse best assess the patient's range of motion (ROM) in the affected leg? A. Observe the patient's unassisted ROM in the affected leg. B. Perform passive ROM, asking the patient to report any pain. C. Ask the patient to lift progressive weights with the affected leg. D. Move both of the patient's legs from a supine position to full flexion.

A. Observe the patient's unassisted ROM in the affected leg. Passive ROM should be performed with extreme caution and may be best avoided when assessing older patients. Observing the patient's active ROM is more accurate and safe than asking the patient to lift weights with her legs.

The nurse is performing a musculoskeletal assessment of an 81-year-old female patient whose mobility has been progressively decreasing in recent months. How should the nurse best assess the patient's range of motion (ROM) in the affected leg? A. Observe the patient's unassisted ROM in the affected leg. B. Perform passive ROM, asking the patient to report any pain. C. Ask the patient to lift progressive weights with the affected leg. D. Move both of the patient's legs from a supine position to full flexion.

A. Observe the patient's unassisted ROM in the affected leg. Passive ROM should be performed with extreme caution and may be best avoided when assessing older patients. Observing the patient's active ROM is more accurate and safe than asking the patient to lift weights with her legs.

A patient has a new order for magnetic resonance imaging (MRI) to evaluate for left femur osteomyelitis after a hip replacement surgery. Which information indicates that the nurse should consult with the health care provider before scheduling the MRI? a. The patient has a pacemaker. b. The patient is claustrophobic. c. The patient wears a hearing aid. d. The patient is allergic to shellfish.

A. Patients with permanent pacemakers cannot have MRI because of the force exerted by the magnetic field on metal objects. An open MRI will not cause claustrophobia. The patient will need to be instructed to remove the hearing aid before the MRI, but this does not require consultation with the health care provider. Because contrast medium will not be used, shellfish allergy is not a contraindication to MRI.

Which nursing action is correct when performing the straight-leg raising test for an ambulatory patient with back pain? a. Raise the patient's legs to a 60-degree angle from the bed. b. Place the patient initially in the prone position on the exam table. c. Have the patient dangle both legs over the edge of the exam table. d. Instruct the patient to elevate the legs and tense the abdominal muscles.

A. Raise the patient's legs to a 60-degree angle from the bed.

A patient with a fractured femur experiences the complication of malunion. The nurse recognizes that what happens with this complication? A. The fracture heals is an unsatisfactory position. B. The fracture fails to heal properly despite treatment. C. Fracture healing progresses more slowly than expected. D. Loss of bone substances occurs as a result of immobilization.

A. The fracture heals in an unsatisfactory position. A malunion occurs when the heals in the expected time but in an unsatisfactory position, possibly resulting in deformity or dysfunction. Nonunion occurs when the fracture fails to heal properly despite treatment and delayed union id healing of the fracture at a slower rate than expected. In posttraumatic osteoporosis, the loss of bone substances occurs as a result of immobilization.

A patient has a new order for magnetic resonance imaging (MRI) to evaluate for left femur osteomyelitis after a hip replacement surgery. Which information indicates that the nurse should consult with the health care provider before scheduling the MRI? a. The patient has a pacemaker. b. The patient is claustrophobic. c. The patient wears a hearing aid. d. The patient is allergic to shellfish.

A. The patient has a pacemaker.

30. Which nursing action for a patient who has had right hip replacement surgery can the nurse delegate to experienced unlicensed assistive personnel (UAP)? a. Reposition the patient every 1 to 2 hours. b. Assess for skin irritation on the patient's back. c. Teach the patient quadriceps-setting exercises. d. Determine the patient's pain level and tolerance.

ANS: A Repositioning of orthopedic patients is within the scope of practice of UAP (after they have been trained and evaluated in this skill). The other actions should be done by licensed nursing staff members.

In which order will the nurse take these actions when caring for a patient with left leg fractures after a motor vehicle accident? Put a comma and space between each answer choice (a, b, c, d, etc.) ____________________ a. Obtain x-rays. b. Check pedal pulses. c. Assess lung sounds. d. Take blood pressure. e. Apply splint to the leg. f. Administer tetanus prophylaxis.

ANS: C, D, B, E, A, F The initial actions should be to ensure that airway, breathing, and circulation are intact. This should be followed by checking the neurovascular status of the leg (before and after splint application). Application of a splint to immobilize the leg should be done before sending the patient for x-rays. The tetanus prophylaxis is the least urgent of the actions.

15. A patient with a right lower leg fracture will be discharged home with an external fixation device in place. Which information will the nurse teach? a. "You will need to check and clean the pin insertion sites daily." b. "The external fixator can be removed for your bath or shower." c. "You will need to remain on bed rest until bone healing is complete." d. "Prophylactic antibiotics are used until the external fixator is removed."

ANS: A Pin insertion sites should be cleaned daily to decrease the risk for infection at the site. An external fixator allows the patient to be out of bed and avoid the risks of prolonged immobility. The device is surgically placed and is not removed until the bone is stable. Prophylactic antibiotics are not routinely given when an external fixator is used.

15. A patient with a right lower leg fracture will be discharged home with an external fixation device in place. Which information will the nurse teach? a. "You will need to check and clean the pin insertion sites daily." b. "The external fixator can be removed for your bath or shower." c. "You will need to remain on bed rest until bone healing is complete." d. "Prophylactic antibiotics are used until the external fixator is removed."

ANS: A Pin insertion sites should be cleaned daily to decrease the risk for infection at the site. An external fixator allows the patient to be out of bed and avoid the risks of prolonged immobility. The device is surgically placed and is not removed until the bone is stable. Prophylactic antibiotics are not routinely given when an external fixator is used.

3. The occupational health nurse will teach the patient whose job involves many hours of typing about the need to a. obtain a keyboard pad to support the wrist. b. do stretching exercises before starting work. c. wrap the wrists with compression bandages every morning. d. avoid using nonsteroidal antiinflammatory drugs (NSAIDs) for pain.

ANS: A Repetitive strain injuries caused by prolonged times working at a keyboard can be prevented by the use of a pad that will keep the wrists in a straight position. Stretching exercises during the day may be helpful, but these would not be needed before starting. Use of a compression bandage is not needed, although a splint may be used for carpal tunnel syndrome. NSAIDs are appropriate to use to decrease swelling.

3. The occupational health nurse will teach the patient whose job involves many hours of typing about the need to a. obtain a keyboard pad to support the wrist. b. do stretching exercises before starting work. c. wrap the wrists with compression bandages every morning. d. avoid using nonsteroidal antiinflammatory drugs (NSAIDs) for pain.

ANS: A Repetitive strain injuries caused by prolonged times working at a keyboard can be prevented by the use of a pad that will keep the wrists in a straight position. Stretching exercises during the day may be helpful, but these would not be needed before starting. Use of a compression bandage is not needed, although a splint may be used for carpal tunnel syndrome. NSAIDs are appropriate to use to decrease swelling.

30. Which nursing action for a patient who has had right hip replacement surgery can the nurse delegate to experienced unlicensed assistive personnel (UAP)? a. Reposition the patient every 1 to 2 hours. b. Assess for skin irritation on the patient's back. c. Teach the patient quadriceps-setting exercises. d. Determine the patient's pain level and tolerance.

ANS: A Repositioning of orthopedic patients is within the scope of practice of UAP (after they have been trained and evaluated in this skill). The other actions should be done by licensed nursing staff members.

12. A 32-year-old patient who has had an open reduction and internal fixation (ORIF) of left lower leg fractures continues to complain of severe pain in the leg 15 minutes after receiving the prescribed IV morphine. Pulses are faintly palpable and the foot is cool. Which action should the nurse take next? a. Notify the health care provider. b. Assess the incision for redness. c. Reposition the left leg on pillows. d. Check the patient's blood pressure.

ANS: A The patient's clinical manifestations suggest compartment syndrome and delay in diagnosis and treatment may lead to severe functional impairment. The data do not suggest problems with blood pressure or infection. Elevation of the leg will decrease arterial flow and further reduce perfusion.

12. A 32-year-old patient who has had an open reduction and internal fixation (ORIF) of left lower leg fractures continues to complain of severe pain in the leg 15 minutes after receiving the prescribed IV morphine. Pulses are faintly palpable and the foot is cool. Which action should the nurse take next? a. Notify the health care provider. b. Assess the incision for redness. c. Reposition the left leg on pillows. d. Check the patient's blood pressure.

ANS: A The patient's clinical manifestations suggest compartment syndrome and delay in diagnosis and treatment may lead to severe functional impairment. The data do not suggest problems with blood pressure or infection. Elevation of the leg will decrease arterial flow and further reduce perfusion.

A patient arrives in the emergency department with ankle swelling and severe pain after twisting the ankle playing soccer. Which of these prescribed collaborative interventions will the nurse implement first? a. Wrap the ankle and apply an ice pack. b. Administer naproxen (Naprosyn) 500 mg PO. c. Give acetaminophen with codeine (Tylenol #3). d. Take the patient to the radiology department for x-rays.

ANS: A Immediate care after a sprain or strain injury includes the application of cold and compression to the injury to minimize swelling. The other actions should be taken after the ankle is wrapped with a compression bandage and ice is applied.

A 20-year-old baseball pitcher has an arthroscopic repair of a rotator cuff injury performed in same-day surgery. When the nurse plans postoperative teaching for the patient, which information will be included? a. "You have an appointment with a physical therapist for tomorrow." b. "You can still play baseball but you will not be able to return to pitching." c. "The doctor will use the drop-arm test to determine the success of surgery." d. "Leave the shoulder immobilizer on for the first few days to minimize pain."

ANS: A Physical therapy after a rotator cuff repair begins on the first postoperative day to prevent "frozen shoulder." A shoulder immobilizer is used immediately after the surgery, but leaving the arm immobilized for several days would lead to loss of range of motion (ROM). The drop-arm test is used to test for rotator cuff injury, but not after surgery. The patient may be able to return to pitching after rehabilitation.

A patient with a right lower leg fracture will be discharged home with an external fixation device in place. Which information will the nurse teach? a. "You will need to check and clean the pin insertion sites daily." b. "The external fixator can be removed for your bath or shower." c. "You will need to remain on bed rest until bone healing is complete." d. "Prophylactic antibiotics are used until the external fixator is removed."

ANS: A Pin insertion sites should be cleaned daily to decrease the risk for infection at the site. An external fixator allows the patient to be out of bed and avoid the risks of prolonged immobility. The device is surgically placed and is not removed until the bone is stable. Prophylactic antibiotics are not routinely given when an external fixator is used.

A patient with lower leg fracture has an external fixation device in place and is scheduled for discharge. Which information will the nurse include in the discharge teaching? a. "You will need to assess and clean the pin insertion sites daily." b. "The external fixator can be removed during the bath or shower." c. "You will need to remain on bed rest until bone healing is complete." d. "Prophylactic antibiotics are used until the external fixator is removed."

ANS: A Pin insertion sites should be cleaned daily to decrease the risk for infection at the site. An external fixator allows the patient to be out of bed and avoid the risks of prolonged immobility. The device is surgically placed and is not removed until the bone is stable. Prophylactic antibiotics are not routinely given when an external fixator is used.

15. A patient with a right lower leg fracture will be discharged home with an external fixation device in place. Which information will the nurse teach? a. "You will need to check and clean the pin insertion sites daily." b. "The external fixator can be removed for your bath or shower." c. "You will need to remain on bed rest until bone healing is complete." d. "Prophylactic antibiotics are used until the external fixator is removed."

ANS: A Pin insertion sites should be cleaned daily to decrease the risk for infection at the site. An external fixator allows the patient to be out of bed and avoid the risks of prolonged immobility. The device is surgically placed and is not removed until the bone is stable. Prophylactic antibiotics are not routinely given when an external fixator is used. DIF: Cognitive Level: Apply (application) REF: 1516 TOP: Nursing Process: Implementation MSC: NCLEX: Physiological Integrity

The occupational health nurse will teach the patient whose job involves many hours of typing about the need to a. obtain a keyboard pad to support the wrist. b. do stretching exercises before starting work. c. wrap the wrists with compression bandages every morning. d. avoid using nonsteroidal antiinflammatory drugs (NSAIDs) for pain.

ANS: A Repetitive strain injuries caused by prolonged times working at a keyboard can be prevented by the use of a pad that will keep the wrists in a straight position. Stretching exercises during the day may be helpful, but these would not be needed before starting. Use of a compression bandage is not needed, although a splint may be used for carpal tunnel syndrome. NSAIDs are appropriate to use to decrease swelling.

Which nursing action for a patient who has had right hip replacement surgery can the nurse delegate to experienced unlicensed assistive personnel (UAP)? a. Reposition the patient every 1 to 2 hours. b. Assess for skin irritation on the patient's back. c. Teach the patient quadriceps-setting exercises. d. Determine the patient's pain level and tolerance.

ANS: A Repositioning of orthopedic patients is within the scope of practice of UAP (after they have been trained and evaluated in this skill). The other actions should be done by licensed nursing staff members.

30. Which nursing action for a patient who has had right hip replacement surgery can the nurse delegate to experienced unlicensed assistive personnel (UAP)? a. Reposition the patient every 1 to 2 hours. b. Assess for skin irritation on the patient's back. c. Teach the patient quadriceps-setting exercises. d. Determine the patient's pain level and tolerance.

ANS: A Repositioning of orthopedic patients is within the scope of practice of UAP (after they have been trained and evaluated in this skill). The other actions should be done by licensed nursing staff members. DIF: Cognitive Level: Apply (application) REF: 1514 OBJ: Special Questions: Delegation TOP: Nursing Process: Planning MSC: NCLEX: Safe and Effective Care Environment

A patient with ulnar drift caused by rheumatoid arthritis (RA) is scheduled for an arthroplasty of the hand. Which patient statement to the nurse indicates realistic expectation for the surgery? a. "I will be able to use my fingers to grasp objects better." b. "I will not have to do as many hand exercises after the surgery." c. "This procedure will prevent further deformity in my hands and fingers." d. "My fingers will appear more normal in size and shape after this surgery."

ANS: A The goal of hand surgery in RA is to restore function, not to correct for cosmetic deformity or treat the underlying process. Hand exercises will be prescribed after the surgery.

When doing discharge teaching for a patient who has had a repair of a fractured mandible, the nurse will include information about a. when and how to cut the immobilizing wires. b. self-administration of nasogastric tube feedings. c. the use of sterile technique for dressing changes. d. the importance of including high-fiber foods in the diet.

ANS: A The jaw will be wired for stabilization, and the patient should know what emergency situations require that the wires be cut to protect the airway. There are no dressing changes for this procedure. The diet is liquid, and patients are not able to chew high fiber foods. Initially, the patient may receive nasogastric tube feedings, but by discharge the patient will swallow liquid through a straw.

Which statement by a patient who has had an above-the-knee amputation indicates that the nurse's discharge teaching has been effective? a. "I should lay on my abdomen for 30 minutes 3 or 4 times a day." b. "I should elevate my residual limb on a pillow 2 or 3 times a day." c. "I should change the limb sock when it becomes soiled or stretched out." d. "I should use lotion on the stump to prevent drying and cracking of the skin."

ANS: A The patient lies in the prone position several times daily to prevent flexion contractures of the hip. The limb sock should be changed daily. Lotion should not be used on the stump. The residual limb should not be elevated because this would encourage flexion contracture.

Before assisting a patient with ambulation on the day after a total hip replacement, which action is most important for the nurse to take? a. Administer the ordered oral opioid pain medication. b. Instruct the patient about the benefits of ambulation. c. Ensure that the incisional drain has been discontinued. d. Change the hip dressing and document the wound appearance.

ANS: A The patient should be adequately medicated for pain before any attempt to ambulate. Instructions about the benefits of ambulation may increase the patient's willingness to ambulate, but decreasing pain with ambulation is more important. The presence of an incisional drain or timing of dressing change will not affect ambulation.

A 32-year-old patient who has had an open reduction and internal fixation (ORIF) of left lower leg fractures continues to complain of severe pain in the leg 15 minutes after receiving the prescribed IV morphine. Pulses are faintly palpable and the foot is cool. Which action should the nurse take next? a. Notify the health care provider. b. Assess the incision for redness. c. Reposition the left leg on pillows. d. Check the patient's blood pressure.

ANS: A The patient's clinical manifestations suggest compartment syndrome and delay in diagnosis and treatment may lead to severe functional impairment. The data do not suggest problems with blood pressure or infection. Elevation of the leg will decrease arterial flow and further reduce perfusion.

A patient who has had an open reduction and internal fixation (ORIF) of left lower leg fractures complains of constant severe pain in the leg, which is unrelieved by the prescribed morphine. Pulses are faintly palpable and the foot is cool. Which action should the nurse take next? a. Notify the health care provider. b. Assess the incision for redness. c. Reposition the left leg on pillows. d. Check the patient's blood pressure.

ANS: A The patient's clinical manifestations suggest compartment syndrome and delay in diagnosis and treatment may lead to severe functional impairment. The data do not suggest problems with blood pressure or infection. Elevation of the leg will decrease arterial flow and further reduce perfusion.

12. A 32-year-old patient who has had an open reduction and internal fixation (ORIF) of left lower leg fractures continues to complain of severe pain in the leg 15 minutes after receiving the prescribed IV morphine. Pulses are faintly palpable and the foot is cool. Which action should the nurse take next? a. Notify the health care provider. b. Assess the incision for redness. c. Reposition the left leg on pillows. d. Check the patient's blood pressure.

ANS: A The patient's clinical manifestations suggest compartment syndrome and delay in diagnosis and treatment may lead to severe functional impairment. The data do not suggest problems with blood pressure or infection. Elevation of the leg will decrease arterial flow and further reduce perfusion. DIF: Cognitive Level: Apply (application) REF: 1522 TOP: Nursing Process: Implementation MSC: NCLEX: Physiological Integrity

8. Which nursing intervention will be included in the plan of care after a patient with a right femur fracture has a hip spica cast applied? a. Avoid placing the patient in prone position. b. Ask the patient about abdominal discomfort. c. Discuss remaining on bed rest for several weeks. d. Use the cast support bar to reposition the patient.

ANS: B Assessment of bowel sounds, abdominal pain, and nausea and vomiting will detect the development of cast syndrome. To avoid breakage, the support bar should not be used for repositioning. After the cast dries, the patient can begin ambulating with the assistance of physical therapy personnel and may be turned to the prone position.

8. Which nursing intervention will be included in the plan of care after a patient with a right femur fracture has a hip spica cast applied? a. Avoid placing the patient in prone position. b. Ask the patient about abdominal discomfort. c. Discuss remaining on bed rest for several weeks. d. Use the cast support bar to reposition the patient.

ANS: B Assessment of bowel sounds, abdominal pain, and nausea and vomiting will detect the development of cast syndrome. To avoid breakage, the support bar should not be used for repositioning. After the cast dries, the patient can begin ambulating with the assistance of physical therapy personnel and may be turned to the prone position.

6. The nurse will instruct the patient with a fractured left radius that the cast will need to remain in place a. for several months. b. for at least 3 weeks. c. until swelling of the wrist has resolved. d. until x-rays show complete bony union.

ANS: B Bone healing starts immediately after the injury, but since ossification does not begin until 3 weeks post injury, the cast will need to be worn for at least 3 weeks. Complete union may take up to a year. Resolution of swelling does not indicate bone healing

6. The nurse will instruct the patient with a fractured left radius that the cast will need to remain in place a. for several months. b. for at least 3 weeks. c. until swelling of the wrist has resolved. d. until x-rays show complete bony union.

ANS: B Bone healing starts immediately after the injury, but since ossification does not begin until 3 weeks postinjury, the cast will need to be worn for at least 3 weeks. Complete union may take up to a year. Resolution of swelling does not indicate bone healing.

14. Which action will the nurse take in order to evaluate the effectiveness of Buck's traction for a 62-year-old patient who has an intracapsular fracture of the right femur? a. Check peripheral pulses. b. Ask about hip pain level. c. Assess for hip contractures. d. Monitor for hip dislocation.

ANS: B Buck's traction keeps the leg immobilized and reduces painful muscle spasm. Hip contractures and dislocation are unlikely to occur in this situation. The peripheral pulses will be assessed, but this does not help in evaluating the effectiveness of Buck's traction.

14. Which action will the nurse take in order to evaluate the effectiveness of Buck's traction for a 62-year-old patient who has an intracapsular fracture of the right femur? a. Check peripheral pulses. b. Ask about hip pain level. c. Assess for hip contractures. d. Monitor for hip dislocation.

ANS: B Buck's traction keeps the leg immobilized and reduces painful muscle spasm. Hip contractures and dislocation are unlikely to occur in this situation. The peripheral pulses will be assessed, but this does not help in evaluating the effectiveness of Buck's traction.

42. Which information obtained by the nurse about a 29-year-old patient with a lumbar vertebral compression fracture is most important to report to the health care provider? a. Patient refuses to be turned due to back pain. b. Patient has been incontinent of urine and stool. c. Patient reports lumbar area tenderness to palpation. d. Patient frequently uses oral corticosteroids to treat asthma.

ANS: B Changes in bowel or bladder function indicate possible spinal cord compression and should be reported immediately because surgical intervention may be needed. The other findings are also pertinent but are consistent with the patient's diagnosis and do not require immediate intervention

42. Which information obtained by the nurse about a 29-year-old patient with a lumbar vertebral compression fracture is most important to report to the health care provider? a. Patient refuses to be turned due to back pain. b. Patient has been incontinent of urine and stool. c. Patient reports lumbar area tenderness to palpation. d. Patient frequently uses oral corticosteroids to treat asthma.

ANS: B Changes in bowel or bladder function indicate possible spinal cord compression and should be reported immediately because surgical intervention may be needed. The other findings are also pertinent but are consistent with the patient's diagnosis and do not require immediate intervention.

29. A young man arrives in the emergency department with ankle swelling and severe pain after twisting his ankle playing basketball. Which of these prescribed collaborative interventions will the nurse implement first? a. Take the patient to have x-rays. b. Wrap the ankle and apply an ice pack. c. Administer naproxen (Naprosyn) 500 mg PO. d. Give acetaminophen with codeine (Tylenol #3).

ANS: B Immediate care after a sprain or strain injury includes the application of cold and compression to the injury to minimize swelling. The other actions should be taken after the ankle is wrapped with a compression bandage and ice is applied.

29. A young man arrives in the emergency department with ankle swelling and severe pain after twisting his ankle playing basketball. Which of these prescribed collaborative interventions will the nurse implement first? a. Take the patient to have x-rays. b. Wrap the ankle and apply an ice pack. c. Administer naproxen (Naprosyn) 500 mg PO. d. Give acetaminophen with codeine (Tylenol #3).

ANS: B Immediate care after a sprain or strain injury includes the application of cold and compression to the injury to minimize swelling. The other actions should be taken after the ankle is wrapped with a compression bandage and ice is applied.

26. A patient is being discharged 4 days after hip replacement surgery using the posterior approach. Which patient action requires immediate intervention by the nurse? a. The patient uses crutches with a swing-to gait. b. The patient leans over to pull shoes and socks on. c. The patient sits straight up on the edge of the bed. d. The patient bends over the sink while brushing teeth.

ANS: B Leaning over would flex the hip at greater than 90 degrees and predispose the patient to hip dislocation. The other patient actions are appropriate and do not require any immediate action by the nurse to protect the patient.

26. A patient is being discharged 4 days after hip replacement surgery using the posterior approach. Which patient action requires immediate intervention by the nurse? a. The patient uses crutches with a swing-to gait. b. The patient leans over to pull shoes and socks on. c. The patient sits straight up on the edge of the bed. d. The patient bends over the sink while brushing teeth.

ANS: B Leaning over would flex the hip at greater than 90 degrees and predispose the patient to hip dislocation. The other patient actions are appropriate and do not require any immediate action by the nurse to protect the patient.

11. A patient who is to have no weight bearing on the left leg is learning to walk using crutches. Which observation by the nurse indicates that the patient can safely ambulate independently? a. The patient moves the right crutch with the right leg and then the left crutch with the left leg. b. The patient advances the left leg and both crutches together and then advances the right leg. c. The patient uses the bedside chair to assist in balance as needed when ambulating in the room. d. The patient keeps the padded area of the crutch firmly in the axillary area when ambulating.

ANS: B Patients are usually taught to move the crutches and the injured leg forward at the same time and then to move the unaffected leg. Patients are discouraged from using furniture to assist with ambulation. The patient is taught to place weight on the hands, not in the axilla, to avoid nerve damage. If the 2- or 4-point gaits are to be used, the crutch and leg on opposite sides move forward, not the crutch and same-side leg.

11. A patient who is to have no weight bearing on the left leg is learning to walk using crutches. Which observation by the nurse indicates that the patient can safely ambulate independently? a. The patient moves the right crutch with the right leg and then the left crutch with the left leg. b. The patient advances the left leg and both crutches together and then advances the right leg. c. The patient uses the bedside chair to assist in balance as needed when ambulating in the room. d. The patient keeps the padded area of the crutch firmly in the axillary area when ambulating.

ANS: B Patients are usually taught to move the crutches and the injured leg forward at the same time and then to move the unaffected leg. Patients are discouraged from using furniture to assist with ambulation. The patient is taught to place weight on the hands, not in the axilla, to avoid nerve damage. If the 2- or 4-point gaits are to be used, the crutch and leg on opposite sides move forward, not the crutch and same-side leg.

19. The day after a having a right below-the-knee amputation, a patient complains of pain in the right foot. Which action is best for the nurse to take? a. Explain the reasons for the phantom limb pain. b. Administer prescribed analgesics to relieve the pain. c. Loosen the compression bandage to decrease incisional pressure. d. Inform the patient that this phantom pain will diminish over time.

ANS: B Phantom limb sensation is treated like any other type of postoperative pain would be treated. Explanations of the reason for the pain may be given, but the nurse should still medicate the patient. The compression bandage is left in place except during physical therapy or bathing. Although the pain may decrease over time, it still requires treatment now.

19. The day after a having a right below-the-knee amputation, a patient complains of pain in the right foot. Which action is best for the nurse to take? a. Explain the reasons for the phantom limb pain. b. Administer prescribed analgesics to relieve the pain. c. Loosen the compression bandage to decrease incisional pressure. d. Inform the patient that this phantom pain will diminish over time.

ANS: B Phantom limb sensation is treated like any other type of postoperative pain would be treated. Explanations of the reason for the pain may be given, but the nurse should still medicate the patient. The compression bandage is left in place except during physical therapy or bathing. Although the pain may decrease over time, it still requires treatment now.

5. A 22-year-old tennis player has an arthroscopic repair of a rotator cuff injury performed in same-day surgery. When the nurse plans postoperative teaching for the patient, which information will be included? a. "You will not be able to serve a tennis ball again." b. "You will work with a physical therapist tomorrow." c. "The doctor will use the drop-arm test to determine the success of surgery." d. "Leave the shoulder immobilizer on for the first 4 days to minimize pain."

ANS: B Physical therapy after a rotator cuff repair begins on the first postoperative day to prevent "frozen shoulder." A shoulder immobilizer is used immediately after the surgery, but leaving the arm immobilized for several days would lead to loss of range of motion (ROM). The drop-arm test is used to test for rotator cuff injury, but not after surgery. The patient may be able to return to pitching after rehabilitation

5. A 22-year-old tennis player has an arthroscopic repair of a rotator cuff injury performed in same-day surgery. When the nurse plans postoperative teaching for the patient, which information will be included? a. "You will not be able to serve a tennis ball again." b. "You will work with a physical therapist tomorrow." c. "The doctor will use the drop-arm test to determine the success of surgery." d. "Leave the shoulder immobilizer on for the first 4 days to minimize pain."

ANS: B Physical therapy after a rotator cuff repair begins on the first postoperative day to prevent "frozen shoulder." A shoulder immobilizer is used immediately after the surgery, but leaving the arm immobilized for several days would lead to loss of range of motion (ROM). The drop-arm test is used to test for rotator cuff injury, but not after surgery. The patient may be able to return to pitching after rehabilitation.

39. When assessing for Tinel's sign in a patient with possible right-sided carpal tunnel syndrome, the nurse will ask the patient about a. weakness in the right little finger. b. tingling in the right thumb and fingers. c. burning in the right elbow and forearm. d. tremor when gripping with the right hand.

ANS: B Testing for Tinel's sign will cause tingling in the thumb and first three fingers of the affected hand in patients who have carpal tunnel syndrome. The median nerve does not innervate the right little finger or elbow and forearm. Tremor is not associated with carpal tunnel syndrome.

39. When assessing for Tinel's sign in a patient with possible right-sided carpal tunnel syndrome, the nurse will ask the patient about a. weakness in the right little finger. b. tingling in the right thumb and fingers. c. burning in the right elbow and forearm. d. tremor when gripping with the right hand.

ANS: B Testing for Tinel's sign will cause tingling in the thumb and first three fingers of the affected hand in patients who have carpal tunnel syndrome. The median nerve does not innervate the right little finger or elbow and forearm. Tremor is not associated with carpal tunnel syndrome.

13. A patient with a complex pelvic fracture from a motor vehicle crash is on bed rest. Which nursing assessment finding is important to report to the health care provider? a. The patient states that the pelvis feels unstable. b. Abdomen is distended and bowel sounds are absent. c. There are ecchymoses across the abdomen and hips. d. The patient complains of pelvic pain with palpation.

ANS: B The abdominal distention and absent bowel sounds may be due to complications of pelvic fractures such as paralytic ileus or hemorrhage or trauma to the bladder, urethra, or colon. Pelvic instability, abdominal pain with palpation, and abdominal bruising would be expected with this type of injury.

13. A patient with a complex pelvic fracture from a motor vehicle crash is on bed rest. Which nursing assessment finding is important to report to the health care provider? a. The patient states that the pelvis feels unstable. b. Abdomen is distended and bowel sounds are absent. c. There are ecchymoses across the abdomen and hips. d. The patient complains of pelvic pain with palpation.

ANS: B The abdominal distention and absent bowel sounds may be due to complications of pelvic fractures such as paralytic ileus or hemorrhage or trauma to the bladder, urethra, or colon. Pelvic instability, abdominal pain with palpation, and abdominal bruising would be expected with this type of injury.

17. When doing discharge teaching for a 19-year-old patient who has had a repair of a fractured mandible, the nurse will include information about a. administration of nasogastric tube feedings. b. how and when to cut the immobilizing wires. c. the importance of high-fiber foods in the diet. d. the use of sterile technique for dressing changes.

ANS: B The jaw will be wired for stabilization, and the patient should know what emergency situations require that the wires be cut to protect the airway. There are no dressing changes for this procedure. The diet is liquid, and patients are not able to chew high-fiber foods. Initially, the patient may receive nasogastric tube feedings, but by discharge, the patient will swallow liquid through a straw.

16. A patient who has had an open reduction and internal fixation (ORIF) of a hip fracture tells the nurse that he is ready to get out of bed for the first time. Which action should the nurse take? a. Use a mechanical lift to transfer the patient from the bed to the chair. b. Check the postoperative orders for the patient's weight-bearing status. c. Avoid administration of pain medications before getting the patient up. d. Delegate the transfer of the patient to nursing assistive personnel (NAP).

ANS: B The nurse should be familiar with the weight-bearing orders for the patient before attempting the transfer. Mechanical lifts are not typically needed after this surgery. Pain medications should be given because the movement is likely to be painful for the patient. The registered nurse (RN) should supervise the patient during the initial transfer to evaluate how well the patient is able to accomplish this skill.

16. A patient who has had an open reduction and internal fixation (ORIF) of a hip fracture tells the nurse that he is ready to get out of bed for the first time. Which action should the nurse take? a. Use a mechanical lift to transfer the patient from the bed to the chair. b. Check the postoperative orders for the patient's weight-bearing status. c. Avoid administration of pain medications before getting the patient up. d. Delegate the transfer of the patient to nursing assistive personnel (NAP).

ANS: B The nurse should be familiar with the weight-bearing orders for the patient before attempting the transfer. Mechanical lifts are not typically needed after this surgery. Pain medications should be given because the movement is likely to be painful for the patient. The registered nurse (RN) should supervise the patient during the initial transfer to evaluate how well the patient is able to accomplish this skill.

37. A patient undergoes a left above-the-knee amputation with an immediate prosthetic fitting. When the patient arrives on the orthopedic unit after surgery, the nurse should a. place the patient in a prone position. b. check the surgical site for hemorrhage. c. remove the prosthesis and wrap the site. d. keep the residual leg elevated on a pillow.

ANS: B The nurse should monitor for hemorrhage after the surgery. The prosthesis will not be removed. To avoid flexion contracture of the hip, the leg will not be elevated on a pillow. The patient is placed in a prone position after amputation to prevent hip flexion, but this would not be done during the immediate postoperative period.

37. A patient undergoes a left above-the-knee amputation with an immediate prosthetic fitting. When the patient arrives on the orthopedic unit after surgery, the nurse should a. place the patient in a prone position. b. check the surgical site for hemorrhage. c. remove the prosthesis and wrap the site. d. keep the residual leg elevated on a pillow.

ANS: B The nurse should monitor for hemorrhage after the surgery. The prosthesis will not be removed. To avoid flexion contracture of the hip, the leg will not be elevated on a pillow. The patient is placed in a prone position after amputation to prevent hip flexion, but this would not be done during the immediate postoperative period.

. Which statement by a 62-year-old patient who has had an above-the-knee amputation indicates that the nurse's discharge teaching has been effective? a. "I should elevate my residual limb on a pillow 2 or 3 times a day." b. "I should lay flat on my abdomen for 30 minutes 3 or 4 times a day." c. "I should change the limb sock when it becomes soiled or each week." d. "I should use lotion on the stump to prevent skin drying and cracking."

ANS: B The patient lies in the prone position several times daily to prevent flexion contractures of the hip. The limb sock should be changed daily. Lotion should not be used on the stump. The residual limb should not be elevated because this would encourage flexion contracture.

20. Which statement by a 62-year-old patient who has had an above-the-knee amputation indicates that the nurse's discharge teaching has been effective? a. "I should elevate my residual limb on a pillow 2 or 3 times a day." b. "I should lay flat on my abdomen for 30 minutes 3 or 4 times a day." c. "I should change the limb sock when it becomes soiled or each week." d. "I should use lotion on the stump to prevent skin drying and cracking."

ANS: B The patient lies in the prone position several times daily to prevent flexion contractures of the hip. The limb sock should be changed daily. Lotion should not be used on the stump. The residual limb should not be elevated because this would encourage flexion contracture.

38. Before assisting a patient with ambulation 2 days after a total hip replacement, which action is most important for the nurse to take? a. Observe the status of the incisional drain device. b. Administer the ordered oral opioid pain medication. c. Instruct the patient about the benefits of ambulation. d. Change the hip dressing and document the wound appearance.

ANS: B The patient should be adequately medicated for pain before any attempt to ambulate. Instructions about the benefits of ambulation may increase the patient's willingness to ambulate, but decreasing pain with ambulation is more important. The presence of an incisional drain or timing of dressing change will not affect ambulation.

38. Before assisting a patient with ambulation 2 days after a total hip replacement, which action is most important for the nurse to take? a. Observe the status of the incisional drain device. b. Administer the ordered oral opioid pain medication. c. Instruct the patient about the benefits of ambulation. d. Change the hip dressing and document the wound appearance.

ANS: B The patient should be adequately medicated for pain before any attempt to ambulate. Instructions about the benefits of ambulation may increase the patient's willingness to ambulate, but decreasing pain with ambulation is more important. The presence of an incisional drain or timing of dressing change will not affect ambulation.

27. After being hospitalized for 3 days with a right femur fracture, a 32-year-old patient suddenly develops shortness of breath and tachypnea. The patient tells the nurse, "I feel like I am going to die!" Which action should the nurse take first? a. Stay with the patient and offer reassurance. b. Administer the prescribed PRN oxygen at 4 L/min. c. Check the patient's legs for swelling or tenderness. d. Notify the health care provider about the symptoms.

ANS: B The patient's clinical manifestations and history are consistent with a pulmonary embolus, and the nurse's first action should be to ensure adequate oxygenation. The nurse should offer reassurance to the patient, but meeting the physiologic need for oxygen is a higher priority. The health care provider should be notified after the oxygen is started and pulse oximetry and assessment for fat embolus or venous thromboembolism (VTE) are obtained.

27. After being hospitalized for 3 days with a right femur fracture, a 32-year-old patient suddenly develops shortness of breath and tachypnea. The patient tells the nurse, "I feel like I am going to die!" Which action should the nurse take first? a. Stay with the patient and offer reassurance. b. Administer the prescribed PRN oxygen at 4 L/min. c. Check the patient's legs for swelling or tenderness. d. Notify the health care provider about the symptoms.

ANS: B The patient's clinical manifestations and history are consistent with a pulmonary embolus, and the nurse's first action should be to ensure adequate oxygenation. The nurse should offer reassurance to the patient, but meeting the physiologic need for oxygen is a higher priority. The health care provider should be notified after the oxygen is started and pulse oximetry and assessment for fat embolus or venous thromboembolism (VTE) are obtained.

45. When caring for a patient who is using Buck's traction after a hip fracture, which action can the nurse delegate to unlicensed assistive personnel (UAP)? a. Monitor the skin under the traction boot for redness. b. Ensure that the weight for the traction is off the floor. c. Check for intact sensation and movement in the affected leg. d. Offer reassurance that hip and leg pain are normal after hip fracture.

ANS: B UAP can be responsible for maintaining the integrity of the traction once it has been established. Assessment of skin integrity and circulation should be done by the registered nurse (RN). UAP should notify the RN if the patient experiences hip and leg pain because pain and effectiveness of pain relief measures should be assessed by the RN.

45. When caring for a patient who is using Buck's traction after a hip fracture, which action can the nurse delegate to unlicensed assistive personnel (UAP)? a. Monitor the skin under the traction boot for redness. b. Ensure that the weight for the traction is off the floor. c. Check for intact sensation and movement in the affected leg. d. Offer reassurance that hip and leg pain are normal after hip fracture.

ANS: B UAP can be responsible for maintaining the integrity of the traction once it has been established. Assessment of skin integrity and circulation should be done by the registered nurse (RN). UAP should notify the RN if the patient experiences hip and leg pain because pain and effectiveness of pain relief measures should be assessed by the RN.

9. A patient has a long-arm plaster cast applied for immobilization of a fractured left radius. Until the cast has completely dried, the nurse should a. keep the left arm in dependent position. b. avoid handling the cast using fingertips. c. place gauze around the cast edge to pad any roughness. d. cover the cast with a small blanket to absorb the dampness.

ANS: B Until a plaster cast has dried, using the palms rather than the fingertips to handle the cast helps prevent creating protrusions inside the cast that could place pressure on the skin. The left arm should be elevated to prevent swelling. The edges of the cast may be petaled once the cast is dry, but padding the edges before that may cause the cast to be misshapen. The cast should not be covered until it is dry because heat builds up during drying.

9. A patient has a long-arm plaster cast applied for immobilization of a fractured left radius. Until the cast has completely dried, the nurse should a. keep the left arm in dependent position. b. avoid handling the cast using fingertips. c. place gauze around the cast edge to pad any roughness. d. cover the cast with a small blanket to absorb the dampness.

ANS: B Until a plaster cast has dried, using the palms rather than the fingertips to handle the cast helps prevent creating protrusions inside the cast that could place pressure on the skin. The left arm should be elevated to prevent swelling. The edges of the cast may be petaled once the cast is dry, but padding the edges before that may cause the cast to be misshapen. The cast should not be covered until it is dry because heat builds up during drying.

A patient is seen at the urgent care center after falling on the right arm and shoulder. Which finding is most important for the nurse to communicate to the health care provider? a. There is bruising at the shoulder area. b. The right arm appears shorter than the left. c. There is decreased range of motion of the shoulder. d. The patient is complaining of arm and shoulder pain.

ANS: B A shorter limb after a fall indicates a possible dislocation, which is an orthopedic emergency. Bruising, pain, and decreased range of motion also should be reported, but these do not indicate that emergent treatment is needed to preserve function.

8. Which nursing intervention will be included in the plan of care after a patient with a right femur fracture has a hip spica cast applied? a. Avoid placing the patient in prone position. b. Ask the patient about abdominal discomfort. c. Discuss remaining on bed rest for several weeks. d. Use the cast support bar to reposition the patient.

ANS: B Assessment of bowel sounds, abdominal pain, and nausea and vomiting will detect the development of cast syndrome. To avoid breakage, the support bar should not be used for repositioning. After the cast dries, the patient can begin ambulating with the assistance of physical therapy personnel and may be turned to the prone position. DIF: Cognitive Level: Apply (application) REF: 1516 TOP: Nursing Process: Planning MSC: NCLEX: Physiological Integrity

A patient who has a cast in place after fracturing the radius asks when the cast can be removed. The nurse will instruct the patient that the cast will need to remain in place a. for several months. b. for at least 3 weeks. c. until swelling of the wrist has resolved. d. until x-rays show complete bony union.

ANS: B Bone healing starts immediately after the injury, but since ossification does not begin until 3 weeks postinjury, the cast will need to be worn for at least 3 weeks. Complete union may take up to a year. Resolution of swelling does not indicate bone healing.

Which action will the nurse take in order to evaluate the effectiveness of Buck's traction for a 62-year-old patient who has an intracapsular fracture of the right femur? a. Check peripheral pulses. b. Ask about hip pain level. c. Assess for hip contractures. d. Monitor for hip dislocation.

ANS: B Buck's traction keeps the leg immobilized and reduces painful muscle spasm. Hip contractures and dislocation are unlikely to occur in this situation. The peripheral pulses will be assessed, but this does not help in evaluating the effectiveness of Buck's traction.

A young man arrives in the emergency department with ankle swelling and severe pain after twisting his ankle playing basketball. Which of these prescribed collaborative interventions will the nurse implement first? a. Take the patient to have x-rays. b. Wrap the ankle and apply an ice pack. c. Administer naproxen (Naprosyn) 500 mg PO. d. Give acetaminophen with codeine (Tylenol #3).

ANS: B Immediate care after a sprain or strain injury includes the application of cold and compression to the injury to minimize swelling. The other actions should be taken after the ankle is wrapped with a compression bandage and ice is applied.

29. A young man arrives in the emergency department with ankle swelling and severe pain after twisting his ankle playing basketball. Which of these prescribed collaborative interventions will the nurse implement first? a. Take the patient to have x-rays. b. Wrap the ankle and apply an ice pack. c. Administer naproxen (Naprosyn) 500 mg PO. d. Give acetaminophen with codeine (Tylenol #3).

ANS: B Immediate care after a sprain or strain injury includes the application of cold and compression to the injury to minimize swelling. The other actions should be taken after the ankle is wrapped with a compression bandage and ice is applied. DIF: Cognitive Level: Apply (application) REF: 1507 OBJ: Special Questions: Prioritization TOP: Nursing Process: Implementation MSC: NCLEX: Physiological Integrity

A patient has hip replacement surgery using the posterior approach. Which patient action requires rapid intervention by the nurse? a. The patient uses crutches with a swing-to gait. b. The patient leans over to pull shoes and socks on. c. The patient sits straight up on the edge of the bed. d. The patient bends over the sink while brushing the teeth.

ANS: B Leaning over would flex the hip at greater than 90 degrees and predispose the patient to hip dislocation. The other patient actions are appropriate and do not require any immediate action by the nurse to protect the patient.

A patient is being discharged 4 days after hip replacement surgery using the posterior approach. Which patient action requires immediate intervention by the nurse? a. The patient uses crutches with a swing-to gait. b. The patient leans over to pull shoes and socks on. c. The patient sits straight up on the edge of the bed. d. The patient bends over the sink while brushing teeth.

ANS: B Leaning over would flex the hip at greater than 90 degrees and predispose the patient to hip dislocation. The other patient actions are appropriate and do not require any immediate action by the nurse to protect the patient.

A patient arrived at the emergency department after tripping over a rug and falling at home. Which finding is most important for the nurse to communicate to the health care provider? a. There is bruising at the shoulder area. b. The patient reports arm and shoulder pain. c. The right arm appears shorter than the left. d. There is decreased shoulder range of motion.

ANS: C A shorter limb after a fall indicates a possible dislocation, which is an orthopedic emergency. Bruising, pain, and decreased range of motion also should be reported, but these do not indicate that emergent treatment is needed to preserve function.

On the first postoperative day, a patient with a below-the-knee amputation complains of pain in the amputated limb. Which action is best for the nurse to take? a. Explain the reasons for the phantom limb pain. b. Administer prescribed analgesics to relieve the pain. c. Loosen the compression bandage to decrease incisional pressure. d. Remind the patient that this phantom pain will diminish over time.

ANS: B Phantom limb pain is treated like any other type of postoperative pain would be treated. Explanations of the reason for the pain may be given, but the nurse should still medicate the patient. The compression bandage is left in place except during physical therapy or bathing. Although the pain may decrease over time, it still requires treatment now.

The day after a having a right below-the-knee amputation, a patient complains of pain in the right foot. Which action is best for the nurse to take? a. Explain the reasons for the phantom limb pain. b. Administer prescribed analgesics to relieve the pain. c. Loosen the compression bandage to decrease incisional pressure. d. Inform the patient that this phantom pain will diminish over time.

ANS: B Phantom limb sensation is treated like any other type of postoperative pain would be treated. Explanations of the reason for the pain may be given, but the nurse should still medicate the patient. The compression bandage is left in place except during physical therapy or bathing. Although the pain may decrease over time, it still requires treatment now.

19. The day after a having a right below-the-knee amputation, a patient complains of pain in the right foot. Which action is best for the nurse to take? a. Explain the reasons for the phantom limb pain. b. Administer prescribed analgesics to relieve the pain. c. Loosen the compression bandage to decrease incisional pressure. d. Inform the patient that this phantom pain will diminish over time.

ANS: B Phantom limb sensation is treated like any other type of postoperative pain would be treated. Explanations of the reason for the pain may be given, but the nurse should still medicate the patient. The compression bandage is left in place except during physical therapy or bathing. Although the pain may decrease over time, it still requires treatment now. DIF: Cognitive Level: Understand (comprehension) REF: 1532 TOP: Nursing Process: Implementation MSC: NCLEX: Physiological Integrity

When planning care for a patient who has had hip replacement surgery, which nursing action can the nurse delegate to experienced nursing assistive personnel (NAP)? a. Teach quadriceps-setting exercises. b. Reposition the patient every 1 to 2 hours. c. Assess for skin irritation on the patient's back. d. Determine the patient's pain level and tolerance.

ANS: B Repositioning of patients is within the scope of practice of NAP (after they have been trained and evaluated in this skill). The other actions should be done by licensed nursing staff members.

When assessing for Tinel's sign in a patient with possible right-sided carpal tunnel syndrome, the nurse will ask the patient about a. weakness in the right little finger. b. tingling in the right thumb and fingers. c. burning in the right elbow and forearm. d. tremor when gripping with the right hand.

ANS: B Testing for Tinel's sign will cause tingling in the thumb and first three fingers of the affected hand in patients who have carpal tunnel syndrome. The median nerve does not innervate the right little finger or elbow and forearm. Tremor is not associated with carpal tunnel syndrome.

A patient with a complex pelvic fracture from a motor vehicle crash is on bed rest. Which nursing assessment finding is important to report to the health care provider? a. The patient states that the pelvis feels unstable. b. Abdomen is distended and bowel sounds are absent. c. There are ecchymoses across the abdomen and hips. d. The patient complains of pelvic pain with palpation.

ANS: B The abdominal distention and absent bowel sounds may be due to complications of pelvic fractures such as paralytic ileus or hemorrhage or trauma to the bladder, urethra, or colon. Pelvic instability, abdominal pain with palpation, and abdominal bruising would be expected with this type of injury.

When the nurse is caring for a patient who is on bed rest after having a complex pelvic fracture, which assessment finding is most important to report to the health care provider? a. The patient states that the pelvis feels unstable. b. Abdominal distention is present and bowel tones are absent. c. There are ecchymoses on the abdomen and hips. d. The patient complains of pelvic pain with palpation.

ANS: B The abdominal distention and absent bowel tones may be due to complications of pelvic fractures such as paralytic ileus or hemorrhage or trauma to the bladder, urethra, or colon. Pelvic instability, abdominal pain with palpation, and abdominal bruising would be expected with this type of injury.

A patient who has had an open reduction and internal fixation (ORIF) of a hip fracture tells the nurse that he is ready to get out of bed for the first time. Which action should the nurse take? a. Use a mechanical lift to transfer the patient from the bed to the chair. b. Check the postoperative orders for the patient's weight-bearing status. c. Avoid administration of pain medications before getting the patient up. d. Delegate the transfer of the patient to nursing assistive personnel (NAP).

ANS: B The nurse should be familiar with the weight-bearing orders for the patient before attempting the transfer. Mechanical lifts are not typically needed after this surgery. Pain medications should be given because the movement is likely to be painful for the patient. The registered nurse (RN) should supervise the patient during the initial transfer to evaluate how well the patient is able to accomplish this skill.

16. A patient who has had an open reduction and internal fixation (ORIF) of a hip fracture tells the nurse that he is ready to get out of bed for the first time. Which action should the nurse take? a. Use a mechanical lift to transfer the patient from the bed to the chair. b. Check the postoperative orders for the patient's weight-bearing status. c. Avoid administration of pain medications before getting the patient up. d. Delegate the transfer of the patient to nursing assistive personnel (NAP).

ANS: B The nurse should be familiar with the weight-bearing orders for the patient before attempting the transfer. Mechanical lifts are not typically needed after this surgery. Pain medications should be given because the movement is likely to be painful for the patient. The registered nurse (RN) should supervise the patient during the initial transfer to evaluate how well the patient is able to accomplish this skill. DIF: Cognitive Level: Apply (application) REF: 1528 TOP: Nursing Process: Implementation MSC: NCLEX: Physiological Integrity

The nurse is preparing to assist a patient who has had an open reduction and internal fixation (ORIF) of a hip fracture out of bed for the first time. Which action should the nurse take? a. Use a mechanical lift to transfer the patient from the bed to the chair. b. Check the postoperative orders for the patient's weight-bearing status. c. Avoid administration of pain medications before getting the patient up. d. Delegate the transfer of the patient out of bed to nursing assistive personnel (NAP).

ANS: B The nurse should be familiar with the weight-bearing orders for the patient before attempting the transfer. Mechanical lifts are not typically needed after this surgery. Pain medications should be given, since the movement is likely to be painful for the patient. The RN should supervise the patient during the initial transfer to evaluate how well the patient is able to accomplish this skill.

A patient undergoes a left above-the-knee amputation with an immediate prosthetic fitting. When the patient arrives on the orthopedic unit after surgery, the nurse should a. place the patient in a prone position. b. check the surgical site for hemorrhage. c. remove the prosthesis and wrap the site. d. keep the residual leg elevated on a pillow.

ANS: B The nurse should monitor for hemorrhage after the surgery. The prosthesis will not be removed. To avoid flexion contracture of the hip, the leg will not be elevated on a pillow. The patient is placed in a prone position after amputation to prevent hip flexion, but this would not be done during the immediate postoperative period.

A patient undergoes a right above-the-knee amputation with an immediate prosthetic fitting. When the patient first arrives on the orthopedic unit after surgery, the nurse should a. place the patient in a prone position. b. check the surgical site for hemorrhage. c. remove the prosthesis and wrap the site. d. keep the residual leg elevated on a pillow.

ANS: B The nurse should monitor for hemorrhage after the surgery. The prosthesis will not be removed. To avoid flexion contracture of the hip, the leg will not be elevated on a pillow. The patient is placed in a prone position after amputation to prevent hip flexion, but this would not be done during the immediate postoperative period.

37. A patient undergoes a left above-the-knee amputation with an immediate prosthetic fitting. When the patient arrives on the orthopedic unit after surgery, the nurse should a. place the patient in a prone position. b. check the surgical site for hemorrhage. c. remove the prosthesis and wrap the site. d. keep the residual leg elevated on a pillow.

ANS: B The nurse should monitor for hemorrhage after the surgery. The prosthesis will not be removed. To avoid flexion contracture of the hip, the leg will not be elevated on a pillow. The patient is placed in a prone position after amputation to prevent hip flexion, but this would not be done during the immediate postoperative period. DIF: Cognitive Level: Apply (application) REF: 1532 TOP: Nursing Process: Implementation MSC: NCLEX: Physiological Integrity

20. Which statement by a 62-year-old patient who has had an above-the-knee amputation indicates that the nurse's discharge teaching has been effective? a. "I should elevate my residual limb on a pillow 2 or 3 times a day." b. "I should lay flat on my abdomen for 30 minutes 3 or 4 times a day." c. "I should change the limb sock when it becomes soiled or each week." d. "I should use lotion on the stump to prevent skin drying and cracking."

ANS: B The patient lies in the prone position several times daily to prevent flexion contractures of the hip. The limb sock should be changed daily. Lotion should not be used on the stump. The residual limb should not be elevated because this would encourage flexion contracture. DIF: Cognitive Level: Apply (application) REF: 1532 TOP: Nursing Process: Evaluation MSC: NCLEX: Physiological Integrity

Before assisting a patient with ambulation 2 days after a total hip replacement, which action is most important for the nurse to take? a. Observe the status of the incisional drain device. b. Administer the ordered oral opioid pain medication. c. Instruct the patient about the benefits of ambulation. d. Change the hip dressing and document the wound appearance.

ANS: B The patient should be adequately medicated for pain before any attempt to ambulate. Instructions about the benefits of ambulation may increase the patient's willingness to ambulate, but decreasing pain with ambulation is more important. The presence of an incisional drain or timing of dressing change will not affect ambulation.

38. Before assisting a patient with ambulation 2 days after a total hip replacement, which action is most important for the nurse to take? a. Observe the status of the incisional drain device. b. Administer the ordered oral opioid pain medication. c. Instruct the patient about the benefits of ambulation. d. Change the hip dressing and document the wound appearance.

ANS: B The patient should be adequately medicated for pain before any attempt to ambulate. Instructions about the benefits of ambulation may increase the patient's willingness to ambulate, but decreasing pain with ambulation is more important. The presence of an incisional drain or timing of dressing change will not affect ambulation. DIF: Cognitive Level: Apply (application) REF: 1526 OBJ: Special Questions: Prioritization TOP: Nursing Process: Implementation MSC: NCLEX: Physiological Integrity

A patient who has been hospitalized for 3 days with a hip fracture has sudden onset shortness of breath and tachypnea. The patient tells the nurse, "I feel like I am going to die!" Which action should the nurse take first? a. Stay with the patient and offer reassurance. b. Administer the prescribed PRN oxygen at 4 L/min. c. Check the patient's legs for swelling or tenderness. d. Notify the health care provider about the symptoms.

ANS: B The patient's clinical manifestations and history are consistent with a pulmonary embolus, and the nurse's first action should be to ensure adequate oxygenation. The nurse should offer reassurance to the patient, but meeting the physiologic need for oxygen is a higher priority. The health care provider should be notified after the oxygen is started and pulse oximetry and assessment for fat embolus or venous thromboembolism (VTE) are obtained.

After being hospitalized for 3 days with a right femur fracture, a 32-year-old patient suddenly develops shortness of breath and tachypnea. The patient tells the nurse, "I feel like I am going to die!" Which action should the nurse take first? a. Stay with the patient and offer reassurance. b. Administer the prescribed PRN oxygen at 4 L/min. c. Check the patient's legs for swelling or tenderness. d. Notify the health care provider about the symptoms.

ANS: B The patient's clinical manifestations and history are consistent with a pulmonary embolus, and the nurse's first action should be to ensure adequate oxygenation. The nurse should offer reassurance to the patient, but meeting the physiologic need for oxygen is a higher priority. The health care provider should be notified after the oxygen is started and pulse oximetry and assessment for fat embolus or venous thromboembolism (VTE) are obtained.

27. After being hospitalized for 3 days with a right femur fracture, a 32-year-old patient suddenly develops shortness of breath and tachypnea. The patient tells the nurse, "I feel like I am going to die!" Which action should the nurse take first? a. Stay with the patient and offer reassurance. b. Administer the prescribed PRN oxygen at 4 L/min. c. Check the patient's legs for swelling or tenderness. d. Notify the health care provider about the symptoms.

ANS: B The patient's clinical manifestations and history are consistent with a pulmonary embolus, and the nurse's first action should be to ensure adequate oxygenation. The nurse should offer reassurance to the patient, but meeting the physiologic need for oxygen is a higher priority. The health care provider should be notified after the oxygen is started and pulse oximetry and assessment for fat embolus or venous thromboembolism (VTE) are obtained. DIF: Cognitive Level: Apply (application) REF: 1528 OBJ: Special Questions: Prioritization TOP: Nursing Process: Implementation MSC: NCLEX: Physiological Integrity

36. A 42-year-old patient is admitted to the emergency department with a left femur fracture. Which information obtained by the nurse is most important to report to the health care provider? a. Ecchymosis of the left thigh b. Complaints of severe thigh pain c. Slow capillary refill of the left foot d. Outward pointing toes on the left foot

ANS: C Prolonged capillary refill may indicate complications such as arterial damage or compartment syndrome. The other findings are typical with a left femur fracture.

When caring for a patient who is using Buck's traction after a hip fracture, which action can the nurse delegate to unlicensed assistive personnel (UAP)? a. Monitor the skin under the traction boot for redness. b. Ensure that the weight for the traction is off the floor. c. Check for intact sensation and movement in the affected leg. d. Offer reassurance that hip and leg pain are normal after hip fracture.

ANS: B UAP can be responsible for maintaining the integrity of the traction once it has been established. Assessment of skin integrity and circulation should be done by the registered nurse (RN). UAP should notify the RN if the patient experiences hip and leg pain because pain and effectiveness of pain relief measures should be assessed by the RN.

A patient has a long-arm plaster cast applied for immobilization of a fractured left radius. Until the cast has completely dried, the nurse should a. keep the left arm in a dependent position. b. handle the cast with the palms of the hands. c. place gauze around the cast edge to pad any roughness. d. cover the cast with a small blanket to absorb the dampness.

ANS: B Until a plaster cast has dried, placing pressure on the cast should be avoided to prevent creating areas inside the cast that could place pressure on the arm. The left arm should be elevated to prevent swelling. The edges of the cast may be petaled once the cast is dry, but padding the edges before that may cause the cast to be misshapen. The cast should not be covered until it is dry because heat builds up during drying.

9. A patient has a long-arm plaster cast applied for immobilization of a fractured left radius. Until the cast has completely dried, the nurse should a. keep the left arm in dependent position. b. avoid handling the cast using fingertips. c. place gauze around the cast edge to pad any roughness. d. cover the cast with a small blanket to absorb the dampness.

ANS: B Until a plaster cast has dried, using the palms rather than the fingertips to handle the cast helps prevent creating protrusions inside the cast that could place pressure on the skin. The left arm should be elevated to prevent swelling. The edges of the cast may be petaled once the cast is dry, but padding the edges before that may cause the cast to be misshapen. The cast should not be covered until it is dry because heat builds up during drying. DIF: Cognitive Level: Apply (application) REF: 1515 TOP: Nursing Process: Implementation MSC: NCLEX: Physiological Integrity

Which of the following observations made by the nurse who is evaluating the crutch-walking technique of a patient who is to have no weight bearing on the right leg indicates that the patient can safely ambulate independently? a. The patient keeps the padded area of the crutch firmly in the axillary area when ambulating. b. The patient advances the right leg and both crutches together and then advances the left leg. c. The patient moves the left crutch with the left leg and then the right crutch with the right leg. d. The patient uses the bedside chair to assist in balance as needed when ambulating in the room.

ANS: B When using crutches, patients are usually taught to move the assistive device and the injured leg forward at the same time and then to move the unaffected leg. Patients are discouraged from using furniture to assist with ambulation. The patient is taught to place weight on the hands, not in the axilla, to avoid nerve damage. If the 2- or 4-point gaits are to be used, the crutch and leg on opposite sides move forward, not the crutch and same-side leg.

17. When doing discharge teaching for a 19-year-old patient who has had a repair of a fractured mandible, the nurse will include information about a. administration of nasogastric tube feedings. b. how and when to cut the immobilizing wires. c. the importance of high-fiber foods in the diet. d. the use of sterile technique for dressing changes.

ANS: B The jaw will be wired for stabilization, and the patient should know what emergency situations require that the wires be cut to protect the airway. There are no dressing changes for this procedure. The diet is liquid, and patients are not able to chew high-fiber foods. Initially, the patient may receive nasogastric tube feedings, but by discharge, the patient will swallow liquid through a straw.

40. Which action will the urgent care nurse take when caring for a patient who has a possible knee meniscus injury? a. Encourage bed rest for 24 to 48 hours. b. Avoid palpation or movement of the knee. c. Apply a knee immobilizer to the affected leg. d. Administer intravenous narcotics for pain relief.

ANS: C A knee immobilizer may be used for several days after a meniscus injury to stabilize the knee and minimize pain. Patients are encouraged to ambulate with crutches. The knee is assessed by flexing, internally rotating, and extending the knee (McMurray's test). The pain associated with a meniscus injury will not typically require IV opioid administration; nonsteroidal antiinflammatory drugs (NSAIDs) are usually recommended for pain relief.

40. Which action will the urgent care nurse take when caring for a patient who has a possible knee meniscus injury? a. Encourage bed rest for 24 to 48 hours. b. Avoid palpation or movement of the knee. c. Apply a knee immobilizer to the affected leg. d. Administer intravenous narcotics for pain relief.

ANS: C A knee immobilizer may be used for several days after a meniscus injury to stabilize the knee and minimize pain. Patients are encouraged to ambulate with crutches. The knee is assessed by flexing, internally rotating, and extending the knee (McMurray's test). The pain associated with a meniscus injury will not typically require IV opioid administration; nonsteroidal antiinflammatory drugs (NSAIDs) are usually recommended for pain relief.

28. A patient arrived at the emergency department after tripping over a rug and falling at home. Which finding is most important for the nurse to communicate to the health care provider? a. There is bruising at the shoulder area. b. The patient reports arm and shoulder pain. c. The right arm appears shorter than the left. d. There is decreased shoulder range of motion.

ANS: C A shorter limb after a fall indicates a possible dislocation, which is an orthopedic emergency. Bruising, pain, and decreased range of motion also should be reported, but these do not indicate that emergent treatment is needed to preserve function.

28. A patient arrived at the emergency department after tripping over a rug and falling at home. Which finding is most important for the nurse to communicate to the health care provider? a. There is bruising at the shoulder area. b. The patient reports arm and shoulder pain. c. The right arm appears shorter than the left. d. There is decreased shoulder range of motion.

ANS: C A shorter limb after a fall indicates a possible dislocation, which is an orthopedic emergency. Bruising, pain, and decreased range of motion also should be reported, but these do not indicate that emergent treatment is needed to preserve function.

1. When teaching seniors at a community recreation center, which information will the nurse include about ways to prevent fractures? a. Tack down scatter rugs in the home. b. Most falls happen outside the home. c. Buy shoes that provide good support and are comfortable to wear. d. Range-of-motion exercises should be taught by a physical therapist.

ANS: C Comfortable shoes with good support will help decrease the risk for falls. Scatter rugs should be eliminated, not just tacked down. Activities of daily living provide range of motion exercise; these do not need to be taught by a physical therapist. Falls inside the home are responsible for many injuries.

1. When teaching seniors at a community recreation center, which information will the nurse include about ways to prevent fractures? a. Tack down scatter rugs in the home. b. Most falls happen outside the home. c. Buy shoes that provide good support and are comfortable to wear. d. Range-of-motion exercises should be taught by a physical therapist.

ANS: C Comfortable shoes with good support will help decrease the risk for falls. Scatter rugs should be eliminated, not just tacked down. Activities of daily living provide range of motion exercise; these do not need to be taught by a physical therapist. Falls inside the home are responsible for many injuries.

4. Which discharge instruction will the emergency department nurse include for a patient with a sprained ankle? a. Keep the ankle loosely wrapped with gauze. b. Apply a heating pad to reduce muscle spasms. c. Use pillows to elevate the ankle above the heart. d. Gently move the ankle through the range of motion.

ANS: C Elevation of the leg will reduce the amount of swelling and pain. Compression bandages are used to decrease swelling. For the first 24 to 48 hours, cold packs are used to reduce swelling. The ankle should be rested and kept immobile to prevent further swelling or injury.

4. Which discharge instruction will the emergency department nurse include for a patient with a sprained ankle? a. Keep the ankle loosely wrapped with gauze. b. Apply a heating pad to reduce muscle spasms. c. Use pillows to elevate the ankle above the heart. d. Gently move the ankle through the range of motion.

ANS: C Elevation of the leg will reduce the amount of swelling and pain. Compression bandages are used to decrease swelling. For the first 24 to 48 hours, cold packs are used to reduce swelling. The ankle should be rested and kept immobile to prevent further swelling or injury.

10. Which statement by the patient indicates a good understanding of the nurse's teaching about a new short-arm plaster cast? a. "I can get the cast wet as long as I dry it right away with a hair dryer." b. "I should avoid moving my fingers and elbow until the cast is removed." c. "I will apply an ice pack to the cast over the fracture site off and on for 24 hours." d. "I can use a cotton-tipped applicator to rub lotion on any dry areas under the cast."

ANS: C Ice application for the first 24 hours after a fracture will help reduce swelling and can be placed over the cast. Plaster casts should not get wet. The patient should be encouraged to move the joints above and below the cast. Patients should not insert objects inside the cast.

10. Which statement by the patient indicates a good understanding of the nurse's teaching about a new short-arm plaster cast? a. "I can get the cast wet as long as I dry it right away with a hair dryer." b. "I should avoid moving my fingers and elbow until the cast is removed." c. "I will apply an ice pack to the cast over the fracture site off and on for 24 hours." d. "I can use a cotton-tipped applicator to rub lotion on any dry areas under the cast.

ANS: C Ice application for the first 24 hours after a fracture will help reduce swelling and can be placed over the cast. Plaster casts should not get wet. The patient should be encouraged to move the joints above and below the cast. Patients should not insert objects inside the cast.

24. When giving home care instructions to a patient who has comminuted forearm fractures and a long-arm cast on the left arm, which information should the nurse include? a. Keep the left shoulder elevated on a pillow or cushion. b. Keep the hand immobile to prevent soft tissue swelling. c. Call the health care provider for increased swelling or numbness of the hand. d. Avoid nonsteroidal antiinflammatory drugs (NSAIDs) for 24 hours after the injury.

ANS: C Increased swelling or numbness may indicate increased pressure at the injury, and the health care provider should be notified immediately to avoid damage to nerves and other tissues. The patient should be encouraged to move the joints above and below the cast to avoid stiffness. There is no need to elevate the shoulder, although the forearm should be elevated to reduce swelling. NSAIDs are appropriate to treat pain after a fracture.

24. When giving home care instructions to a patient who has comminuted forearm fractures and a long-arm cast on the left arm, which information should the nurse include? a. Keep the left shoulder elevated on a pillow or cushion. b. Keep the hand immobile to prevent soft tissue swelling. c. Call the health care provider for increased swelling or numbness of the hand. d. Avoid nonsteroidal antiinflammatory drugs (NSAIDs) for 24 hours after the injury.

ANS: C Increased swelling or numbness may indicate increased pressure at the injury, and the health care provider should be notified immediately to avoid damage to nerves and other tissues. The patient should be encouraged to move the joints above and below the cast to avoid stiffness. There is no need to elevate the shoulder, although the forearm should be elevated to reduce swelling. NSAIDs are appropriate to treat pain after a fracture.

43. When a patient arrives in the emergency department with a facial fracture, which action will the nurse take first? a. Assess for nasal bleeding and pain. b. Apply ice to the face to reduce swelling. c. Use a cervical collar to stabilize the spine. d. Check the patient's alertness and orientation.

ANS: C Patients who have facial fractures are at risk for cervical spine injury and should be treated as if they have a cervical spine injury until this is ruled out. The other actions are also necessary, but the most important action is to prevent cervical spine injury.

43. When a patient arrives in the emergency department with a facial fracture, which action will the nurse take first? a. Assess for nasal bleeding and pain. b. Apply ice to the face to reduce swelling. c. Use a cervical collar to stabilize the spine. d. Check the patient's alertness and orientation.

ANS: C Patients who have facial fractures are at risk for cervical spine injury and should be treated as if they have a cervical spine injury until this is ruled out. The other actions are also necessary, but the most important action is to prevent cervical spine injury.

36. A 42-year-old patient is admitted to the emergency department with a left femur fracture. Which information obtained by the nurse is most important to report to the health care provider? a. Ecchymosis of the left thigh b. Complaints of severe thigh pain c. Slow capillary refill of the left foot d. Outward pointing toes on the left foot

ANS: C Prolonged capillary refill may indicate complications such as arterial damage or compartment syndrome. The other findings are typical with a left femur fracture

25. A patient who slipped and fell in the shower at home has a proximal humerus fracture immobilized with a left-sided long-arm cast and a sling. Which nursing intervention will be included in the plan of care? a. Use surgical net dressing to hang the arm from an IV pole. b. Immobilize the fingers of the left hand with gauze dressings. c. Assess the left axilla and change absorbent dressings as needed. d. Assist the patient in passive range of motion (ROM) for the right arm.

ANS: C The axilla can become excoriated when a sling is used to support the arm, and the nurse should check the axilla and apply absorbent dressings to prevent this. A patient with a sling would not have traction applied by hanging. The patient will be encouraged to move the fingers on the injured arm to maintain function and to help decrease swelling. The patient will do active ROM on the uninjured side.

25. A patient who slipped and fell in the shower at home has a proximal humerus fracture immobilized with a left-sided long-arm cast and a sling. Which nursing intervention will be included in the plan of care? a. Use surgical net dressing to hang the arm from an IV pole. b. Immobilize the fingers of the left hand with gauze dressings. c. Assess the left axilla and change absorbent dressings as needed. d. Assist the patient in passive range of motion (ROM) for the right arm.

ANS: C The axilla can become excoriated when a sling is used to support the arm, and the nurse should check the axilla and apply absorbent dressings to prevent this. A patient with a sling would not have traction applied by hanging. The patient will be encouraged to move the fingers on the injured arm to maintain function and to help decrease swelling. The patient will do active ROM on the uninjured side.

1. A patient who arrives at the emergency department experiencing severe left knee pain is diagnosed with a patellar dislocation. The initial patient teaching by the nurse will focus on the need for a. a knee immobilizer. b. gentle knee flexion. c. monitored anesthesia care. d. physical activity restrictions.

ANS: C The first goal of collaborative management is realignment of the knee to its original anatomic position, which will require anesthesia or monitored anesthesia care (MAC), formerly called conscious sedation. Immobilization, gentle range-of-motion (ROM) exercises, and discussion about activity restrictions will be implemented after the knee is realigned.

31. A patient who arrives at the emergency department experiencing severe left knee pain is diagnosed with a patellar dislocation. The initial patient teaching by the nurse will focus on the need for a. a knee immobilizer. b. gentle knee flexion. c. monitored anesthesia care. d. physical activity restrictions.

ANS: C The first goal of collaborative management is realignment of the knee to its original anatomic position, which will require anesthesia or monitored anesthesia care (MAC), formerly called conscious sedation. Immobilization, gentle range-of-motion (ROM) exercises, and discussion about activity restrictions will be implemented after the knee is realigned.

23. A high school teacher with ulnar drift caused by rheumatoid arthritis (RA) is scheduled for a left hand arthroplasty. Which patient statement to the nurse indicates a realistic expectation for the surgery? a. "This procedure will correct the deformities in my fingers." b. "I will not have to do as many hand exercises after the surgery." c. "I will be able to use my fingers with more flexibility to grasp things." d. "My fingers will appear more normal in size and shape after this surgery."

ANS: C The goal of hand surgery in RA is to restore function, not to correct for cosmetic deformity or treat the underlying process. Hand exercises will be prescribed after the surgery.

23. A high school teacher with ulnar drift caused by rheumatoid arthritis (RA) is scheduled for a left hand arthroplasty. Which patient statement to the nurse indicates a realistic expectation for the surgery? a. "This procedure will correct the deformities in my fingers." b. "I will not have to do as many hand exercises after the surgery." c. "I will be able to use my fingers with more flexibility to grasp things." d. "My fingers will appear more normal in size and shape after this surgery."

ANS: C The goal of hand surgery in RA is to restore function, not to correct for cosmetic deformity or treat the underlying process. Hand exercises will be prescribed after the surgery.

32. Following a motorcycle accident, a 58-year-old patient arrives in the emergency department with massive left lower leg swelling. Which action will the nurse take first? a. Elevate the leg on 2 pillows. b. Apply a compression bandage. c. Check leg pulses and sensation. d. Place ice packs on the lower leg.

ANS: C The initial action by the nurse will be to assess the circulation to the leg and to observe for any evidence of injury such as fractures or dislocations. After the initial assessment, the other actions may be appropriate, based on what is observed during the assessment.

32. Following a motorcycle accident, a 58-year-old patient arrives in the emergency department with massive left lower leg swelling. Which action will the nurse take first? a. Elevate the leg on 2 pillows. b. Apply a compression bandage. c. Check leg pulses and sensation. d. Place ice packs on the lower leg.

ANS: C The initial action by the nurse will be to assess the circulation to the leg and to observe for any evidence of injury such as fractures or dislocations. After the initial assessment, the other actions may be appropriate, based on what is observed during the assessment.

33. A pedestrian who was hit by a car is admitted to the emergency department with possible right lower leg fractures. The initial action by the nurse should be to a. elevate the right leg. b. splint the lower leg. c. check the pedal pulses. d. verify tetanus immunizations.

ANS: C The initial nursing action should be assessment of the neurovascular status of the injured leg. After assessment, the nurse may need to splint and elevate the leg, based on the assessment data. Information about tetanus immunizations should be done if there is an open wound.

33. A pedestrian who was hit by a car is admitted to the emergency department with possible right lower leg fractures. The initial action by the nurse should be to a. elevate the right leg. b. splint the lower leg. c. check the pedal pulses. d. verify tetanus immunizations.

ANS: C The initial nursing action should be assessment of the neurovascular status of the injured leg. After assessment, the nurse may need to splint and elevate the leg, based on the assessment data. Information about tetanus immunizations should be done if there is an open wound.

18. After the health care provider has recommended amputation for a patient who has nonhealing ischemic foot ulcers, the patient tells the nurse that he would rather die than have an amputation. Which response by the nurse is best? a. "You are upset, but you may lose the foot anyway." b. "Many people are able to function with a foot prosthesis." c. "Tell me what you know about your options for treatment." d. "If you do not want an amputation, you do not have to have it."

ANS: C The initial nursing action should be to assess the patient's knowledge level and feelings about the options available. Discussion about the patient's option to not have the procedure, the seriousness of the condition, or rehabilitation after the procedure may be appropriate after the nurse knows more about the patient's current level of knowledge and emotional state.

18. After the health care provider has recommended amputation for a patient who has nonhealing ischemic foot ulcers, the patient tells the nurse that he would rather die than have an amputation. Which response by the nurse isbest? a. "You are upset, but you may lose the foot anyway." b. "Many people are able to function with a foot prosthesis." c. "Tell me what you know about your options for treatment." d. "If you do not want an amputation, you do not have to have it."

ANS: C The initial nursing action should be to assess the patient's knowledge level and feelings about the options available. Discussion about the patient's option to not have the procedure, the seriousness of the condition, or rehabilitation after the procedure may be appropriate after the nurse knows more about the patient's current level of knowledge and emotional state.

35. The second day after admission with a fractured pelvis, a 64-year-old patient suddenly develops confusion. Which action should the nurse take first? a. Take the blood pressure. b. Assess patient orientation. c. Check the oxygen saturation. d. Observe for facial asymmetry.

ANS: C The patient's history and clinical manifestations suggest a fat embolus. The most important assessment is oxygenation. The other actions are also appropriate but will be done after the nurse assesses gas exchange.

35. The second day after admission with a fractured pelvis, a 64-year-old patient suddenly develops confusion. Which action should the nurse take first? a. Take the blood pressure. b. Assess patient orientation. c. Check the oxygen saturation. d. Observe for facial asymmetry.

ANS: C The patient's history and clinical manifestations suggest a fat embolus. The most important assessment is oxygenation. The other actions are also appropriate but will be done after the nurse assesses gas exchange.

Which action will the urgent care nurse take when caring for a patient who has a possible knee meniscus injury? a. Encourage bed rest for 24 to 48 hours. b. Avoid palpation or movement of the knee. c. Apply a knee immobilizer to the affected leg. d. Administer intravenous narcotics for pain relief.

ANS: C A knee immobilizer may be used for several days after a meniscus injury to stabilize the knee and minimize pain. Patients are encouraged to ambulate with crutches. The knee is assessed by flexing, internally rotating, and extending the knee (McMurray's test). The pain associated with a meniscus injury will not typically require IV opioid administration; nonsteroidal antiinflammatory drugs (NSAIDs) are usually recommended for pain relief.

28. A patient arrived at the emergency department after tripping over a rug and falling at home. Which finding is most important for the nurse to communicate to the health care provider? a. There is bruising at the shoulder area. b. The patient reports arm and shoulder pain. c. The right arm appears shorter than the left. d. There is decreased shoulder range of motion.

ANS: C A shorter limb after a fall indicates a possible dislocation, which is an orthopedic emergency. Bruising, pain, and decreased range of motion also should be reported, but these do not indicate that emergent treatment is needed to preserve function. DIF: Cognitive Level: Apply (application) REF: 1508 OBJ: Special Questions: Prioritization TOP: Nursing Process: Assessment MSC: NCLEX: Physiological Integrity

Which nursing action will the nurse include in the plan of care for a patient who has had a total knee arthroplasty? a. Avoid extension of the knee beyond 120 degrees. b. Use a compression bandage to keep the knee flexed. c. Start progressive knee exercises to obtain 90-degree flexion. d. Teach about the need to avoid weight bearing for 4 weeks.

ANS: C After knee arthroplasty, active or passive flexion exercises are used to obtain a 90-degree flexion of the knee. The goal for extension of the knee will be 180 degrees. A compression bandage is used to hold the knee in an extended position after surgery. Full weight bearing is expected before discharge

After a patient with a left femur fracture has a hip spica cast applied, which nursing intervention will be included in the plan of care? a. Avoid placing the patient in the prone position. b. Use the cast support bar to reposition the patient. c. Ask the patient about any abdominal discomfort or nausea. d. Discuss the reasons for remaining on bed rest for several weeks.

ANS: C Assessment of bowel tones, abdominal pain, and nausea and vomiting will detect the development of cast syndrome. To avoid breakage, the support bar should not be used for repositioning. After the cast dries, the patient can begin ambulating with the assistance of physical therapy personnel and may be turned to the prone position.

46. Based on the information shown in the accompanying figure and obtained for a patient in the emergency room, which action will the nurse take first? a. Administer the prescribed morphine 4 mg IV. b. Contact the operating room to schedule surgery. c. Check the patient's oxygen saturation using pulse oximetry. d. Ask the patient about the date of the last tetanus immunization.

ANS: C Because fat embolism can occur with tibial fracture, the nurse's first action should be to check the patient's oxygen saturation. The other actions are also appropriate, but not as important at this time as obtaining the patient's oxygen saturation.

46. Based on the information shown in the accompanying figure and obtained for a patient in the emergency room, which action will the nurse take first? a. Administer the prescribed morphine 4 mg IV. b. Contact the operating room to schedule surgery. c. Check the patient's oxygen saturation using pulse oximetry. d. Ask the patient about the date of the last tetanus immunization.

ANS: C Because fat embolism can occur with tibial fracture, the nurse's first action should be to check the patient's oxygen saturation. The other actions are also appropriate, but not as important at this time as obtaining the patient's oxygen saturation. DIF: Cognitive Level: Apply (application) REF: 1523 OBJ: Special Questions: Prioritization TOP: Nursing Process: Implementation MSC: NCLEX: Physiological Integrity

When counseling an older patient about ways to prevent fractures, which information will the nurse include? a. Tack down scatter rugs in the home. b. Most falls happen outside the home. c. Buy shoes that provide good support and are comfortable to wear. d. Range-of-motion exercises should be taught by a physical therapist.

ANS: C Comfortable shoes with good support will help decrease the risk for falls. Scatter rugs should be eliminated, not just tacked down. Activities of daily living provide range of motion exercise; these do not need to be taught by a physical therapist. Falls inside the home are responsible for many injuries.

When teaching seniors at a community recreation center, which information will the nurse include about ways to prevent fractures? a. Tack down scatter rugs in the home. b. Most falls happen outside the home. c. Buy shoes that provide good support and are comfortable to wear. d. Range-of-motion exercises should be taught by a physical therapist.

ANS: C Comfortable shoes with good support will help decrease the risk for falls. Scatter rugs should be eliminated, not just tacked down. Activities of daily living provide range of motion exercise; these do not need to be taught by a physical therapist. Falls inside the home are responsible for many injuries.

1. When teaching seniors at a community recreation center, which information will the nurse include about ways to prevent fractures? a. Tack down scatter rugs in the home. b. Most falls happen outside the home. c. Buy shoes that provide good support and are comfortable to wear. d. Range-of-motion exercises should be taught by a physical therapist.

ANS: C Comfortable shoes with good support will help decrease the risk for falls. Scatter rugs should be eliminated, not just tacked down. Activities of daily living provide range of motion exercise; these do not need to be taught by a physical therapist. Falls inside the home are responsible for many injuries. DIF: Cognitive Level: Apply (application) REF: 1506 TOP: Nursing Process: Implementation MSC: NCLEX: Safe and Effective Care Environment

Which information will the nurse include when discharging a patient with a sprained wrist from the emergency department? a. Keep the wrist loosely wrapped with gauze. b. Apply a heating pad to reduce muscle spasms. c. Use pillows to elevate the arm above the heart. d. Gently move the wrist through the range of motion.

ANS: C Elevation of the arm will reduce the amount of swelling and pain. Compression bandages are used to decrease swelling. For the first 24 to 48 hours, cold packs are used to reduce swelling. The wrist should be rested and kept immobile to prevent further swelling or injury.

Which discharge instruction will the emergency department nurse include for a patient with a sprained ankle? a. Keep the ankle loosely wrapped with gauze. b. Apply a heating pad to reduce muscle spasms. c. Use pillows to elevate the ankle above the heart. d. Gently move the ankle through the range of motion.

ANS: C Elevation of the leg will reduce the amount of swelling and pain. Compression bandages are used to decrease swelling. For the first 24 to 48 hours, cold packs are used to reduce swelling. The ankle should be rested and kept immobile to prevent further swelling or injury.

4. Which discharge instruction will the emergency department nurse include for a patient with a sprained ankle? a. Keep the ankle loosely wrapped with gauze. b. Apply a heating pad to reduce muscle spasms. c. Use pillows to elevate the ankle above the heart. d. Gently move the ankle through the range of motion.

ANS: C Elevation of the leg will reduce the amount of swelling and pain. Compression bandages are used to decrease swelling. For the first 24 to 48 hours, cold packs are used to reduce swelling. The ankle should be rested and kept immobile to prevent further swelling or injury. DIF: Cognitive Level: Apply (application) REF: 1508 TOP: Nursing Process: Implementation MSC: NCLEX: Physiological Integrity

A 42-year-old patient is admitted to the emergency department with a left femur fracture. Which information obtained by the nurse is most important to report to the health care provider? a. Ecchymosis of the left thigh b. Complaints of severe thigh pain c. Slow capillary refill of the left foot d. Outward pointing toes on the left foot

ANS: C Prolonged capillary refill may indicate complications such as arterial damage or compartment syndrome. The other findings are typical with a left femur fracture.

After a patient has a short-arm plaster cast applied in the emergency department, which statement by the patient indicates a good understanding of the nurse's discharge teaching? a. "I can get the cast wet as long as I dry it right away with a hair dryer." b. "I should avoid moving my fingers and elbow until the cast is removed." c. "I will apply an ice pack to the cast over the fracture site for the next 24 hours." d. "I can use a cotton-tipped applicator to rub lotion on any dry areas under the cast."

ANS: C Ice application for the first 24 hours after a fracture will help reduce swelling and can be placed over the cast. Plaster casts should not get wet. The patient should be encouraged to move the joints above and below the cast. Patients should not insert objects inside the cast.

10. Which statement by the patient indicates a good understanding of the nurse's teaching about a new short-arm plaster cast? a. "I can get the cast wet as long as I dry it right away with a hair dryer." b. "I should avoid moving my fingers and elbow until the cast is removed." c. "I will apply an ice pack to the cast over the fracture site off and on for 24 hours." d. "I can use a cotton-tipped applicator to rub lotion on any dry areas under the cast.

ANS: C Ice application for the first 24 hours after a fracture will help reduce swelling and can be placed over the cast. Plaster casts should not get wet. The patient should be encouraged to move the joints above and below the cast. Patients should not insert objects inside the cast. DIF: Cognitive Level: Apply (application) REF: 1520 TOP: Nursing Process: Evaluation MSC: NCLEX: Physiological Integrity

When giving home care instructions to a patient who has comminuted forearm fractures and a long-arm cast on the left arm, which information should the nurse include? a. Keep the left shoulder elevated on a pillow or cushion. b. Keep the hand immobile to prevent soft tissue swelling. c. Call the health care provider for increased swelling or numbness of the hand. d. Avoid nonsteroidal antiinflammatory drugs (NSAIDs) for 24 hours after the injury.

ANS: C Increased swelling or numbness may indicate increased pressure at the injury, and the health care provider should be notified immediately to avoid damage to nerves and other tissues. The patient should be encouraged to move the joints above and below the cast to avoid stiffness. There is no need to elevate the shoulder, although the forearm should be elevated to reduce swelling. NSAIDs are appropriate to treat pain after a fracture.

24. When giving home care instructions to a patient who has comminuted forearm fractures and a long-arm cast on the left arm, which information should the nurse include? a. Keep the left shoulder elevated on a pillow or cushion. b. Keep the hand immobile to prevent soft tissue swelling. c. Call the health care provider for increased swelling or numbness of the hand. d. Avoid nonsteroidal antiinflammatory drugs (NSAIDs) for 24 hours after the injury.

ANS: C Increased swelling or numbness may indicate increased pressure at the injury, and the health care provider should be notified immediately to avoid damage to nerves and other tissues. The patient should be encouraged to move the joints above and below the cast to avoid stiffness. There is no need to elevate the shoulder, although the forearm should be elevated to reduce swelling. NSAIDs are appropriate to treat pain after a fracture. DIF: Cognitive Level: Apply (application) REF: 1520 TOP: Nursing Process: Implementation MSC: NCLEX: Physiological Integrity

36. A 42-year-old patient is admitted to the emergency department with a left femur fracture. Which information obtained by the nurse is most important to report to the health care provider? a. Ecchymosis of the left thigh b. Complaints of severe thigh pain c. Slow capillary refill of the left foot d. Outward pointing toes on the left foot

ANS: C Prolonged capillary refill may indicate complications such as arterial damage or compartment syndrome. The other findings are typical with a left femur fracture. DIF: Cognitive Level: Apply (application) REF: 1517-1518 OBJ: Special Questions: Prioritization TOP: Nursing Process: Assessment MSC: NCLEX: Physiological Integrity

A patient who has a proximal humerus fracture that is immobilized with a left-sided long-arm cast and a sling is admitted to the medical-surgical unit. Which nursing intervention will be included in the plan of care? a. Use surgical net dressing to hang the arm from an IV pole. b. Immobilize the fingers on the left hand with gauze dressings. c. Assess the left axilla and change absorbent dressings as needed. d. Assist the patient in passive range of motion (ROM) for the right arm.

ANS: C The axilla can become excoriated when a sling is used to support the arm, and the nurse should check the axilla and apply absorbent dressings to prevent this. A patient with a sling would not have traction applied by hanging. The patient will be encouraged to move the fingers on the injured arm to maintain function and to help decrease swelling. The patient will do active ROM on the uninjured side.

31. A patient who arrives at the emergency department experiencing severe left knee pain is diagnosed with a patellar dislocation. The initial patient teaching by the nurse will focus on the need for a. a knee immobilizer. b. gentle knee flexion. c. monitored anesthesia care. d. physical activity restrictions.

ANS: C The first goal of collaborative management is realignment of the knee to its original anatomic position, which will require anesthesia or monitored anesthesia care (MAC), formerly called conscious sedation. Immobilization, gentle range-of-motion (ROM) exercises, and discussion about activity restrictions will be implemented after the knee is realigned. DIF: Cognitive Level: Apply (application) REF: 1508 OBJ: Special Questions: Prioritization TOP: Nursing Process: Implementation MSC: NCLEX: Physiological Integrity

Following a motor vehicle accident, a patient arrives in the emergency department with massive right lower leg swelling. Which action will the nurse take first? a. Elevate the leg on pillows. b. Apply a compression bandage. c. Check leg pulses and sensation. d. Place ice packs on the lower leg.

ANS: C The initial action by the nurse will be to assess the circulation to the leg and to observe for any evidence of injury such as fractures or dislocations. After the initial assessment, the other actions may be appropriate based on what is observed during the assessment.

Following a motorcycle accident, a 58-year-old patient arrives in the emergency department with massive left lower leg swelling. Which action will the nurse take first? a. Elevate the leg on 2 pillows. b. Apply a compression bandage. c. Check leg pulses and sensation. d. Place ice packs on the lower leg.

ANS: C The initial action by the nurse will be to assess the circulation to the leg and to observe for any evidence of injury such as fractures or dislocations. After the initial assessment, the other actions may be appropriate, based on what is observed during the assessment.

32. Following a motorcycle accident, a 58-year-old patient arrives in the emergency department with massive left lower leg swelling. Which action will the nurse take first? a. Elevate the leg on 2 pillows. b. Apply a compression bandage. c. Check leg pulses and sensation. d. Place ice packs on the lower leg.

ANS: C The initial action by the nurse will be to assess the circulation to the leg and to observe for any evidence of injury such as fractures or dislocations. After the initial assessment, the other actions may be appropriate, based on what is observed during the assessment. DIF: Cognitive Level: Apply (application) REF: 1518 OBJ: Special Questions: Prioritization TOP: Nursing Process: Implementation MSC: NCLEX: Physiological Integrity

A pedestrian who was hit by a car is admitted to the emergency department with possible right lower leg fractures. The initial action by the nurse should be to a. elevate the right leg. b. splint the lower leg. c. check the pedal pulses. d. verify tetanus immunizations.

ANS: C The initial nursing action should be assessment of the neurovascular status of the injured leg. After assessment, the nurse may need to splint and elevate the leg, based on the assessment data. Information about tetanus immunizations should be done if there is an open wound.

33. A pedestrian who was hit by a car is admitted to the emergency department with possible right lower leg fractures. The initial action by the nurse should be to a. elevate the right leg. b. splint the lower leg. c. check the pedal pulses. d. verify tetanus immunizations.

ANS: C The initial nursing action should be assessment of the neurovascular status of the injured leg. After assessment, the nurse may need to splint and elevate the leg, based on the assessment data. Information about tetanus immunizations should be done if there is an open wound. DIF: Cognitive Level: Apply (application) REF: 1518 OBJ: Special Questions: Prioritization TOP: Nursing Process: Implementation MSC: NCLEX: Physiological Integrity

After the health care provider has recommended amputation for a patient who has nonhealing ischemic foot ulcers, the patient tells the nurse that he would rather die than have an amputation. Which response by the nurse is best? a. "You are upset, but you may lose the foot anyway." b. "Many people are able to function with a foot prosthesis." c. "Tell me what you know about your options for treatment." d. "If you do not want an amputation, you do not have to have it."

ANS: C The initial nursing action should be to assess the patient's knowledge level and feelings about the options available. Discussion about the patient's option to not have the procedure, the seriousness of the condition, or rehabilitation after the procedure may be appropriate after the nurse knows more about the patient's current level of knowledge and emotional state.

After the health care provider has recommended an amputation for a patient who has ischemic foot ulcers, the patient tells the nurse, "If they want to cut off my foot, they should just shoot me instead." Which response by the nurse is best? a. "Many people are able to function normally with a foot prosthesis." b. "I understand that you are upset, but you may lose the foot anyway." c. "Tell me what you know about what your options for treatment are." d. "If you do not want the surgery, you do not have to have an amputation."

ANS: C The initial nursing action should be to assess the patient's knowledge level and feelings about the options available. Discussion about the patient's option to not have the procedure, the seriousness of the condition, or rehabilitation after the procedure may be appropriate after the nurse knows more about the patient's current level of knowledge and emotional state.

18. After the health care provider has recommended amputation for a patient who has nonhealing ischemic foot ulcers, the patient tells the nurse that he would rather die than have an amputation. Which response by the nurse is best? a. "You are upset, but you may lose the foot anyway." b. "Many people are able to function with a foot prosthesis." c. "Tell me what you know about your options for treatment." d. "If you do not want an amputation, you do not have to have it."

ANS: C The initial nursing action should be to assess the patient's knowledge level and feelings about the options available. Discussion about the patient's option to not have the procedure, the seriousness of the condition, or rehabilitation after the procedure may be appropriate after the nurse knows more about the patient's current level of knowledge and emotional state. DIF: Cognitive Level: Apply (application) REF: 1531 TOP: Nursing Process: Implementation MSC: NCLEX: Psychosocial Integrity

The second day after admission with a fractured pelvis, a 64-year-old patient suddenly develops confusion. Which action should the nurse take first? a. Take the blood pressure. b. Assess patient orientation. c. Check the oxygen saturation. d. Observe for facial asymmetry.

ANS: C The patient's history and clinical manifestations suggest a fat embolus. The most important assessment is oxygenation. The other actions are also appropriate but will be done after the nurse assesses gas exchange.

35. The second day after admission with a fractured pelvis, a 64-year-old patient suddenly develops confusion. Which action should the nurse take first? a. Take the blood pressure. b. Assess patient orientation. c. Check the oxygen saturation. d. Observe for facial asymmetry.

ANS: C The patient's history and clinical manifestations suggest a fat embolus. The most important assessment is oxygenation. The other actions are also appropriate but will be done after the nurse assesses gas exchange. DIF: Cognitive Level: Apply (application) REF: 1523 OBJ: Special Questions: Prioritization TOP: Nursing Process: Assessment MSC: NCLEX: Physiological Integrity

In which order will the nurse take these actions when caring for a patient in the emergency department with a right leg fracture after a motor vehicle accident? (Put a comma and a space between each answer choice [A, B, C, D, E, F].) a. Obtain x-rays. b. Check pedal pulses. c. Assess lung sounds. d. Take blood pressure. e. Apply splint to the leg. f. Administer tetanus prophylaxis.

ANS: C, D, B, E, A, F The initial actions should be to ensure that airway, breathing, and circulation are intact. This should be followed by checking the neurovascular status of the leg (before and after splint application). Application of a splint to immobilize the leg should be done before sending the patient for x-rays. The tetanus prophylaxis is the least urgent of the actions.

34. The day after a 60-year-old patient has an open reduction and internal fixation (ORIF) for an open, displaced tibial fracture, the priority nursing diagnosis is a. activity intolerance related to deconditioning. b. risk for constipation related to prolonged bed rest. c. risk for impaired skin integrity related to immobility. d. risk for infection related to disruption of skin integrity.

ANS: D A patient having an ORIF is at risk for problems such as wound infection and osteomyelitis. After an ORIF, patients typically are mobilized starting the first postoperative day, so problems caused by immobility are not as likely

34. The day after a 60-year-old patient has an open reduction and internal fixation (ORIF) for an open, displaced tibial fracture, the priority nursing diagnosis is a. activity intolerance related to deconditioning. b. risk for constipation related to prolonged bed rest. c. risk for impaired skin integrity related to immobility. d. risk for infection related to disruption of skin integrity.

ANS: D A patient having an ORIF is at risk for problems such as wound infection and osteomyelitis. After an ORIF, patients typically are mobilized starting the first postoperative day, so problems caused by immobility are not as likely.

22. Which action will the nurse include in the plan of care for a patient who has had a total right knee arthroplasty? a. Avoid extension of the right knee beyond 120 degrees. b. Use a compression bandage to keep the right knee flexed. c. Teach about the need to avoid weight bearing for 4 weeks. d. Start progressive knee exercises to obtain 90-degree flexion.

ANS: D After knee arthroplasty, active or passive flexion exercises are used to obtain a 90-degree flexion of the knee. The goal for extension of the knee will be 180 degrees. A compression bandage is used to hold the knee in an extended position after surgery. Full weight bearing is expected before discharge.

22. Which action will the nurse include in the plan of care for a patient who has had a total right knee arthroplasty? a. Avoid extension of the right knee beyond 120 degrees. b. Use a compression bandage to keep the right knee flexed. c. Teach about the need to avoid weight bearing for 4 weeks. d. Start progressive knee exercises to obtain 90-degree flexion.

ANS: D After knee arthroplasty, active or passive flexion exercises are used to obtain a 90-degree flexion of the knee. The goal for extension of the knee will be 180 degrees. A compression bandage is used to hold the knee in an extended position after surgery. Full weight bearing is expected before discharge.

44. After change-of-shift report, which patient should the nurse assess first? a. Patient with a Colles' fracture who has right wrist swelling and deformity b. Patient with a intracapsular left hip fracture whose leg is externally rotated c. Patient with a repaired mandibular fracture who is complaining of facial pain d. Patient with right femoral shaft fracture whose thigh is swollen and ecchymotic

ANS: D Swelling and bruising after a femoral shaft fracture suggest hemorrhage and risk for compartment syndrome. The nurse should assess the patient rapidly and then notify the health care provider. The other patients have symptoms that are typical for their injuries, but do not require immediate intervention.

In developing a care plan for a patient with an open reduction and internal fixation (ORIF) of an open, displaced fracture of the tibia, the priority nursing diagnosis is a. activity intolerance related to deconditioning. b. risk for constipation related to prolonged bed rest. c. risk for impaired skin integrity related to immobility. d. risk for infection related to disruption of skin integrity.

ANS: D A patient having an ORIF is at risk for problems such as wound infection and osteomyelitis. After an ORIF, patients typically are mobilized starting the first postoperative day, so problems caused by immobility are not as likely.

44. After change-of-shift report, which patient should the nurse assess first? a. Patient with a Colles' fracture who has right wrist swelling and deformity b. Patient with a intracapsular left hip fracture whose leg is externally rotated c. Patient with a repaired mandibular fracture who is complaining of facial pain d. Patient with right femoral shaft fracture whose thigh is swollen and ecchymotic

ANS: D Swelling and bruising after a femoral shaft fracture suggest hemorrhage and risk for compartment syndrome. The nurse should assess the patient rapidly and then notify the health care provider. The other patients have symptoms that are typical for their injuries, but do not require immediate intervention.

41. Which finding in a patient with a Colles' fracture of the left wrist is most important to communicate to the health care provider? a. Swelling is noted around the wrist. b. The patient is reporting severe pain. c. The wrist has a deformed appearance. d. Capillary refill to the fingers is prolonged.

ANS: D Swelling, pain, and deformity are common findings with a Colles' fracture. Prolonged capillary refill indicates decreased circulation and risk for ischemia. This is not an expected finding and should be immediately reported.

41. Which finding in a patient with a Colles' fracture of the left wrist is most important to communicate to the health care provider? a. Swelling is noted around the wrist. b. The patient is reporting severe pain. c. The wrist has a deformed appearance. d. Capillary refill to the fingers is prolonged.

ANS: D Swelling, pain, and deformity are common findings with a Colles' fracture. Prolonged capillary refill indicates decreased circulation and risk for ischemia. This is not an expected finding and should be immediately reported.

7. A 48-year-old patient with a comminuted fracture of the left femur has Buck's traction in place while waiting for surgery. To assess for pressure areas on the patient's back and sacral area and to provide skin care, the nurse should a. loosen the traction and help the patient turn onto the unaffected side. b. place a pillow between the patient's legs and turn gently to each side. c. turn the patient partially to each side with the assistance of another nurse. d. have the patient lift the buttocks by bending and pushing with the right leg.

ANS: D The patient can lift the buttocks off the bed by using the left leg without changing the right-leg alignment. Turning the patient will tend to move the leg out of alignment. Disconnecting the traction will interrupt the weight needed to immobilize and align the fracture

7. A 48-year-old patient with a comminuted fracture of the left femur has Buck's traction in place while waiting for surgery. To assess for pressure areas on the patient's back and sacral area and to provide skin care, the nurse should a. loosen the traction and help the patient turn onto the unaffected side. b. place a pillow between the patient's legs and turn gently to each side. c. turn the patient partially to each side with the assistance of another nurse. d. have the patient lift the buttocks by bending and pushing with the right leg.

ANS: D The patient can lift the buttocks off the bed by using the left leg without changing the right-leg alignment. Turning the patient will tend to move the leg out of alignment. Disconnecting the traction will interrupt the weight needed to immobilize and align the fracture.

21. The nurse is caring for a patient who is to be discharged from the hospital 5 days after insertion of a femoral head prosthesis using a posterior approach. Which statement by the patient indicates a need for additional instruction? a. "I should not cross my legs while sitting." b. "I will use a toilet elevator on the toilet seat." c. "I will have someone else put on my shoes and socks." d. "I can sleep in any position that is comfortable for me."

ANS: D The patient needs to sleep in a position that prevents excessive internal rotation or flexion of the hip. The other patient statements indicate that the patient has understood the teaching.

21. The nurse is caring for a patient who is to be discharged from the hospital 5 days after insertion of a femoral head prosthesis using a posterior approach. Which statement by the patient indicates a need for additional instruction? a. "I should not cross my legs while sitting." b. "I will use a toilet elevator on the toilet seat." c. "I will have someone else put on my shoes and socks." d. "I can sleep in any position that is comfortable for me."

ANS: D The patient needs to sleep in a position that prevents excessive internal rotation or flexion of the hip. The other patient statements indicate that the patient has understood the teaching.

2. A factory line worker has repetitive strain syndrome in the left elbow. The nurse will plan to teach the patient about a. surgical options. b. elbow injections. c. wearing a left wrist splint. d. modifying arm movements.

ANS: D Treatment for repetitive strain syndrome includes changing the ergonomics of the activity. Elbow injections and surgery are not initial options for this type of injury. A wrist splint might be used for hand or wrist pain.

2. A factory line worker has repetitive strain syndrome in the left elbow. The nurse will plan to teach the patient about a. surgical options. b. elbow injections. c. wearing a left wrist splint. d. modifying arm movements.

ANS: D Treatment for repetitive strain syndrome includes changing the ergonomics of the activity. Elbow injections and surgery are not initial options for this type of injury. A wrist splint might be used for hand or wrist pain.

The day after a 60-year-old patient has an open reduction and internal fixation (ORIF) for an open, displaced tibial fracture, the priority nursing diagnosis is a. activity intolerance related to deconditioning. b. risk for constipation related to prolonged bed rest. c. risk for impaired skin integrity related to immobility. d. risk for infection related to disruption of skin integrity.

ANS: D A patient having an ORIF is at risk for problems such as wound infection and osteomyelitis. After an ORIF, patients typically are mobilized starting the first postoperative day, so problems caused by immobility are not as likely.

34. The day after a 60-year-old patient has an open reduction and internal fixation (ORIF) for an open, displaced tibial fracture, the priority nursing diagnosis is a. activity intolerance related to deconditioning. b. risk for constipation related to prolonged bed rest. c. risk for impaired skin integrity related to immobility. d. risk for infection related to disruption of skin integrity.

ANS: D A patient having an ORIF is at risk for problems such as wound infection and osteomyelitis. After an ORIF, patients typically are mobilized starting the first postoperative day, so problems caused by immobility are not as likely. DIF: Cognitive Level: Apply (application) REF: 1514 OBJ: Special Questions: Prioritization TOP: Nursing Process: Diagnosis MSC: NCLEX: Physiological Integrity

Which action will the nurse include in the plan of care for a patient who has had a total right knee arthroplasty? a. Avoid extension of the right knee beyond 120 degrees. b. Use a compression bandage to keep the right knee flexed. c. Teach about the need to avoid weight bearing for 4 weeks. d. Start progressive knee exercises to obtain 90-degree flexion.

ANS: D After knee arthroplasty, active or passive flexion exercises are used to obtain a 90-degree flexion of the knee. The goal for extension of the knee will be 180 degrees. A compression bandage is used to hold the knee in an extended position after surgery. Full weight bearing is expected before discharge.

22. Which action will the nurse include in the plan of care for a patient who has had a total right knee arthroplasty? a. Avoid extension of the right knee beyond 120 degrees. b. Use a compression bandage to keep the right knee flexed. c. Teach about the need to avoid weight bearing for 4 weeks. d. Start progressive knee exercises to obtain 90-degree flexion.

ANS: D After knee arthroplasty, active or passive flexion exercises are used to obtain a 90-degree flexion of the knee. The goal for extension of the knee will be 180 degrees. A compression bandage is used to hold the knee in an extended position after surgery. Full weight bearing is expected before discharge. DIF: Cognitive Level: Apply (application) REF: 1535 TOP: Nursing Process: Planning MSC: NCLEX: Physiological Integrity

Which action will the nurse take in order to evaluate the effectiveness of Buck's traction for a patient who has an intracapsular fracture of the left femur? a. Assess for hip contractures. b. Monitor for hip dislocation. c. Check the peripheral pulses. d. Ask about left hip pain level.

ANS: D Buck's traction keeps the leg immobilized and reduces painful muscle spasm. Hip contractures and dislocation are unlikely to occur in this situation. The peripheral pulses will be assessed, but this does not help in evaluating the effectiveness of Buck's traction.

When giving home care instructions to a patient who has multiple forearm fractures and a long-arm cast on the right arm, which information should the nurse include? a. Keep the hand immobile to prevent soft tissue swelling. b. Keep the right shoulder elevated on a pillow or cushion. c. Avoid the use of nonsteroidal anti-inflammatory drugs (NSAIDs) for the first 48 hours after the injury. d. Call the health care provider for increased swelling or numbness.

ANS: D Increased swelling or numbness may indicate increased pressure at the injury, and the health care provider should be notified immediately to avoid damage to nerves and other tissues. The patient should be encouraged to move the joints above and below the cast to avoid stiffness. There is no need to elevate the shoulder, although the forearm should be elevated to reduce swelling. NSAIDs are appropriate to treat pain after a fracture.

Which information obtained by the emergency department nurse when admitting a patient with a left femur fracture is most important to report to the health care provider? a. Bruising of the left thigh b. Complaints of left thigh pain c. Outward pointing toes on the left foot d. Prolonged capillary refill of the left foot

ANS: D Prolonged capillary refill may indicate complications such as arterial damage or compartment syndrome. The other findings are typical with a left femur fracture.

When working with a patient whose job involves many hours of word processing, the nurse will teach the patient about the need to a. do stretching and warm-up exercises before starting work. b. wrap the wrists with a compression bandage every morning. c. use acetaminophen (Tylenol) instead of nonsteroidal anti-inflammatory drugs (NSAIDs) for wrist pain. d. obtain a keyboard pad to support the wrist while word processing.

ANS: D Repetitive strain injuries caused by prolonged times working at a keyboard can be prevented by the use of a pad that will keep the wrists in a straight position. Stretching exercises during the day may be helpful, but these would not be needed before starting. Use of a compression bandage is not needed, although a splint may be used for carpal tunnel syndrome. NSAIDs are appropriate to use to decrease swelling.

44. After change-of-shift report, which patient should the nurse assess first? a. Patient with a Colles' fracture who has right wrist swelling and deformity b. Patient with a intracapsular left hip fracture whose leg is externally rotated c. Patient with a repaired mandibular fracture who is complaining of facial pain d. Patient with right femoral shaft fracture whose thigh is swollen and ecchymotic

ANS: D Swelling and bruising after a femoral shaft fracture suggest hemorrhage and risk for compartment syndrome. The nurse should assess the patient rapidly and then notify the health care provider. The other patients have symptoms that are typical for their injuries, but do not require immediate intervention. DIF: Cognitive Level: Analyze (analysis) REF: 1512 OBJ: Special Questions: Prioritization; Multiple Patients TOP: Nursing Process: Assessment MSC: NCLEX: Safe and Effective Care Environment

A patient in the emergency department who is experiencing severe pain is diagnosed with a patellar dislocation. The initial patient teaching by the nurse will focus on the need for a. a knee immobilizer. b. gentle knee flexion. c. activity restrictions. d. monitored anesthesia care (conscious sedation).

ANS: D The first goal of collaborative management is realignment of the knee to its original anatomic position, which will require anesthesia or monitored anesthesia care (MAC), formerly called conscious sedation. Immobilization, gentle range of motion (ROM) exercises, and discussion about activity restrictions will be implemented after the knee is realigned.

A patient is admitted to the emergency department with possible left lower leg fractures. The initial action by the nurse should be to a. elevate the left leg. b. splint the lower leg. c. obtain information about the tetanus immunization status. d. check the popliteal, dorsalis pedis, and posterior tibial pulses.

ANS: D The initial nursing action should be assessment of the neurovascular status of the injured leg. After assessment, the nurse may need to splint and elevate the leg, based on the assessment data. Information about tetanus immunizations should be done if there is an open wound.

A 48-year-old patient with a comminuted fracture of the left femur has Buck's traction in place while waiting for surgery. To assess for pressure areas on the patient's back and sacral area and to provide skin care, the nurse should a. loosen the traction and help the patient turn onto the unaffected side. b. place a pillow between the patient's legs and turn gently to each side. c. turn the patient partially to each side with the assistance of another nurse. d. have the patient lift the buttocks by bending and pushing with the right leg.

ANS: D The patient can lift the buttocks off the bed by using the left leg without changing the right-leg alignment. Turning the patient will tend to move the leg out of alignment. Disconnecting the traction will interrupt the weight needed to immobilize and align the fracture.

A patient with a comminuted fracture of the right femur has Buck's traction in place while waiting for surgery. To assess for pressure areas on the patient's back and sacral area and to provide skin care, the nurse should a. loosen the traction and have the patient turn onto the unaffected side. b. place a pillow between the patient's legs and turn gently to each side. c. turn the patient partially to each side with the assistance of another nurse. d. have the patient lift the buttocks by bending and pushing with the left leg.

ANS: D The patient can lift the buttocks off the bed by using the left leg without changing the right-leg alignment. Turning the patient will tend to move the leg out of alignment. Disconnecting the traction will interrupt the weight needed to immobilize and align the fracture.

A patient is to be discharged from the hospital 4 days after insertion of a femoral head prosthesis using a posterior approach. A statement by the patient that indicates a need for additional discharge instructions is a. "I should not cross my legs while sitting." b. "I will use a toilet elevator on the toilet seat." c. "I will have someone else put on my shoes and socks." d. "I can sleep in any position that is comfortable for me."

ANS: D The patient needs to sleep in a position that prevents excessive internal rotation or flexion of the hip. The other patient statements indicate that the patient has understood the teaching.

The nurse is caring for a patient who is to be discharged from the hospital 5 days after insertion of a femoral head prosthesis using a posterior approach. Which statement by the patient indicates a need for additional instruction? a. "I should not cross my legs while sitting." b. "I will use a toilet elevator on the toilet seat." c. "I will have someone else put on my shoes and socks." d. "I can sleep in any position that is comfortable for me."

ANS: D The patient needs to sleep in a position that prevents excessive internal rotation or flexion of the hip. The other patient statements indicate that the patient has understood the teaching.

The second day after admission with a fractured pelvis, a patient develops acute onset confusion. Which action should the nurse take first? a. Take the blood pressure. b. Assess patient orientation. c. Check pupil reaction to light. d. Assess the oxygen saturation.

ANS: D The patient's history and clinical manifestations suggest a fat embolus. The most important assessment is oxygenation. The other actions also are appropriate but will be done after the nurse assesses gas exchange.

A checkout clerk in a grocery store has repetitive strain syndrome in the left elbow. The nurse will plan to teach the patient about a. surgical options. b. elbow injections. c. utilization of a left wrist splint. d. modifications in arm movement.

ANS: D Treatment for repetitive strain syndrome includes changing the ergonomics of the activity. Elbow injections and surgery are not initial options for this type of injury. A wrist splint might be used for hand or wrist pain.

A factory line worker has repetitive strain syndrome in the left elbow. The nurse will plan to teach the patient about a. surgical options. b. elbow injections. c. wearing a left wrist splint. d. modifying arm movements.

ANS: D Treatment for repetitive strain syndrome includes changing the ergonomics of the activity. Elbow injections and surgery are not initial options for this type of injury. A wrist splint might be used for hand or wrist pain.

2. A factory line worker has repetitive strain syndrome in the left elbow. The nurse will plan to teach the patient about a. surgical options. b. elbow injections. c. wearing a left wrist splint. d. modifying arm movements.

ANS: D Treatment for repetitive strain syndrome includes changing the ergonomics of the activity. Elbow injections and surgery are not initial options for this type of injury. A wrist splint might be used for hand or wrist pain. DIF: Cognitive Level: Apply (application) REF: 1509 TOP: Nursing Process: Planning MSC: NCLEX: Physiological Integrity

What modifications should be made for the patient going home after hip replacement? A. Elevate the toilet seat. B. Pad side rails. C. Use a bathmat when taking baths. D. Do not drive for 2 weeks.

Answer: A Rationale: An elevated toilet seat is required to prevent the hip from more than 90 degrees of flexion. Padded side rails are used for patients with seizures. Showers, not baths, should be used the first 4 to 6 weeks. No driving is allowed for at least 4 to 6 weeks.

You explain to a patient with a distal tibial fracture who is returning for a 3-week checkup that healing is indicated by A. formation of callus. B. complete bony union. C. hematoma at fracture site. D. presence of granulation tissue.

Answer: A Rationale: Bone goes through a remarkable reparative process of self-healing (called union) that occurs in stages. The third stage is callus formation. As minerals (calcium, phosphorus, and magnesium) and new bone matrix are deposited in the osteoid, an unorganized network of bone is formed that is woven around the fractured parts. Callus is primarily composed of cartilage, osteoblasts, calcium, and phosphorus. It usually appears by the end of the second week after injury. Evidence of callus formation can be verified by x-ray.

What co-morbidity can cause bursitis? A. Increased serum uric acid B. Increased eosinophils C. Sickle cell anemia D. Joint effusion

Answer: A Rationale: Bursitis results from repeated or excessive trauma or friction, gout, rheumatoid arthritis, or infection. Gout has high levels of uric acid that result in the formation of urate crystals. Increased eosinophils are related to an allergic response. Sickle cell anemia has joint pain related to hemorrhage into the joint. Joint effusion is a symptom of bursitis, not a cause.

What should the patient be taught after a hip arthroplasty? A. Setting gluteal muscles exercises B. Swinging the leg side-to-side to strengthen quadriceps C. Flexion exercises to prevent joint "freezing" D. Elevating the foot to heart level to minimize edema

Answer: A Rationale: Exercises are done to restore strength and muscle tone in the quadriceps and other muscles to improve function and range of motion. They include quadriceps setting (e.g., tightening the kneecap) and gluteal muscle setting (e.g., tightening the buttocks). The leg is raised and swung, but never crosses the midline. There should not be 90-degree flexion of the hip for 4 to 6 weeks. Flexion exercises and raising the leg are not done because they cause more than 90 degrees of flexion.

What is the appropriate nursing action for a patient with CTS? A. Put the patient's wrist in a hyperextended position. B. Remind the patient not to lift heavy objects. C. Assess the hands for discoloration and brittle nails. D. Inform the patient that surgery is a cure for CTS.

Answer: B Rationale: It is correct to remind the patient not to lift heavy objects for 4 to 6 weeks. The splint should be in a neutral or slightly extended position. Assessment for discoloration and brittle nails is not important for this diagnosis. CTS can recur; paraesthesia and pain can last for an extended time after surgery.

During the postoperative period, you instruct the patient with an above-the-knee amputation that the residual limb should not be routinely elevated because this position promotes A. hip flexion contractures. B. skin irritation and breakdown. C. clot formation at the incision site. D. increased risk of wound dehiscence.

Answer: A Rationale: Flexion contractures may delay the rehabilitation process after amputations. The most common and debilitating contracture is hip flexion. To prevent flexion contractures, have patients avoid sitting in a chair for more than 1 hour with hips flexed or having pillows under the surgical extremity. Unless specifically contraindicated, patients should lie on their abdomen for 30 minutes three or four times each day and position the hip in extension while prone.

The patient has a long cast applied to the right leg. You are uncertain if the right foot feels cool to touch. What is the best method to make this determination? A. Compare the right foot with the left foot. B. Ask the patient if the foot feels cold to him. C. Apply a sock and recheck it in 1 hour. D. Have the patient dorsiflex the ankle, and reassess the situation.

Answer: A Rationale: Neurovascular assessment includes a peripheral vascular assessment (color, temperature, capillary refill, peripheral pulses, and edema) and a peripheral neurologic assessment (sensation, motor function, and pain). Through the assessment, compare both extremities to obtain an accurate assessment. The patient may not be aware of a change, especially in the presence of co-morbidities such as diabetes. If the patient is experiencing neurovascular changes, they must be dealt with now, not later. The issue is not to attempt other measures to improve circulation but to determine if the neurovascular compromise or compartment syndrome exists.

A patient with a fracture of the pelvis should be monitored for A. changes in urinary output. B. petechiae on the abdomen. C. a palpable lump in the buttock. D. sudden decrease in blood pressure.

Answer: A Rationale: Pelvic fractures may cause serious intraabdominal injury such as paralytic ileus, hemorrhage, and laceration of the urethra, bladder, or colon. Patients may survive the initial pelvic injury, only to die from sepsis, fat embolism syndrome, or thromboembolism. Because a pelvic fracture can damage other organs, assess bowel and urinary elimination and distal neurovascular status.

The computer operator has tingling in his hands when he dangles it. What interventions should you expect? A. Immobilize with splints B. Acetaminophen (Tylenol) C. Alternate hot and cold applications D. Wrap the wrist with ACE wrap

Answer: A Rationale: Phalen's sign is diagnostic of carpal tunnel syndrome (CTS). It is elicited by allowing the wrists to fall freely into maximum flexion and maintain that position for more than 60 seconds. A positive response is a sensation of tingling in the distribution of the median nerve over the hand. Treatment involves stopping the aggravating movement and resting the hand and wrist by immobilization in a hand splint. Hot and cold applications are not used because of the risk of thermal injury.

What is proper management of skeletal traction? A. Assess pin sites for purulent drainage. B. Remove weights intermittently to promote circulation. C. Place weights on the floor at the foot of the bed. D. Keep the maximum weight at 10 pounds (4.5 kg).

Answer: A Rationale: Skeletal traction requires a pin in the bone, and pin sites need assessment for infection. Weights should not be removed and should be free hanging. Skin traction uses only 5 to 10 pounds, but skeletal traction uses 5 to 45 pounds (2.3 to 20.4 kg).

What is the difference between a sprain and strain? A. Sprain is ligamentous injury made by twisting; strain is excessive muscle stretching. B. A sprain is initially treated with ice; strain is treated with heat. C. Sprains may require surgery, but strains are self-limiting. D. Sprains symptoms are pain and edema; strains symptoms are ecchymosis and striae

Answer: A Rationale: Sprain is an injury to ligamentous structures surrounding a joint, usually caused by twisting motion. A strain is excessive stretching of a muscle and its fascial sheath, often involving a tendon. Both are treated with RICE (rest, ice, compression, elevation). Both are usually self-limiting, although severe strains may require surgical repair. Both have similar symptoms of pain, edema, and decreased function.

What symptoms reported by a grocery store clerk and bagger are most indicative of repetitive strain injury? A. Numbness and pain in the shoulder B. Shoulder bump and inability to raise the same-side arm C. Weakness in the thumb and clumsiness with fine hand movements D. Point tenderness and unilateral forearm swelling

Answer: A Rationale: Symptoms include pain, weakness, numbness, and impairment of motor function. The shoulder bump and lack of function indicate a dislocation. Thumb weakness and clumsiness indicate carpal tunnel syndrome. Point tenderness and unilateral forearm swelling likely indicate a fracture

What is the classic presentation of a hip fracture? A. Shortened external rotation of the limb B. Decreased pedal pulse in the affected limb C. Point tenderness at the iliac crest D. Abnormal pelvic movement and bruising

Answer: A Rationale: The clinical manifestations of hip fractures are shortened external rotation, muscle spasm, shortening of the affected extremity, and severe pain and tenderness in the region of the fracture site.

It is most important that you monitor for what complication in a fracture of the humerus? A. Growing pulsatile hematoma B. Abnormal mobility in extremity C. Muscle spasm D. Shortened extremity

Answer: A Rationale: The major complications associated with fracture of the humerus are radial nerve injury and vascular injury to the brachial artery as a result of laceration, transaction, or muscle spasm. A sign of a brachial artery injury is a growing pulsatile hematoma. The other options are expected findings in a patient with a fractured humerus.

You are caring for a 76-year-old man who has undergone left knee arthroplasty and prosthetic replacement of the knee joint to relieve the pain of severe osteoarthritis. What should be included in the postoperative care of the affected leg? A. Progressive leg exercises to obtain 90-degree flexion B. Early ambulation with full weight bearing on the left leg C. Bed rest for 3 days with the left leg immobilized in extension D. Immobilization of the left knee in 30-degree flexion for 2 weeks to prevent dislocation

Answer: A Rationale: The patient is encouraged to engage in progressive leg exercises until 90-degree flexion is possible. Because this is painful after surgery, the patient requires good pain management and often the use of a continuous passive motion (CPM) machine.

The patient has a suspected facial fracture. During an ocular assessment, you notice brown tissue on the surface of the globe. What action should you take? A. Place a protective shield over the eye. B. Assess CNs III, IV, and VI. C. Check for pupil constriction. D. Press gently on the globe to feel for hardness.

Answer: A Rationale: When eye globe rupture is suspected, the examination is stopped, and a protective shield is placed over the eye. Signs of globe rupture include the extrusion of vitreous humor or brown tissue (iris or ciliary body) on the surface of the globe or penetrating through a laceration with an eccentric or teardrop-shaped pupil.

The patient has a fractured humerus. What is most important for you to communicate to the nursing assistant that he or she should report immediately? A. Reports of pain B. Patient appears newly confused C. Blood pressure of 150/90 mm Hg D. Voided 900 mL/8-hour shift

Answer: B Rationale: New-onset confusion requires immediate assessment for major complications, including a fat embolism.

What should you teach the patient related to cast care? A. Apply ice directly over the fracture site for the first 24 hours. B. Do not get the cast wet. C. Elevate the extremity above the level of the heart at night. D. Move joints above and below the cast regularly. E. Fiberglass casts tolerate weight bearing.

Answer: A, B, D Rationale: Cast care includes applying ice directly over the fracture site for the first 24 hours. Casts should not become wet, and the joints are moved. The extremity should be elevated above the level of the heart for the first 48 hours. No new cast, regardless of material, should be used for weight bearing for the first 48 hours. Not all casts are made for weight bearing, and the health care provider should make the determination.

Which are ergonomic recommendations to prevent repetitive strain injury when working at a computer (select all that apply)? A. Keep hips and knees flexed to 90 degrees. B. Keep the wrist straight while typing. C. Take a stretch break every 4 hours. D. Elevate the feet to a level even with the heart. E. Have the top of the computer monitor even with the forehead.

Answer: A, B, E Rationale: Ergonomic considerations include keeping the hips and knees flexed to 90 degrees with the feet flat, keeping the wrist straight while typing, and having the top of computer monitor even with forehead. Hourly stretch breaks should be taken.

The patient with a fractured pelvis is treated with a hip spica cast. What is an appropriate nursing action? A. Position the patient by using gentle pressure on the support bar. B. Monitor the patient for abdominal pain and pressure. C. Cover the plaster cast to aid curing. D. Do not allow weight bearing on the cast for 1 hour

Answer: B Rationale: A hip spica cast is two casts joined together: a trunk or body cast and a long leg cast. A body jacket cast extends from the nipple line to the pubis. After application, it is possible to develop cast syndrome, which occurs if it is too tight. The cast compresses the superior mesenteric artery against the duodenum. Symptoms include abdominal pain, abdominal pressure, nausea, and vomiting. The support bar should not be pulled on. A plaster cast should not be covered because air cannot circulate, heat builds up in the cast, and drying can be delayed. A cast can dry in 15 minutes but is not strong enough for weight bearing until about 24 to 72 hours later.

The patient had a dislocated joint that was treated by realignment and splinting until healed. What teaching should you provide? A. Watch for signs of avascular necrosis. B. Restrict activity to prevent repeated dislocation. C. Wear an elastic support continuously. D. There is a lifelong risk for development of tophi at the site.

Answer: B Rationale: A person who has dislocated a joint may be at a greater risk for repeated dislocations because of loose ligaments. Activity restrictions may be imposed on the use of the affected joint to decrease the risk of repeatedly dislocating the joint. Avascular necrosis (bone cell death as a result of inadequate blood supply) is a risk initially with dislocated joint that is not promptly or correctly treated.

The patient had a right below-the-knee amputation yesterday and reports feeling pain and heaviness in the right lower leg today. What action should you take? A. Reassure the patient that the pain is not real. B. Administer an analgesic. C. Use distraction techniques. D. Apply ice around the upper leg.

Answer: B Rationale: Approximately 90% of amputees have phantom limb sensation. The patient feels pain, coldness, heaviness, cramping, shooting, burning, or crushing pain in the limb that has been amputated. It should be treated with an analgesic, just as if the limb was there. The sensation usually subsides over time.

What instruction is most effective in helping to prevent carpal tunnel syndrome? A. Avoid lifting more than 20 pounds. B. Keep wrists even with the keyboard when typing. C. Wear gloves when going outside in winter. D. Limit overhand throwing by grade school children.

Answer: B Rationale: Carpal tunnel syndrome is caused by compression of the median nerve from continuous wrist movement. Keep wrists even with the keyboard or use adaptive keyboards to prevent injury. The condition is not related to heavy lifting. Wearing gloves outside in the winter is taught to patients with Raynaud's syndrome. Limiting overhand throwing is related to rotator cuff injury.

An 18-year-old male twisted his ankle while playing soccer. What treatment should be done? A. Elevate the foot 6 inches. B. Put ice on for 20 minutes at a time. C. Apply bandage from proximal to distal site. D. Provide aspirin for analgesia.

Answer: B Rationale: Cold should be applied immediately but should not exceed 20 to 30 minutes per application and should not be applied directly to skin. Elevation should be above the level of the heart and done even at night. After 48 hours, heat may be used. Compression bandages should be applied from the distal to the proximal point to aid fluid return. Aspirin is not used because it can promote bleeding; instead, use a nonsteroidal antiinflammatory drug (NSAID).

What is most important in the first 24 hours after a below-the-knee amputation? A. Observe for serosanguineous drainage. B. Elevate the stump to decrease edema. C. Complete passive range-of-motion (ROM) exercise for the unaffected limb. D. Measure the stump for the prosthesis size

Answer: B Rationale: For the first 24 hours only, the stump is elevated to minimize edema and optimize venous return. It is not done for a longer period to prevent development of contracture. Some serosanguineous drainage is expected. Excessive bleeding is the concern. Passive ROM exercises are not done immediately postoperatively, and the stump must heal before prosthesis fitting.

The orthopedic patient is in skeletal traction. The patient's urine pH rose. What action should you take? A. Notify primary provider for antibiotic order. B. Provide cranberry juice to acidify the urine. C. Assess for coexisting vaginal discharge. D. Strain all urine.

Answer: B Rationale: Renal calculi can develop as a result of bone demineralization. The hypercalcemia from demineralization causes a rise in urine pH and stone formation resulting from the precipitation of calculi. Unless contraindicated, fluid intake should be 2500 mL/day, and cranberry juice is provided to acidify the urine and prevent precipitation of minerals into stones.

While completing an admission history for a 73-year-old man with osteoarthritis who was admitted for knee arthroplasty, you ask about his perception of the reason for admission. What response do you expect? A. Recent knee trauma B. Debilitating joint pain C. Repeated knee infections D. Onset of "frozen" knee joint

Answer: B Rationale: The most common reason for knee arthroplasty is debilitating joint pain despite attempts to manage it with exercise and drug therapy.

A patient with rheumatoid arthritis is scheduled for a total hip arthroplasty (THA). You explain that the purpose of this procedure is to (select all that apply) A. fuse the joint. B. replace the joint. C. prevent further damage. D. improve or maintain range of motion. E. decrease the amount of destruction in the joint.

Answer: B, D Rationale: Arthroplasty is the reconstruction or replacement of a joint. This surgical procedure is performed to relieve pain, improve or maintain range of motion, and correct deformity. THA provides significant relief of pain and improvement of function for a patient with rheumatoid arthritis.

What assessment finding indicates an acute meniscus injury in a football player? A. Localized edema B. Decreased pedal pulse C. Pinpoint tenderness D. Muscle weakness of 2/5

Answer: C Rationale: An acutely torn meniscus may be suspected when localized tenderness, pain, and effusion are identified. Pain is elicited by flexion, internal rotation, and then extension of the knee (McMurray's test). Edema usually is not significant because most of the cartilage is avascular. The meniscus injury is a local knee injury rather than affecting the entire limb.

What should the patient understand about knee arthroplasty? A. Avoid weight bearing for 6 weeks. B. It is common in patients with osteoporosis. C. A continuous passive motion (CPM) machine provides flexion. D. Alignment is changed, and weight bearing is shifted as a result.

Answer: C Rationale: Active flexion exercises or passive flexion exercises through the use of a CPM machine postoperatively may promote joint mobility. Full weight bearing is begun before discharge. Patients with osteoporosis have difficulties and may require grafting before the surgery. It is the treatment for patients with unremitting pain and instability as a result of severe destructive deterioration of the knee joint. An osteotomy is performed to remove a wedge or slide of bone to change alignment and to shift weight bearing. It is common in patients with ankylosing spondylitis.

The patient fractured his foot, and you are providing crutch-walking instructions. The patient puts his weight on the axilla. You tell the patient to put his weight on his hands, but the patient continues to put the weight on the axilla. What action should you take? A. Encourage weight lifting to build patient's upper body strength. B. Teach the patient two-point crutch walking technique instead. C. Initiate changing the patient to a walker. D. Obtain a wheelchair for the patient instead of crutches.

Answer: C Rationale: Bearing weight on the axilla rather than the hands endangers the neurovascular bundle that passes across the axilla. A common reason is inadequate upper limb strength. If the patient cannot correct the problem with verbal coaching, another form of ambulation should be provided until the strength is adequate. It is preferable to keep the patient ambulating in some form, rather than a wheelchair.

Thirty-six hours after a major trauma with pelvic and femur fractures, what finding is most important for you to monitor? A. Pulse of 100 beats/minute and blood pressure of 140/90 mm Hg B. Pain rated at 8 C. Restlessness and petechiae D. Bilateral leg edema

Answer: C Rationale: Fat embolism syndrome is characterized by the presence of systemic fat globules, and it is seen most often in fractures in the long bones, ribs, tibia, and pelvis. Clinical manifestations usually occur within 24 to 48 hours after injury, causing hemorrhagic interstitial pneumonitis and acute respiratory distress syndrome (ARDS). The change in level of consciousness and petechiae distinguish it from other problems. Bilateral leg edema can be a generalized symptom resulting from immobility and fluid resuscitation; deep vein thrombosis manifests as unilateral leg swelling

The patient had surgery for a hip fracture using a posterior approach. You must intervene when you notice the patient doing which activity? A. Sitting up at a 45-degree angle B. Putting on a sock using an extension aid C. Crossing legs at the knee D. Sitting on a raised toilet seat

Answer: C Rationale: Fractures treated with a posterior approach are at greater risk for dislocation of the femoral head prosthesis. Activities that predispose the patient to dislocation are more than 90 degrees of flexion, abduction, or internal rotation. This can occur when crossing the legs or feet when seated. The other options are acceptable actions.

You formulate a nursing diagnosis of impaired physical mobility related to decreased muscle strength for a 78-year-old patient after a left total knee replacement. What would be an appropriate nursing intervention for this patient? A. Promote vitamin D and calcium intake in the diet. B. Provide passive range of motion to all of the joints every 4 hours. C. Encourage isometric quadriceps-setting exercises at least four times daily. D. Keep the left leg in extension and abduction to prevent contractures.

Answer: C Rationale: Great emphasis is placed on postoperative exercise of the affected leg, with isometric quadriceps setting beginning on the first day after surgery.

What is an indication of a neurovascular problem noticed during assessment of a patient with a fracture? A. Exaggeration of strength with movement B. Increased redness and heat below the injury C. Decreased sensation distal to the fracture site D. Purulent drainage at the site of an open fracture

Answer: C Rationale: Musculoskeletal injuries can cause changes in the neurovascular status of the injured extremity. With musculoskeletal trauma, application of a cast or constrictive dressing, poor positioning, and the physiologic responses to the traumatic injury can cause nerve or vascular damage, usually distal to the injury. The neurovascular assessment consists of a peripheral vascular assessment (color, temperature, capillary refill, peripheral pulses, and edema) and a peripheral neurologic assessment (sensation, motor function, and pain).

The patient with rotator cuff injury has been treated with rest, ice, and nonsteroidal antiinflammatory drugs (NSAIDs), and surgery is now required. What is most important to teach the patient before surgery? A. Wound care instructions B. Need for physical therapy immediately after surgery C. Stop NSAIDs for 2 weeks before surgery D. Keep the leg elevated after surgery

Answer: C Rationale: NSAIDs affect clotting and should be stopped before surgery.

What nonoperative treatment can be provided for a fractured humerus? A. Immobilize the affected limb for 6 weeks. B. Elevate the arm and apply ice. C. Allow the affected arm to hang freely. D. Wrap the arm in an adducted position.

Answer: C Rationale: Nonoperative treatment can include a hanging arm cast and allowing the arm to hang freely when the patient is sitting and standing. These actions assist gravity in reducing the fracture. An exercise program is required to improve strength and mobility. It includes assisted motion of the hands and fingers.

The patient has been in a significant motor vehicle crash. You notice unusual pelvic movement and ecchymosis on the abdomen. What is most important for you to do? A. Place an ice pack on the abdominal area. B. Apply continuous manual traction to the lower extremities. C. Look for lacerations of the urethra before catheter insertion. D. Use a folded sheet across abdomen to immobilize the pelvis.

Answer: C Rationale: Signs of a pelvic fracture include local swelling, tenderness, deformity, unusual pelvic movement, and ecchymosis of the abdomen. Pelvic fractures may cause serious intraabdominal injury, including laceration of the urethra or bladder. A catheter should not be inserted if there are lacerations of the urethra or blood in the urine.

What would be the most distinct sign that the patient had dislocated his shoulder after the bike accident? A. Localized bruising B. Tingling at night C. Deformity D. Weakness in the thumb

Answer: C Rationale: The most obvious sign of a dislocation is deformity. Other manifestation can include local pain, local tenderness, loss of function of the injured part, and swelling of soft tissue in the joint region. Tingling at night and weakness in the thumb are seen in carpal tunnel syndrome.

You are assessing the patient who had a cast applied 12 hours earlier. The affected foot is cooler and cyanotic compared with the unaffected foot. You notify the doctor, who states it has always been like that. What action should you take next? A. Document the physician's statements. B. Continue to monitor the patient. C. Notify the charge nurse. D. Contact the hospital medical director.

Answer: C Rationale: The patient has symptoms of neurovascular compromise (alteration in the Ps: pallor, pain, pulselessness, paresthesia, and paralysis). This requires an urgent response and cannot be delayed. It is necessary for you to activate the chain of command to get the patient adequate care. Documentation is important, but patient care is first. The patient must be monitored, but there is already evidence that the patient is in need of intervention. The medical director is eventually notified, but having the patient receive adequate care is more important than letting senior administration know.

You are caring for a patient with osteoarthritis who is about to undergo total left knee arthroplasty. You assess the patient carefully to be sure that there is no evidence of which of the following in the preoperative period? A. Pain B. Left knee stiffness C. Left knee infection D. Left knee instability

Answer: C Rationale: The patient must be free of infection before a total knee arthroplasty. Joint infection can cause even greater pain and joint instability, requiring extensive surgery. You monitor the patient for signs of infection, such as redness, swelling, fever, and elevated white blood cell count.

A patient with a stable, closed fracture of the humerus caused by trauma to the arm has a temporary splint with bulky padding and an elastic bandage applied. You suspect compartment syndrome and notify the physician when the patient experiences A. increasing edema of the limb. B. muscle spasms of the lower arm. C. rebounding pulse at the fracture site. D. pain when passively extending the fingers

Answer: D Rationale: One or more of the following are characteristic of compartment syndrome: (1) paresthesia (numbness and tingling); (2) pain distal to the injury that is not relieved by opioid analgesics and pain on passive stretch of muscle traveling through the compartment; (3) pressure increasing in the compartment; (4) pallor, coolness, and loss of normal color of the extremity; (5) paralysis or loss of function; and (6) pulselessness or diminished or absent peripheral pulses. Also assess peripheral edema; pitting edema may occur with severe injury.

he patient had a knee bursectomy. Which finding is most concerning the first postoperative day? A. White blood cell (WBC) count of 11,000/μL B. Decreased breath sounds C. Hypoactive bowel sounds D. Leg pain that worsens with passive stretch

Answer: D Rationale: Pain on passive stretch is a sign of compartment syndrome. The other options are expected findings.

A patient with a comminuted fracture of the femur is scheduled to have an open reduction with internal fixation (ORIF) of the fracture. When is ORIF indicated? A. The patient is unable to tolerate prolonged immobilization. B. The patient cannot tolerate the surgery of a closed reduction. C. A temporary cast would be too unstable to provide normal mobility. D. Adequate alignment cannot be obtained by other nonsurgical methods.

Answer: D Rationale: A comminuted fracture has more than two bone fragments. ORIF is indicated for a comminuted fracture and is used to realign and maintain bony fragments. ORIF has become the preferred method to manage a femoral fracture. Other nonsurgical methods would fail to obtain satisfactory reduction. Internal fixation reduces the hospital stay and the complications associated with prolonged bed rest.

You suspect an ankle sprain when a patient at the urgent care center relates A. being hit by another soccer player during a game. B. having ankle pain after sprinting around the track. C. dropping a 10-lb weight on his lower leg at the health club. D. twisting his ankle while running bases during a baseball game.

Answer: D Rationale: A sprain is an injury to the ligamentous structures surrounding a joint, usually caused by a wrenching or twisting motion. Most sprains occur in the ankle and knee joints.

Why is a sugar-tong splint cast used for a fracture? A. Better immobilization B. Improved comfort C. Cost-effectiveness D. Allows for swelling

Answer: D Rationale: A sugar-tong splint is typically used for acute wrist injuries or injuries that may result in significant swelling.

What assessment would most likely indicate poor venous return (versus arterial insufficiency) in an affected extremity? A. Pallor B. Coldness C. Lack of pulse D. Cyanosis

Answer: D Rationale: A warm cyanotic extremity may indicate poor venous return. Pallor, coldness, and lack of pulse are indications of arterial insufficiency.

In teaching a patient scheduled for total ankle arthroplasty (TAA), it is important that you tell him to avoid what after surgery? A. Lifting heavy objects B. Sleeping on the back C. Abduction exercises of the affected ankle D. Bearing weight on affected leg for 6 weeks

Answer: D Rationale: After TAA, the patient may not bear weight for 6 weeks and must elevate the extremity to reduce and prevent edema, be extremely careful to prevent postoperative infection, and maintain immobilization as directed by the physician.

The teenager is scheduled for a below-the-knee amputation as a result of a motorcycle accident. What is most important to include in the preoperative period? A. Explain that walking with prosthesis will be possible afterward. B. Arrange for a continuation of school work. C. Encourage exercise to build upper body strength. D. Address feelings and fear related to the surgery.

Answer: D Rationale: Before surgery, dealing with feelings and fear related to body image and function is the highest priority, especially for a teenager. The other options can be done after surgery.

What is the pathophysiology of bursitis? A. Chronic inflammation that affects small joints B. Tear in fibrocartilage C. Overstretching of muscle D. Inflammation of sacs near joints

Answer: D Rationale: Bursitis is an inflammation of the bursa, a closed sac that contains a small amount of the synovial fluid located at sites of friction. Chronic inflammation that affects small joints describes rheumatoid arthritis. A fibrocartilage tear is a meniscus injury. Overstretching of muscle is strain.

The patient was not wearing a seatbelt when he drove the car into a tree. The windshield is shattered from the force of his face hitting it, and facial fracture is suspected. What other potential injury is a primary concern? A. Hypovolemic shock B. Ruptured tympanic membrane C. Cranial nerve (CN) IX and X injury D. Cervical spine injury

Answer: D Rationale: Cervical spine injuries are common when there is a facial fracture. All patients with facial injuries should be treated as if they have a cervical injury until proved otherwise. Although there is bleeding because the face is vascular, pressure packing controls hemorrhage, and loss from a facial fracture usually is not profuse. Ruptured tympanic membrane is not a primary injury. The ocular nerve and CNs III, IV, and VI are primary concerns.

What is a key etiologic factor in hip fractures? A. Immobility B. Menopause C. Diabetes D. Osteoporosis

Answer: D Rationale: Falls account for 90% of hip fractures, which occurs more commonly in women with osteoporosis.

Which precipitating activity is typically associated with bursitis? A. Planting the foot and twisting of the knee B. Falling with blunt traumatic force C. Movement causing narrowing of nerve channel D. Repetitive kneeling

Answer: D Rationale: Improper body mechanics, repetitive kneeling (carpet layers, coal miners, and gardeners), jogging in worn-out shoes, and prolonged sitting with crossed legs are common precipitating activities. Twisting of the knee is a meniscus injury. Blunt traumatic force can cause fracture. Narrowing of the nerve channel is carpal tunnel syndrome.

What is correct information about synovectomy? A. It is used for osteoarthritis. B. A slice of bone is removed to change alignment. C. Debridement removes degenerative debris. D. It is done prophylactically to prevent joint damage progression.

Answer: D Rationale: Is used as a prophylactic measure and as a palliative treatment for rheumatoid arthritis. It helps to prevent further progression of joint damage. In osteotomy, a slice of bone is removed to change alignment Debridement is a separate procedure.

The patient had facial fractures and had surgery for a mandibular fracture. What is essential postoperatively? A. Place ice packs on cheek bone areas to minimize ecchymosis. B. Place the patient in a semi-Fowler's position to minimize edema. C. Keep the patient NPO for the first 5 days. D. Tape wire cutters at the head of the bed.

Answer: D Rationale: Postoperative care after surgery for a mandibular fracture should focus on the airway. The jaw is wired shut. Wire cutters (or scissors if rubber bands are used) are taped to the head of the bed and always sent with the patient if the patient leaves the area. They are used to cut the wires (or elastic bands) in case of emergency.

The carpet installer reports that his knee "clicks and pops" and sometimes gives way. You should evaluate for what type of pathology? A. Joint subluxation B. Strain C. Sprain D. Meniscus injury

Answer: D Rationale: Signs of a meniscus injury are localized tenderness, pain, and effusion. Pain is elicited by flexion, internal rotation, and then extension of the knee (McMurray's test). The patient may feel that the knee is unstable and often reports that the knee may "click, pop, lock, or give way."

Which assessment is most helpful to determine a rotator cuff injury in a competitive swimmer? A. Pain when tapping over the median nerve in the wrist B. Shoulder radiograph C. Shoulder deformity D. Arm that suddenly drops when abducted 90 degrees and slowly lowered

Answer: D Rationale: The drop arm test, in which the arm falls suddenly after the patient is asked to slowly lower the arm to the side and abduct it to 90 degrees. Tinel's sign can be elicited by tapping over the median nerve as it passes through the carpal tunnel in the wrist. A positive response is a sensation of tingling in the distribution of the median nerve over the hand. This is a positive finding for carpal tunnel syndrome. A radiograph alone is usually not beneficial. Shoulder deformity is typically seen with a shoulder dislocation.

You are completing discharge teaching for an 80-year-old man who underwent right total hip arthroplasty. You identify a need for further instruction if the patient states the need to do which of the following? A. Avoid crossing his legs B. Use a toilet elevator on toilet seat C. Notify future caregivers about the prosthesis D. Maintain hip in adduction and internal rotation

Answer: D Rationale: The patient should not cross his legs, force the hip into adduction, or force the hip into internal rotation.

What nursing action should be done to prevent hip flexion contractions? A. Dangle the residual limb over the edge of the bed. B. Wear a limb shrinker at all times. C. Position the patient with pillows under the surgical extremity. D. Position the patient prone several times each day

Answer: D Rationale: Unless specifically contraindicated, the patient should lie on his abdomen for 30 minutes three or four times each day and position the hip in extension while prone. The residual limb should not dangle, because it contributes to edema. A limb shrinker is worn to shape the limb for eventual prosthesis fitting, not to prevent flexion contractures. The patient should not sit in a chair for more than 1 hour with the hips flexed or have pillows under the surgical extremity.

You are caring for a 75-year-old woman who underwent left total knee arthroplasty and has a new physician order to be "up in a chair today before noon." Which action would you take to protect the knee joint while carrying out the order? A. Administer a dose of prescribed analgesic before completing the order. B. Ask the physical therapist for a walker to limit weight bearing while getting out of bed. C. Keep the CPM machine in place while lifting the patient from bed to chair. D. Put on a knee immobilizer before moving the patient out of bed, and keep the surgical leg elevated while sitting.

Answer: D Rationale: You should apply a knee immobilizer for stability before assisting the patient to get out of bed. This is a standard measure to protect the knee during movement after surgery.

While completing an admission history for a 73-year-old man with osteoarthritis admitted for knee arthroplasty, the nurse asks about the patient's perception of the reason for admission. The nurse expects the patient to relate which response to this question? a. Recent knee trauma b. Debilitating joint pain c. Repeated knee infections d. Onset of "frozen" knee joint

B The most common reason for knee arthroplasty is debilitating joint pain despite attempts to manage it with exercise and drug therapy. Recent knee trauma, repeated knee infections, and onset of "frozen" knee joint are not primary indicators for a knee arthroplasty.

While completing an admission history for a 73-year-old man with osteoarthritis admitted for knee arthroplasty, the nurse asks about the patient's perception of the reason for admission. The nurse expects the patient to relate which response to this question? A Recent knee trauma B Debilitating joint pain C Repeated knee infections D Onset of "frozen" knee joint

B Debilitating joint pain The most common reason for knee arthroplasty is debilitating joint pain despite attempts to manage it with exercise and drug therapy. Recent knee trauma, repeated knee infections, and onset of "frozen" knee joint are not primary indicators for a knee arthroplasty.

A 42-year-old man who is scheduled for an arthrocentesis arrives at the outpatient surgery unit and states, "I do not want this procedure done today." Which response by the nurse is most appropriate? A. "When would you like to reschedule the procedure?" B. "Tell me what your concerns are about this procedure." C. "The procedure is safe, so why should you be worried?" D. "The procedure is not painful because an anesthetic is used."

B. "Tell me what your concerns are about this procedure." The nurse should use therapeutic communication to determine the patient's concern about the procedure. The nurse should not provide false reassurance. It is not appropriate for the nurse to assume the patient is concerned about pain or to assume the patient is asking to reschedule the procedure.

A 57-year-old postmenopausal woman is scheduled for dual-energy x-ray absorptiometry (DXA). Which statement, if made by the patient to the nurse, indicates understanding of the procedure? A. "The bone density in my heel will be measured." B. "This procedure will not cause any pain or discomfort." C. "I will not be exposed to any radiation during the procedure." D. "I will need to remove my hearing aids before the procedure."

B. "This procedure will not cause any pain or discomfort." Dual-energy x-ray absorptiometry (DXA) is painless and measures the bone mass of spine, femur, forearm, and total body with minimal radiation exposure. A quantitative ultrasound (QUS) evaluates density, elasticity, and strength of bone using ultrasound of the calcaneus (heel). Magnetic resonance imaging would require removal of objects such as hearing aids that have metal parts.

A patient with left knee pain is diagnosed with bursitis. The nurse will explain that bursitis is an inflammation of a. the synovial membrane that lines the joint. b. a small, fluid-filled sac found at some joints. c. the fibrocartilage that acts as a shock absorber in the knee joint. d. any connective tissue that is found supporting the joints of the body.

B. Bursae are fluid-filled sacs that cushion joints and bony prominences. Fibrocartilage is a solid tissue that cushions some joints. Bursae are a specific type of connective tissue. The synovial membrane lines many joints but is not a bursa.

The nurse notes crackling sounds and a grating sensation with palpation of an older patient's elbow. How will this finding be documented? a. Torticollis b. Crepitation c. Subluxation d. Epicondylitis

B. Crackling sounds and a grating sensation that accompany movement are described as crepitus or crepitation. Torticollis is a twisting of the neck to one side, subluxation is a partial dislocation of the joint, and epicondylitis is an inflammation of the elbow that causes a dull ache that increases with movement.

The nurse notes crackling sounds and a grating sensation with palpation of an older patient's elbow. How will this finding be documented? a. Torticollis b. Crepitation c. Subluxation d. Epicondylitis

B. Crepitation

A 42-year-old male patient complains of shoulder pain when the nurse moves his arm behind the back. Which question should the nurse ask? a. "Are you able to feed yourself without difficulty?" b. "Do you have difficulty when you are putting on a shirt?" c. "Are you able to sleep through the night without waking?" d. "Do you ever have trouble lowering yourself to the toilet?"

B. Do you have difficulty when you are putting on a shirt?"

The nurse who notes that a 59-year-old female patient has lost 1 inch in height over the past 2 years will plan to teach the patient about a. discography studies. b. myelographic testing. c. magnetic resonance imaging (MRI). d. dual-energy x-ray absorptiometry (DXA).

D. dual-energy x-ray absorptiometry (DXA).

Which action can the nurse delegate to unlicensed assistive personnel (UAP) who are working in the orthopedic clinic? a. Grade leg muscle strength for a patient with back pain. b. Obtain blood sample for uric acid from a patient with gout. c. Perform straight-leg-raise testing for a patient with sciatica. d. Check for knee joint crepitation before arthroscopic surgery.

B. Obtain blood sample for uric acid from a patient with gout.

Which information in a 67-year-old woman's health history will alert the nurse to the need for a more focused assessment of the musculoskeletal system? a. The patient sprained her ankle at age 13. b. The patient's mother became shorter with aging. c. The patient takes ibuprofen (Advil) for occasional headaches. d. The patient's father died of complications of miliary tuberculosis.

B. The patient's mother became shorter with aging.

A patient is admitted with an open fracture of the tibia following a bicycle accident. During assessment of the patient, what specifically should the nurse question the patient about? A. Any previous injuries to the leg. B. The status of tetanus immunization. C. The use of antibiotics in the last month. D. Whether the injury was exposed to dirt or gravel.

B. The status of tetanus immunization. Infection is the greatest risk with an open fracture and all open fractures are considered contaminated. Tetanus prevention is always indicated if the patient has not been immunized or does not have current boosters. prophylactic antibiotics are often used in management for open fractures but recent antibiotic therapy is not relevant, nor is previous injury to the site.

A patient with left knee pain is diagnosed with bursitis. The nurse will explain that bursitis is an inflammation of a. the synovial membrane that lines the joint. b. a small, fluid-filled sac found at some joints. c. the fibrocartilage that acts as a shock absorber in the knee joint. d. any connective tissue that is found supporting the joints of the body.

B. a small, fluid-filled sac found at some joints.

The home care nurse visits an 84-year-old woman with pneumonia after her discharge from the hospital. Which assessment finding would the nurse expect because of age-related changes in the musculoskeletal system? A. Positive straight-leg-raising test B. Muscle strength is scale grade 3/5 C. Lateral S-shaped curvature of the spine D. Fingers drift to the ulnar side of the forearm

B.Muscle strength is scale grade 3/5 Decreased muscle strength is an age-related change of the musculoskeletal system caused by decreased number and size of the muscle cells. The other assessment findings indicate musculoskeletal abnormalities. A positive straight-leg-raising test indicates nerve root irritation from intervertebral disk prolapse and herniation. An ulnar deviation or drift indicates rheumatoid arthritis due to tendon contracture. Scoliosis is a lateral curvature of the spine.

The nurse formulates a nursing diagnosis of impaired physical mobility related to decreased muscle strength for a 78-year-old patient following left total knee replacement. What would be an appropriate nursing intervention for this patient? a. Promote vitamin C and calcium intake in the diet. b. Provide passive range of motion to all of the joints q4hr. c. Encourage isometric quadriceps-setting exercises at least qid. d. Keep the left leg in extension and abduction to prevent contractures.

C Emphasis is placed on postoperative exercise of the affected leg, with isometric quadriceps setting beginning on the first day after surgery along with a continuous passive motion (CPM) machine. Vitamin C and calcium do not improve muscle strength, but they will facilitate healing. The patient should be able to do active range of motion to all joints. Keeping the leg in one position (extension and abduction) potentially will result in contractures.

The nurse is caring for a patient with osteoarthritis who is about to undergo total left knee arthroplasty. The nurse assesses the patient carefully to be sure that there is no evidence of what in the preoperative period? a. Pain b. Left knee stiffness c. Left knee infection d. Left knee instability

C It is critical that the patient be free of infection before a total knee arthroplasty. An infection in the joint could lead to even greater pain and joint instability, requiring extensive surgery. For this reason, the nurse monitors the patient for signs of infection, such as redness, swelling, fever, and elevated white blood cell count. Pain, knee stiffness, or instability may be present with osteoarthritis.

A 42-year-old man has a recent amputation of the left leg below the knee as a result of a heavy farm machinery accident. Which intervention should the nurse include in the plan of care for this patient? a. Sit in a chair for 1 to 2 hours three times each day. b. Dangle the residual limb for 20 to 30 minutes every 6 hours. c. Lay prone with hip extended for 30 minutes four times per day. d. Elevate the residual limb on a pillow for 4 to 5 days after surgery.

C To prevent hip flexion contractures, patients should lie on their abdomen for 30 minutes three or four times each day and position the hip in extension while prone. Patients should avoid sitting in a chair for more than 1 hour with hips flexed or having pillows under the surgical extremity. The patient should avoid dangling the residual limb over the bedside to minimize edema.

A 21-year-old female soccer player has injured her anterior crucial ligament (ACL) and is having reconstructive surgery. The nurse knows that the patient will need more teaching when the patient makes which statement? A "I probably won't be able to play soccer for 6 to 8 months." B "They will have me do range of motion with my knee soon after surgery." C "I can't wait to get this done now so I can play soccer for the next tournament." D "I will need to wear an immobilizer and progressively bear weight on my knee."

C "I can't wait to get this done now so I can play soccer for the next tournament." When the athlete has ACL reconstructive surgery, the patient does not understand the severity when planning to be back to playing soccer soon, as safe return will not occur for 6 to 8 months after initial range of motion, immobilization, and progressive weight bearing with physical therapy occurs.

The nurse formulates a nursing diagnosis of impaired physical mobility related to decreased muscle strength for a 78-year-old patient following left total knee replacement. What would be an appropriate nursing intervention for this patient? A Promote vitamin C and calcium intake in the diet. B Provide passive range of motion to all of the joints q4hr. C Encourage isometric quadriceps-setting exercises at least qid. D Keep the left leg in extension and abduction to prevent contractures.

C Encourage isometric quadriceps-setting exercises at least qid. Emphasis is placed on postoperative exercise of the affected leg, with isometric quadriceps setting beginning on the first day after surgery along with a continuous passive motion (CPM) machine. Vitamin C and calcium do not improve muscle strength, but they will facilitate healing. The patient should be able to do active range of motion to all joints. Keeping the leg in one position (extension and abduction) potentially will result in contractures.

The nurse is caring for a patient with osteoarthritis who is about to undergo total left knee arthroplasty. The nurse assesses the patient carefully to be sure that there is no evidence of what in the preoperative period? A Pain B Left knee stiffness C Left knee infection D Left knee instability

C Left knee infection It is critical that the patient be free of infection before a total knee arthroplasty. An infection in the joint could lead to even greater pain and joint instability, requiring extensive surgery. For this reason, the nurse monitors the patient for signs of infection, such as redness, swelling, fever, and elevated white blood cell count. Pain, knee stiffness, or instability may be present with osteoarthritis.

A 63-year-old woman has been taking prednisone (Deltasone) daily for several years after a kidney transplant to prevent organ rejection. What is most important for the nurse to assess? A. Staggering gait B. Ruptured tendon C. Back or neck pain D. Tardive dyskinesia

C. Back or neck pain Osteoporosis with resultant fractures is a frequent and serious complication of systemic corticosteroid therapy. The ribs and vertebrae are affected the most, and patients should be observed for signs of compression fractures (back and neck pain). Phenytoin (Dilantin) is an antiseizure medication. An adverse effect of phenytoin is an ataxic (or staggering) gait. A rare adverse effect of ciprofloxacin (Cipro) and other fluoroquinolones is tendon rupture, usually of the Achilles tendon. The highest risk is in people age 60 and older and in people taking corticosteroids. Antipsychotics and antidepressants may cause tardive dyskinesia, which is characterized by involuntary movements of the tongue and face.

Which medication information will the nurse identify as a concern for a patient's musculoskeletal status? a. The patient takes a daily multivitamin and calcium supplement. b. The patient takes hormone therapy (HT) to prevent "hot flashes." c. The patient has severe asthma and requires frequent therapy with oral corticosteroids. d. The patient has migraine headaches treated with nonsteroidal antiinflammatory drugs (NSAIDs).

C. Frequent or chronic corticosteroid use may lead to skeletal problems such as avascular necrosis and osteoporosis. The use of HT and calcium supplements will help prevent osteoporosis. NSAID use does not increase the risk for musculoskeletal problems.

After completing the health history, the nurse assessing the musculoskeletal system will begin by a. having the patient move the extremities against resistance. b. feeling for the presence of crepitus during joint movement. c. observing the patient's body build and muscle configuration. d. checking active and passive range of motion for the extremities.

C. observing the patient's body build and muscle configuration.

Which medication information will the nurse identify as a concern for a patient's musculoskeletal status? a. The patient takes a daily multivitamin and calcium supplement. b. The patient takes hormone therapy (HT) to prevent "hot flashes." c. The patient has severe asthma and requires frequent therapy with oral corticosteroids. d. The patient has migraine headaches treated with nonsteroidal antiinflammatory drugs (NSAIDs).

C. The patient has severe asthma and requires frequent therapy with oral corticosteroids.

Which information obtained during the nurse's assessment of a 30-year-old patient's nutritional-metabolic pattern may indicate the risk for musculoskeletal problems? a. The patient takes a multivitamin daily. b. The patient dislikes fruits and vegetables. c. The patient is 5 ft 2 in and weighs 180 lb. d. The patient prefers whole milk to nonfat milk.

C. The patient is 5 ft 2 in and weighs 180 lb.

Which information obtained during the nurse's assessment of a 30-year-old patient's nutritional-metabolic pattern may indicate the risk for musculoskeletal problems? a. The patient takes a multivitamin daily. b. The patient dislikes fruits and vegetables. c. The patient is 5 ft 2 in and weighs 180 lb. d. The patient prefers whole milk to nonfat milk.

C. The patient's height and weight indicate obesity, which places stress on weight-bearing joints. The use of whole milk, avoiding fruits and vegetables, and use of a daily multivitamin are not risk factors for musculoskeletal problems.

The patient works on a computer 8 hours each day. What kind of repetitive strain injury would be expected in this patient? A. Increase muscle strength with daily isometric exercise. B. Avoid exercising on concrete or hard pavement surfaces. C. Perform stretching and warm-up exercises before exercise. D. Wrap susceptible joints with elastic bandages or adhesive tape before exercise.

Carpal tunnel syndrome. Carpal tunnel syndrome would be expected related to the continuous wrist movements. Injuries of the menisci, which are fibrocartilage in the knee, are common with athletes. Radial-ulnar fractures are seen with great force such as a car accident or a fall. Rotator cuff injuries occur with sudden adduction forces applied to the cuff while the arm is held in abduction. They are commonly seen with repetitive overhead motions.

The nurse suspects an ankle sprain when a patient at the urgent care center relates a. being hit by another soccer player during a game. b. having ankle pain after sprinting around the track. c. dropping a 10-lb weight on his lower leg at the health club. d. twisting his ankle while running bases during a baseball game.

Correct answer: d Rationale: A sprain is an injury to the ligamentous structures surrounding a joint, and a wrenching or twisting motion usually causes it. Most sprains occur in the ankle and knee join

In teaching a patient scheduled for a total ankle replacement, it is important to tell the patient that after surgery he should avoid a. lifting heavy objects. b. sleeping on the back. c. abduction exercises of the affected ankle. d. bearing weight on the affected leg for 6 weeks.

Correct answer: d Rationale: After total ankle arthroplasty (TAA), the patient may not bear weight for 6 weeks and must elevate the extremity to reduce and prevent edema. The patient must be careful to prevent postoperative infection and should maintain immobilization as directed by the physician.

To prevent muscle atrophy, the nurse teaches the pt with a leg immobilized in traction to perform (select all that apply): A. flexion contraction B. tetanic contractions C. isotonic contractions D. isometric contractions E. extension contractions

Correct answer: d Rationale: Isometric contractions increase the tension within a muscle but do not produce movement. Repeated isometric contractions make muscles grow larger and stronger. Muscular atrophy (i.e., decrease in size) occurs with the absence of contraction that results from immobility.

The bone cells that function in the resorption of bone tissue are called: A. osteoids B. osteocytes C. osteocytes D. osteoblasts

Correct answer: c Rationale: Osteoclasts participate in bone remodeling by assisting in the breakdown of bone tissue.

When grading muscle strength, the nurse records a score of 3 which indicates: A. no detection of muscular contraction B. A barely detectable flicker of contraction C. active movement against full resistance without fatigue D. active movement against gravity but not against resistance

Correct answer: D Rationale: Muscle strength score of 3 indicates active movement only against gravity and not against resistance (see Table 62-4).

While obtaining subjective assessment data r/t to the musculoskeletal system, it is particularly important to ask a pt about other medical problems such as: A. hypertension B. thyroid problems C. diabetes mellitus D. chronic bronchitis

Correct answer: c Rationale: The nurse should question the patient about past medical problems because certain illnesses are known to affect the musculoskeletal system directly or indirectly. These diseases include tuberculosis, poliomyelitis, diabetes mellitus, parathyroid problems, hemophilia, rickets, soft tissue infection, and neuromuscular disabilities.

The increased risk for falls in the older adult is most likely due to: A. changes in balance B. Decrease in bone mass C. Loss of ligament elasticity D. Erosion of articular cartilage

Correct answer: a Rationale: Aging can cause changes in a person's sense of balance, making the person unsteady, and proprioception may be altered. The risk for falls also increases in older adults partly because of a loss of strength.

During the postoperative period, the nurse instructs the patient with an above-the-knee amputation that the residual limb should not be routinely elevated because this position promotes a. hip flexion contractures. b. skin irritation and breakdown. c. clot formation at the incision site. d. increased risk of wound dehiscence.

Correct answer: a Rationale: Flexion contractures may delay the rehabilitation process after amputations. The most common and debilitating contracture is hip flexion. To prevent flexion contractures, patients should avoid sitting in a chair for more than 1 hour with hips flexed or with pillows under the surgical extremity. Unless specifically contraindicated, patients should lie on the abdomen for 30 minutes three or four times each day and position the hip in extension while prone.

A normal assessment finding of the musculoskeletal system is A. No deformity or crepitation B. muscle and bone strength of 4 C. ulnar deviation and sublaxation D. angulation of bone toward midline

Correct answer: a Rationale: Normal physical assessment findings of the musculoskeletal system include normal spinal curvatures; no muscle atrophy or asymmetry; no joint swelling, deformity, or crepitation; no tenderness on palpation of muscles and joints; full range of motion of all joints without pain or laxity; and muscle strength score of 5.

A pt with tendonitis asks what the tendon does. The nurses's response in based on the knowledge that the tendons A. connect bone to muscle B. provide strength to the muscle C. lubricate joints with synovial fluid D. relieve friction between moving parts

Correct answer: a Rationale: Tendons are composed of dense, fibrous connective tissue that contains bundles of closely packed collagen fibers arranged in the same plane for additional strength. They connect the muscle sheath to adjacent bone

A pt is scheduled for an electromyogram (EMG). The nurse explains that this diagnostic test involves: A. incision or puncture of the joint capsule B. insertion of small needles into certain muscles C. administration of a radioisotope before the procedure D. placement of skin electrodes to record muscle activity

Correct answer: b Rationale: Electromyography (EMG) is an evaluation of electrical potential associated with skeletal muscle contraction. Small-gauge needles are inserted into certain muscles and attached to leads that record electrical activity of muscle. Results provide information related to lower motor neuron dysfunction and primary muscle disease.

An indication of a neurovascular problem noted during assessment of the patient with a fracture is a. exaggeration of strength with movement. b. increased redness and heat below the injury. c. decreased sensation distal to the fracture site. d. purulent drainage at the site of an open fracture.

Correct answer: c Rationale: Musculoskeletal injuries have the potential for causing changes in the neurovascular status of an injured extremity. In cases of musculoskeletal trauma, application of a cast or constrictive dressing, poor positioning, and the physiologic responses to the traumatic injury can cause nerve or vascular damage, usually distal to the injury. The neurovascular assessment consists of a peripheral evaluation (i.e., color, temperature, capillary refill, peripheral pulses, and edema) and a peripheral neurologic evaluation (i.e., sensation, motor function, and pain).

A patient is brought to the emergency department with an injured lower left leg following a fall while rock climbing. The nurse identifies the presence of a fracture based on what cardinal sign of fracture?

Obvious deformity.

A patient with a stable, closed fracture of the humerus caused by trauma to the arm has a temporary splint with bulky padding applied with an elastic bandage. The nurse suspects compartment syndrome and notifies the physician when the patient experiences a. increasing edema of the limb. b. muscle spasms of the lower arm. c. rebounding pulse at the fracture site. d. pain when passively extending the fingers.

Correct answer: d Rationale: One or more of the following are characteristic of compartment syndrome: (1) paresthesia (i.e., numbness and tingling sensation); (2) pain distal to the injury that is not relieved by opioid analgesics and, on passive stretch of muscle, pain that travels through the compartment; (3) increased pressure in the compartment; (4) pallor, coolness, and loss of normal color of the extremity; (5) paralysis or loss of function; and (6) pulselessness, or diminished or absent peripheral pulses. The examination also includes assessment of peripheral edema, especially pitting edema, which may occur with severe injury.

While performing passive range of motion for a pt, the nurse puts the ankle joint through the movements of (select all that apply): A. flexion and extension B. inversion and eversion C. pronation and supination D. flexion, extension, abduction, and abduction E. pronation, supination, rotation and circumduction

Correct answers: a, b Rationale: Common movements that occur at the ankle include inversion, eversion, flexion, and extension.

This morning a 21-year-old male patient had a long leg cast applied and wants to get up and try out his crutches before dinner. The nurse will not allow this. What is the best rationale that the nurse should give the patient for this decision? a. The cast is not dry yet, and it may be damaged while using crutches. b. The nurse does not have anyone available to accompany the patient. c. Rest, ice, compression, and elevation are in process to decrease pain. d. Excess edema and other problems are prevented when the leg is elevated for 24 hours.

D For the first 24 hours after a lower extremity cast is applied, the leg will be elevated on pillows above the heart level to avoid excessive edema and compartment syndrome. The cast will also be drying during this 24-hour period. RICE is used for soft tissue injuries, not with long leg casts.

The nurse is caring for a 75-year-old woman who underwent left total knee arthroplasty and has a new order to be "up in chair today before noon." What action should the nurse take to protect the knee joint while carrying out the order? a. Administer a dose of prescribed analgesic before completing the order. b. Ask the physical therapist for a walker to limit weight bearing while getting out of bed. c. Keep the continuous passive motion machine in place while lifting the patient from bed to chair. d. Put on a knee immobilizer before moving the patient out of bed and keep the surgical leg elevated while sitting.

D The nurse should apply a knee immobilizer for stability before assisting the patient to get out of bed. This is a standard measure to protect the knee during movement following surgery. Although an analgesic should be given before the patient gets up in the chair for the first time, it will not protect the knee joint. Full weight bearing is begun before discharge, so a walker will not be used if the patient did not need one before the surgery. The CPM machine is not kept in place while the patient is getting up to the chair.

The nurse is completing a neurovascular assessment on the patient with a tibial fracture and a cast. The feet are pulseless, pale, and cool. The patient says they are numb. What should the nurse suspect is occurring? a. Paresthesia b. Pitting edema c. Poor venous return d. Compartment syndrome

D The nurse should suspect compartment syndrome with one or more of the following six Ps: paresthesia, pallor, pulselessness, pain distal to the injury and unrelieved with opioids, pressure increases in the compartment, and paralysis. Although paresthesia and poor venous return are evident, these are just some of the manifestations of compartment syndrome.

The patient is brought to the emergency department after a car accident and has a femur fracture. What nursing intervention should the nurse implement to prevent a fat embolus in this patient? a. Administer enoxaparin (Lovenox). b. Provide range-of-motion exercises. c. Apply sequential compression boots. d. Immobilize the fracture preoperatively.

D To prevent fat emboli, the nurse immobilizes the long bone to reduce movement of the fractured bone ends and decrease the risk of a fat embolus from the bone before surgical reduction. Enoxaparin is used to prevent blood clots not fat emboli. Range of motion and compression boots will not prevent a fat embolus in this patient.

The nurse is completing a neurovascular assessment on the patient with a tibial fracture and a cast. The feet are pulseless, pale, and cool. The patient says they are numb. What should the nurse suspect is occurring? A Paresthesia B Pitting edema C Poor venous return D Compartment syndrome

D Compartment syndrome The nurse should suspect compartment syndrome with one or more of the following six Ps: paresthesia, pallor, pulselessness, pain distal to the injury and unrelieved with opioids, pressure increases in the compartment, and paralysis. Although paresthesia and poor venous return are evident, these are just some of the manifestations of compartment syndrome.

This morning a 21-year-old male patient had a long leg cast applied and wants to get up and try out his crutches before dinner. The nurse will not allow this. What is the best rationale that the nurse should give the patient for this decision? A The cast is not dry yet, and it may be damaged while using crutches. B The nurse does not have anyone available to accompany the patient. C Rest, ice, compression, and elevation are in process to decrease pain. D Excess edema and other problems are prevented when the leg is elevated for 24 hours

D Excess edema and other problems are prevented when the leg is elevated for 24 hours For the first 24 hours after a lower extremity cast is applied, the leg will be elevated on pillows above the heart level to avoid excessive edema and compartment syndrome. The cast will also be drying during this 24-hour period. RICE is used for soft tissue injuries, not with long leg casts.

The nurse is caring for a 75-year-old woman who underwent left total knee arthroplasty and has a new order to be "up in chair today before noon." What action should the nurse take to protect the knee joint while carrying out the order? A Administer a dose of prescribed analgesic before completing the order. B Ask the physical therapist for a walker to limit weight bearing while getting out of bed. C Keep the continuous passive motion machine in place while lifting the patient from bed to chair. D Put on a knee immobilizer before moving the patient out of bed and keep the surgical leg elevated while sitting.

D Put on a knee immobilizer before moving the patient out of bed and keep the surgical leg elevated while sitting. The nurse should apply a knee immobilizer for stability before assisting the patient to get out of bed. This is a standard measure to protect the knee during movement following surgery. Although an analgesic should be given before the patient gets up in the chair for the first time, it will not protect the knee joint. Full weight bearing is begun before discharge, so a walker will not be used if the patient did not need one before the surgery. The CPM machine is not kept in place while the patient is getting up to the chair.

The nurse is completing discharge teaching with an 80-year-old male patient who underwent right total hip arthroplasty. The nurse identifies a need for further instruction if the patient states the need to A avoid crossing his legs. B use a toilet elevator on toilet seat. C notify future caregivers about the prosthesis. D maintain hip in adduction and internal rotation.

D maintain hip in adduction and internal rotation. The patient should not force hip into adduction or force hip into internal rotation as these movements could displace the hip replacement. Avoiding crossing the legs, using a toilet elevator on a toilet seat, and notifying future caregivers about the prosthesis indicate understanding of discharge teaching.

A young patient with a fractured femur has a hip spica cast applied. While the cast is drying, what should the nurse do? A. Elevate the legs above the level of the heart for 24 hours. B. Turn the patient to both sides and prone to supine every 2 hours. C. Cover the cast with a light blanket to avoid chilling for evaporation. D. Assess the patient frequently for abdominal pain, nausea, and vomiting.

D. Assess the patient frequently for abdominal pain, nausea, and vomiting. Complaints of abdominal pain or pressure, nausea, and vomiting are signs of cast syndrome that occur when hip spica casts or body jacket braces are applied too tightly, causing compression of the superior mesenteric artery against the duodenum. The cast may need to be split or removed and the health care provider should be notified. Elevation is not indicated for a spica cast and the patient with a spica cast should not be placed in the prone position during the initial drying stage because the cast is so large and heavy it may break. A cast should never be covered with a blanket because heat builds up in the cast and may increase edema.

Which finding is of highest priority when the nurse is planning care for a 77-year-old patient seen in the outpatient clinic? a. Symmetric joint swelling of fingers b. Decreased right knee range of motion c. Report of left hip aching when jogging d. History of recent loss of balance and fall

D. History of recent loss of balance and fall

A 72- year- old man tells the nurse that he cannot perform most of the physical activities he could 5 years ago because of overall joint aches and pains. What can the nurse do to assist the patient to prevent further deconditioning and decrease the risk of developing musculoskeletal problems? A. Limit weight-bearing exercise to prevent stress on fragile bones and possible hip fractures. B. Advise the patient to avoid the use of canes and walkers because they increase dependence on ambulation aids. C. Advise the patient to increase his activity by more frequently climbing stairs in buildings and other environments with steps. D. Discuss use of over-the-counter (OTC) medications to decrease inflammation and pain so that exercise can be maintained.

Discuss use of over- the- counter (OTC) medications to decrease inflammation and pain so that exercise can be maintained. Almost all older adults have some degree of decreased muscle strength, joint stiffness, and pain with motion. The use of mild anti-inflammatory agents decreases inflammation and pain and can help the patient to maintain activity and prevent further deconditioning but other prescribed drugs and potential abdominal problems must be considered by the patient. Musculoskeletal problems in the older adult can be prevented with appropriate strategies, especially exercise. Stair walking can create enough stress on fragile bones to cause a hip fracture and use of ramps may help to prevent falls. Walkers and canes should be used as necessary to decrease stress on joints so that activity can be maintained.

What is a disadvantage of open reduction and internal fixation of a fracture compared to closed reduction?

Infection

The athlete comes to the clinic with burstitis. What does the nurse know happens to the tissue to cause pain when burstitis occurs? A. Tearing of a ligament. B. Stretching of muscle and fascia sheath. C. Inflammation of synovial membrane sac at friction sites. D. Incomplete separation of articular surfaces of joint caused by ligament injury.

Inflammation of the synovial membrane sac at friction sites. Bursitis is inflammation of synovial membrane sac at friction sites. Tearing of a ligament is a sprain. Stretching of muscle and fascia sheath is a strain. Incomplete separation of articular surfaces of joints caused by ligament injury is subluxation.

An immediate prosthetic fitting during surgery is used for a patient with a traumatic below- the- knee amputation. During the immediate postoperative period, what is a priority intervention?

Monitor the patient's vital signs

A patient has fallen in the bathroom of the hospital room and reports pain in the upper right arm and elbow. Before splinting the injury, the nurse knows that the priority management of a possible fracture should include which action? a. Elevation of the arm b. Application of ice to the site c. Notification of the health care provider d. Neurovascular checks below the site of the injur

Neurovascular checks below the site of injury. Pulses, sensation, and motor function distal to the injury should be checked before and after splinting to assess for nerve or vascular damage and documented to avoid doubts about whether a problem discovered later was missed during the original examination or was caused by the treatment. Elevation of the limb and application of ice should be instituted after the extremity is splinted.

The patient had a fracture. At 3 weeks to 6 months there is clinical union, and this is the first stage of healing that is sufficient to prevent movement of the fracture site when the bones are gently stressed. How is this stage of fracture healing documented? A. Ossification B. Remodeling C. Consolidation D. Callus formation

Ossification Ossification is the stage of fracture healing when there is clinical union and enough strength to prevent movement at the fracture site. Remodeling is the normal function of the bone. Consolidation is when the distance between bone fragments eventually closes and radiologic union first occurs. The callus formation stage appears by the end of the second week of injury when minerals and new bone matrix are deposited in the osteoid that is produced in the granulation tissue stage.

The patient with osteoporosis had a spontaneous hip fracture. How should the nurse document this before the x- ray results return? A. Increase muscle strength with daily isometric exercise. B. Avoid exercising on concrete or hard pavement surfaces. C. Perform stretching and warm-up exercises before exercise. D. Wrap susceptible joints with elastic bandages or adhesive tape before exercise.

Pathologic fracture. A pathologic fracture is a spontaneous fracture at the site of bone disease, such as osteoporosis. An open fracture is when there is communication with the external environment. The oblique fracture has a slanted fracture line. A greenstick fracture is splintered on one side and the other side is bent.

The nurse teaches individuals that one of the best ways to prevent musculoskeletal injuries during physical exercise is by doing what? A. Increase muscle strength with daily isometric exercise. B. Avoid exercising on concrete or hard pavement surfaces. C. Perform stretching and warm-up exercises before exercise. D. Wrap susceptible joints with elastic bandages or adhesive tape before exercise.

Perform stretching and warm- up exercises before exercise. Warm up exercises "prelengthen" potentially strained tissues by avoiding the quick stretch often encountered in sports and also increase the temperature of muscle, resulting in increased speed of cell metabolism, increased speed of nerve impulses, and improved oxygenation of muscle fibers. Stretching is also thought to improve kinesthetic awareness, lessening the chance of uncoordinated movement. Muscle strength is not a key factor in soft tissue injuries and taping or wrapping joints may actually predispose a person to injury by weakening the joint, unless a previous injury is being treated.

The x- ray shows that the patient's fracture is at the remodeling stage. What characteristics of the fracture healing process are happening at this stage? A. Radiologic union B. Absorption of excess cells C. Return to preinjury strength and shape D. Semisolid blood clot at the ends of fragments E. Deposition and absorption of bone in response to stress F. Unorganized network of bone woven around fracture parts

Radiologic union Absorption of excess cells Return to reinjury strength and shape Deposition and absorption of bone in response to stress A,B,C,E When the remodeling stage of healing occurs, radiologic union is present. Excess callus is reabsorbed, trabecular bone is laid, and the bone returns to its preinjury structure strength and shape. The osteoblasts and osteoclasts function normally in response to stress. The fracture hematoma stage is when the hematoma at the ends of the fragments become a semisolid blood clot. There is an unorganized network of bone composed of cartilage., osteoblasts, calcium, and phosphorus woven around fracture parts in callus formation stage.

Application of RICE (rest, ice, compression, and elevation) is indicated for initial management of which type of injury? A. Muscle spasms B. Sprains and strains C. Repetitive strain injury D. Dislocations and subluxations

Strains and sprains Application of cold, compression, and elevation are indicated to prevent edema resulting from sprain and some strain injuries. Muscle spasms are usually treated with heat applications and massage and repetitive strain injuries require cessation of the precipitating activity and physical therapy. Dislocations or subluxations require immediate reduction and immobilization to prevent vascular impairment and bone cell death.

What should be included in the management during the first 48 hours after an acute soft tissue injury of the ankle? A. Use of elastic wrap. B. Initial immobilization and rest. C. Elevation of ankle above the heart. D. Alternating the use of heat and cold. E. Administration of anti-inflammatory drugs.

Use of elastic wrap Initial immobilization and rest Elevation of ankle above heart Administration of antiinflammatory drugs Consider the principle of RICE. Rest: movement should be restricted. Ice: cold should be used to promote vasoconstriction and to reduce edema. C: compression helps to reduce swelling. E: elevate the extremity above the level of the heart. Mild nonsteroidal anti-inflammatory drugs (NSAIDs) may be needed to manage pain. Warm, moist compresses may be used after 48 hours for 20 to 30 minutes at a time to reduce swelling and provide comfort.

23. What emergency considerations must be included with facial fractures (select all that apply)? a. Airway patency b. Oral examination c. Cervical spine injury d. Cranial nerve assessment e. Immobilization of the jaw

a, c. Airway patency and cervical spinal cord injury are the emergency considerations with facial fractures. Oral examination and cranial nerve assessment will be done after the patient is stabilized. Immobilization of the jaw is done surgically for a mandibular fracture.

When is a fat embolism most likely to occur? a. 24 to 48 hours following a fractured tibia b. 36 to 72 hours following a skull fracture c. 4 to 5 days following a fractured femur d. 5 to 6 days following a pelvic fracture

a. 24 to 48 hours following a fractured tibia Initial manifestation of fat embolism usually occur 24 to 48 hours after injury and are associated with fractures of long bones and multiple fractures related to pelvic injuries, including fractures of the femur, tibia, ribs, and pelvis.

What emergency considerations must be included with facial fractures? a. Airway patency b. Oral examination c. Cervical spine injury d. Cranial nerve assessment e. Immobilization of the jaw

a. Airway patency c. Cervical spine injury Airway patency and cervical spinal cord injury are the emergency considerations with facial fractures. Oral examination and cranial nerve assessment will be done after the patient is stabilized. Immobilization of the jaw is done surgically for a mandibular fracture.

What surgical treatment will the nurse prepare the patient for in the presence of compartment syndrome? a. Fasciotomy b. Amputation c. Internal fixation d. Release of tendons

a. Fasciotomy Soft tissue edema in the area of the injury may cause an increase of pressure within the closed spaces of the tissue compartments formed by the nonelastic fascia, creating compartment syndrome. If symptoms occur, it may be necessary to incise the fascia surgically, a procedure known as fasciotomy. Amputation is usually necessary only if the limb becomes septic because of untreated compartment syndrome.

A patient with a fractured right hip has an anterior open reduction and internal fixation of the fracture. What should the nurse plan to do postoperatively? a. Get the patient up to the chair on the first postoperative day. b. Position the patient only on the back and the unoperative side. c. Keep the leg abductor pillow on the patient even when bathing. d. Ambulate the patient with partial weight bearing by discharge.

a. Get the patient up to the chair on the first postoperative day. Because the fracture site is internally fixed with pins or plates, the fracture site is stable and the patient is moved from the bed to the chair on the first postoperative day. Ambulation begins on the first or second postoperative day without weight bearing on the affected leg. Weight bearing on the affected extremity is usually restricted for 6 to 12 weeks until adequate healing is evident on x-ray. The patient may be positioned on the operative side following internal fixation and abductor pillows are used for patients who have total hip replacements.

A patient who has a below- the- knee amputation is to be fitted with a temporary prosthesis. It is most important for the nurse to teach the patient to do what? a. Inspect the residual limb daily for irritation. b. Apply an elastic shrinker before applying the prosthesis. c. Perform range-of-motion (ROM) exercises to the affected leg four times a day. d. Apply alcohol to the residual limb every morning and evening to toughen the skin.

a. Inspect the residual limb daily for irritation. Skin breakdown on the residual limb can prevent the use of a prosthesis so the limb should be inspected every day for signs of irritation or pressure areas. No substances except water and mild soap should be used on the residual limb and range-of-motion (ROM) exercises are not necessary when the patient is using a prosthesis. A residual limb shrinker is an electric stocking that is used to mold the limb in preparation for prosthesis use but a cotton residual limb sock is worn with the prosthesis.

Which kind of hip fracture is usually repaired with a hip prosthesis? a. Intracapsular b. Extracapsular c. Subtrochanteric d. Intertrochanteric

a. Intracapsular A hip prosthesis is usually used for intracapsular fractures. The other option are all extracapsular fractures.

Why does a nurse position a patient with an above-the knee amputation with a delayed prosthetic fitting prone several times a day? a. To prevent flexion contractures b. To assess the posterior skin flap c. To reduce edema in the residual limb d. To relieve pressure on the incision site

a. To prevent flexion contractures Flexion contractures, especially of the hip, may be debilitating and delay rehabilitation of the patient with a leg amputation. To prevent hip flexed or having pillows under the surgical extremity for prolonged periods and the patient should lie on the abdomen for 30 minutes three to four times a day to extend the hip.

The increased risk for falls in the older adult is most likely due to a.changes in balance. b.decrease in bone mass. c.loss of ligament elasticity. d.erosion of articular cartilage.

a.changes in balance Aging can cause changes in a person's sense of balance, making the person unsteady, and proprioception may be altered. The risk for falls also increases in older adults partly because of a loss of strength.

A patient with tendonitis asks what the tendon does. The nurse's response is based on the knowledge that tendons a.connect bone to muscle. b.provide strength to muscle. c.lubricate joints with synovial fluid. d.relieve friction between moving parts.

a.connect bone to muscle Tendons are composed of dense, fibrous connective tissue that contains bundles of closely packed collagen fibers arranged in the same plane for additional strength. They connect the muscle sheath to adjacent bone.

To assess for neurologic status in a patient with a fractured humerus, what should the nurse ask the patient to do? a. Evert, invert, dorsiflex, and plantar flex the foot. b. Abduct, adduct, and oppose the fingers and pronate and supinate the hand. c. Assess the location, quality, and intensity of pain below the site of the injury. d. Assess the color, temperature, capillary refill, peripheral pulses, and presence of edema in the extremity.

b. Abduct, adduct, and oppose the fingers and pronate and supinate the hand. Neurologic assessment includes evaluation of motor and sensory function and, in the upper extremity, includes abduction and adduction of the fingers, opposition of the fingers, and supination and pronation of the hands. It also includes sensory perception in the fingers. Evaluation of the feet would occur in lower extremity injuries. Assessment of color, temperature, capillary refill, peripheral pulses, and edema evaluates vascular status.

When positioning the patient after a total hip arthroplasty with a posterior approach it is important that the nurse maintain the affected extremity in what position? a. Adduction and flexion b. Abduction and extension c. Abduction and internal rotation d. Adduction and external rotation .

b. Abduction and extension Following a total hip arthroplasty with a posterior approach, during hospitalization an abduction pillow is placed between the legs to maintain abduction and the leg is extended. Extremes of internal rotation, adduction, and 90-degree flexion of the hip must be avoided for 4 to 6 weeks postoperatively to prevent dislocation of the prosthesis

A patient complains of pain in the foot of a leg that was recently amputated. What should the nurse recognize about this pain? a. It is caused by swelling at the incision. b. It should be treated with ordered analgesics. c. It will become worse with the use of a prosthesis. d. It can be managed with diversion because it is psychologic.

b. It should be treated with ordered analgesics. Phantom sensation or phantom pain may occur following amputation, especially if pain was present in the affected limb preoperatively. The pain is a real sensation to the patient and will first be treated with analgesics and other pain interventions (i.e., tricyclic antidepressants, antiseizure drugs, transcutaneous electrical nerve stimulation (TENS), mirror therapy, acupuncture). As recovery and ambulation progress, phantom limb sensation usually subsides.

In a patient with a stable vertebral fracture, what should the nurse teach the patient to do? a. Remain on bed rest until the pain is gone. b. Logroll to keep the spine straight when turning. c. How to use bone cement to correct the problem. d. Take as much analgesic as needed to relieve the pain.

b. Logroll to keep the spine straight when turning. The spine should be kept straight by turning the shoulders and hips together (logrolling). This keeps the spine in good alignment until union has been accomplished. Bed rest may be required for a short time but not until the pain is gone. Analgesics should be taken only as ordered. If they do not relieve the pain, the health care provider should be notified. Bone cement is used by the surgeon to stabilize vertebral compression fractures.

A patient is discharged from the outpatient clinic following application of a synthetic fiberglass long arm cast for a fractured ulna. Before discharge, the nurse instructs the patient to do what? a. Never get the cast wet. b. Move the shoulder and fingers frequently. c. Place tape petals around the edges of the cast when it is dry. d. Use a sling to support the arm at waist level for the first 48 hours.

b. Move the shoulder and fingers frequently. A patient with any type of cast should exercise the joints above and below the cast frequently and moving the fingers frequently will improve circulation and help to prevent edema. Unlike plaster casts, thermoplastic resin or fiberglass casts are relatively waterproof and, if they become wet, can be dried with a hair dryer on low setting. Tape petals are used on plaster casts to protect the edges from breaking and crumbling but are not necessary for synthetic casts. After the cast is applied, the extremity should be elevated at about the level of the heart to promote venous return and ice may be used to prevent edema.

Following a knee arthroplasty, a patient has a continuous passive motion machine for the affected joint. The nurse explains to the patient that this device is used for what purpose? a. To relieve edema and pain at the incision site b. To promote early joint mobility and increase knee flexion c. To prevent venous stasis and the formation of a deep venous thrombosis d. To improve arterial circulation to the affected extremity to promote healing

b. To promote early joint mobility and increase knee flexion Continuous motion machines are frequently used following knee surgery to promote earlier joint mobility. Because joint dislocation is not a problem with knee replacements, early exercise with straight leg raises and gentle ROM is also encouraged postoperatively.

When preparing a patient for discharge following fixation of a mandibular fracture, the nurse determines that teaching has been successful when the patient says what? a. "I can keep my mouth moist by sucking on hard candy." b. "I should cut the wires with scissors if I begin to vomit." c. "I may use a bulk-forming laxative if my liquid diet causes constipation." d. "I should use a moist swab to clean my mouth every time I eat something."

c. "I may use a bulk-forming laxative if my liquid diet causes constipation." The low-bulk, high-carbohydrate liquid diet and intake of air through a straw required during mandibular fixation often lead to constipation and flatus, which may be relieved with bulk-forming laxatives, prune juice, or ambulation. Wires or rubber bands should be cut only in the case of cardiac or respiratory arrest and patient should be taught to clear their mouth of vomitus or secretions. The mouth should be thoroughly cleaned with water, saline, or alkaline mouthwashes or using a Water Pik as necessary to remove food debris. Hard candy should not be held in the mouth.

The woman with osteoporosis slipped on the ice and now her wrist hurts. If there is a fracture, what type of fracture is expected? a. Dislocation b. Open fracture c. Colles' fracture d. Incomplete fracture

c. Colles' fracture A Colles' fracture most often occurs in patients over 50 years of age with osteoporosis and frequently when the patient attempts to break a fall with an outstretched arm and hand. Dislocation is the complete separation of articular surfaces of the joint caused by a ligament injury. Open fracture is when there is communication with the external environment. A fracture is incomplete if only part of the bone shaft is fractured and the bone is still in one piece.

Which joint surgery is used to arthroscopically remove degenerative tissue in joints? a. Osteotomy b. Arthrodesis c. Debridement d. Synovectomy

c. Debridement Debridement removes degenerative tissue from joints. Osteotomy correct bone deformity by removal of a wedge or slice of bone. Arthrodesis surgically fuses a joint to relieve pain. Synovectomy removes tissues involved in joint destruction from rheumatoid arthritis (RA).

A patient with a fractured tibia accompanied by extensive soft tissue damage initially has a splint applied and held in place with an elastic bandage. What early sign should alert the nurse that the patient is developing compartment syndrome? a. Paralysis of the toes b. Absence of peripheral pulses c. Distal pain unrelieved by opioid analgesics d. Skin over the injury site is blanched when the bandage is removed

c. Distal pain unrelieved by opioid analgesics Pain that is distal to the injury and is unrelieved by opioid analgesics is the earliest sign of compartment syndrome. Paralysis and absence of peripheral pulses will eventually occur if it is not treated but these are late signs that often appear after permanent damage has occurred. The overlying skin may appear normal because the surface vessels are not occluded.

Which type of fracture occurred when there is radial nerve and brachial artery damage and the fracture is reduced with a hanging arm cast? a. Fractured tibia b. Colles' fracture c. Fractured humerus d. Femoral shaft fracture

c. Fractured humerus The fractured humerus may cause radial nerve and brachial artery damage and it may be reduced nonsurgically with a hanging arm cast. A fractured tibia and femoral shaft are in the leg. The Colles' fracture is in the wrist and manifests with pronounced swelling and obvious deformity of the wrist; it is treated with closed manipulation and immobilization.

The nurse suspects a fat embolism rather than a pulmonary embolism from a venous thrombosis when the patient with a fracture develops what? a. Tachycardia and dyspnea b. A sudden onset of chest pain c. Petechiae around the neck and upper chest d. Electrocargiographic (ECG) changes and decreased PaO2

c. Petechiae around the neck and upper chest Patient with fractures are at risk for both fat embolism and pulmonary embolism from venous thromboembolism but there is a difference in the time of occurrence, with fat embolism occurring shortly after the injury and thrombotic embolism occurring several days after immobilization. They both may cause pulmonary symptoms of chest pain, tachypnea, dyspnea, apprehension, tachycardia, and cyanosis. However, fat embolism may cause petechiae located around the neck, anterior chest wall, axilla, buccal membrane of the mouth, and conjunctiva of the eye, which differentiates it from thrombotic embolism.

24 hours after a below- the- knee amputation, a patient uses the call system to tell the nurse that his dressing (a compression bandage) has fallen off. What is the first action that the nurse should take? a. Apply ice to the site. b. Cover the incision with dry gauze. c. Reapply the compression dressing. d. Elevate the extremity on a couple of pillows.

c. Reapply the compression dressing. The compression dressing or bandage supports the soft tissues, reduces edema, hastens healing, minimizes pain, and promotes residual limb shrinkage. If the dressing is left off, edema will form quickly and may delay rehabilitation. Elevation and ice will not be as effective at preventing the edema that will form. Dressing the incision with dry gauze will not provide the benefits of a compression dressing.

When the nursing student asks the RN what an arthroplasty is, what is the best description the RN can give the student? a. Surgical fusion of a joint to relieve pain b. Correction of bone deformity by removal of a wedge or slice of bone c. Reconstruction or replacement of a joint to relieve pain and correct deformity d. Used in rheumatoid arthritis to remove the tissue involved in joint destruction

c. Reconstruction or replacement of a joint to relieve pain and correct deformity An arthroplasty is reconstruction or replacement of a joint to relieve pain and correct deformity, especially with osteoarthritis, RA, avascular necrosis, congenital deformities, or dislocations. Arthrodesis is the surgical fusion of a joint to relieve pain. An osteotomy removes a wedge of bone to correct a bone deformity. Synovectomy is used in RA to remove the tissue involved in joint destruction.

A patient with an extracapsular hip fracture is admitted to the orthopedic unit and placed in Buck's traction. The nurse explains to the patient that the purpose of the traction is to do what? a. Pull bone fragments into alignment b. Immobilize the leg until healing is complete c. Reduce pain and muscle spasms before surgery d. Prevent damage to the blood vessels at the fracture site

c. Reduce pain and muscle spasms before surgery Although surgical repair is the preferred method of managing intracapsular and extracapsular hip fractures, initially patient frequently may be treated with skin traction, such as Buck's traction or Russell's traction, to immobilize the limb temporarily and to relieve the painful muscle spasms before surgery is performed. Prolonged traction would be required to reduce the fracture or immobilize it for healing, creating a very high risk for complication of immobility.

A 72-year-old man tells the nurse that he cannot perform most of the physical activities he could do 5 years ago because of overall joint aches and pains. What can the nurse do to assist the patient to prevent further deconditioning and decrease the risk for developing musculoskeletal problems? a. Limit weight-bearing exercise to prevent stress on fragile bones and possible hip fractures. b. Advise the patient to avoid the use of canes and walkers because they increase dependence on ambulation aids. c. Advise the patient to increase his activity by more frequently climbing stairs in buildings and other environments with steps. d. Discuss use of over-the-counter (OTC) medications to decrease inflammation and pain so that exercise can be maintained.

d. Almost all older adults have some degree of decreased muscle strength, joint stiffness, and pain with motion. The use of mild antiinflammatory agents decreases inflammation and pain and can help the patient to maintain activity and prevent further deconditioning but other prescribed drugs and potential abdominal problems must be considered by the patient. Musculoskeletal problems in the older adult can be prevented with appropriate strategies, especially exercise. Stair walking can create enough stress on fragile bones to cause a hip fracture and use of ramps may help to prevent falls. Walkers and canes should be used as necessary to decrease stress on joints so that activity can be maintained.

A 65- year- old patient has undergone a right total hip arthroplasty with a cemented prosthesis for treatment of severe osteoarthritis of the hip. What is included in the activity the nurse anticipates for the patient on the patient's first or second postoperative day? a. Transfer from the bed to the chair twice a day only b. Turning form the back to the unaffected side q2hr only c. Crutch walking with non-weight bearing on the operative leg d. Ambulation and weight bearing on the right leg with a walker

d. Ambulation and weight bearing on the right leg with a walker Physical therapy is initiated on the first postoperative day with ambulation and weight bearing using a walker for a patient with a cemented prosthesis and non-weight bearing on the operative side for an uncemented prosthesis. In addition, the patient sits in the chair at least twice a day and is turned to both sides and back with the operative leg supported.

What should the nurse include in discharge instructions for the patient following a hip prosthesis with a posterior approach? a. Restrict walking for 2 to 3 months. b. Take a bath rather than a shower to prevent falling. c. Keep the leg internally rotated while sitting and standing. d. Have a family member put on the patient's shoes and socks.

d. Have a family member put on the patient's shoes and socks. Patients with hip prostheses with a posterior approach must avoid extreme flexion, adduction, or internal rotation for at least 6 weeks to prevent dislocation of the prosthesis. Gradual weight bearing on the limb is allowed and ambulation should be encouraged. The leg should not be externally rotated.

An older adult woman is admitted to the emergency department after falling at home. The nurse cautions her not to put weight on the leg after finding what in the patient assessment? a. Inability to move the toes and ankle b. Edema of the thigh extending to the knee c. Internal rotation of the leg with groin pain d. Shortening and external rotation of the leg

d. Shortening and external rotation of the leg The classic signs of a hip fracture are shortening of the leg and external rotation accompanied by severe pain at the fracture site and additional injury could be caused by the weight bearing on the extremity. The patient may not be able to move the hip or the knee but movement in the ankle and toes is not affected.

When working with patients, the nurse knows that patients have the most difficulties with diarthrodial joints. Which joints are included in this group of joints (select all that apply)? A. Hinge joint of the knee B. Ligaments joining the vertebrae C. Fibrous connective tissue of the skull D. Ball and socket joint of the shoulder or hip E. Cartilaginous connective tissue of the pubis joint

A . Hinge joint of the knee D. Ball and socket joint of the shoulder or hip The diarthrodial joints include the hinge joint of the knee and elbow, the ball and socket joint of the shoulder and hip, the pivot joint of the radioulnar joint, and the condyloid, saddle, and gliding joints of the wrist and hand. The ligaments and cartilaginous connective tissue joining the vertebrae and pubis joint and the fibrous connective tissue of the skull are synarthrotic joints.

A 50-year-old patient is reporting a sore shoulder after raking the yard. The nurse should suspect which problem? A. Bursitis B. Fasciitis C. Sprained ligament D. Achilles tendonitis

A. Bursitis Bursitis is common in adults over age 40 and with repetitive motion, such as raking. Plantar fasciitis frequently occurs as a stabbing pain at the heel caused by straining the ligament that supports the arch. Achilles tendonitis is an inflammation of the tendon that attaches the calf muscle to the heel bone, not the shoulder, and causes pain with walking or running. A sprained ligament occurs when a ligament is stretched or torn from a direct injury or sudden twisting of the joint, not repetitive motion.

Musculoskeletal assessment is an important component of care for patients on what type of long-term therapy? A. Corticosteroids B. β-Adrenergic blockers C. Antiplatelet aggregators D. Calcium-channel blockers

A. Corticosteroids Corticosteroids are associated with avascular necrosis and decreased bone and muscle mass. β-blockers, calcium-channel blockers, and antiplatelet aggregators are not commonly associated with damage to the musculoskeletal system.

Musculoskeletal assessment is an important component of care for patients on what type of long-term therapy? A. Corticosteroids B. β-Adrenergic blockers C. Antiplatelet aggregators D. Calcium-channel blockers

A. Corticosteroids Corticosteroids are associated with avascular necrosis and decreased bone and muscle mass. β-blockers, calcium-channel blockers, and antiplatelet aggregators are not commonly associated with damage to the musculoskeletal system.

A 72-year-old patient with kyphosis is scheduled for dual-energy x-ray absorptiometry (DXA) testing. The nurse will plan to a. explain the procedure. b. start an IV line for contrast medium injection. c. give an oral sedative 60 to 90 minutes before the procedure. d. screen the patient for allergies to shellfish or iodine products.

A. DXA testing is painless and noninvasive. No IV access is necessary. Contrast medium is not used. Because the procedure is painless, no antianxiety medications are required.

Which finding from a patient's right knee arthrocentesis will be of concern to the nurse? a. Cloudy fluid b. Scant thin fluid c. Pale yellow fluid d. Straw-colored fluid

A. The presence of purulent fluid suggests a possible joint infection. Normal synovial fluid is scant in amount and pale yellow/straw-colored.

Which nursing action is correct when performing the straight-leg raising test for an ambulatory patient with back pain? a. Raise the patient's legs to a 60-degree angle from the bed. b. Place the patient initially in the prone position on the exam table. c. Have the patient dangle both legs over the edge of the exam table. d. Instruct the patient to elevate the legs and tense the abdominal muscles.

A. When performing the straight leg-raising test, the patient is in the supine position and the nurse passively lifts the patient's legs to a 60-degree angle. The other actions would not be correct for this test.

A 72-year-old patient with kyphosis is scheduled for dual-energy x-ray absorptiometry (DXA) testing. The nurse will plan to a. explain the procedure. b. start an IV line for contrast medium injection. c. give an oral sedative 60 to 90 minutes before the procedure. d. screen the patient for allergies to shellfish or iodine products.

A. explain the procedure.

A 42-year-old man who is scheduled for an arthrocentesis arrives at the outpatient surgery unit and states, "I do not want this procedure done today." Which response by the nurse is most appropriate? A. "When would you like to reschedule the procedure?" B. "Tell me what your concerns are about this procedure." C. "The procedure is safe, so why should you be worried?" D. "The procedure is not painful because an anesthetic is used."

B. "Tell me what your concerns are about this procedure." The nurse should use therapeutic communication to determine the patient's concern about the procedure. The nurse should not provide false reassurance. It is not appropriate for the nurse to assume the patient is concerned about pain or to assume the patient is asking to reschedule the procedure.

A 57-year-old postmenopausal woman is scheduled for dual-energy x-ray absorptiometry (DXA). Which statement, if made by the patient to the nurse, indicates understanding of the procedure? A. "The bone density in my heel will be measured." B. "This procedure will not cause any pain or discomfort." C. "I will not be exposed to any radiation during the procedure." D. "I will need to remove my hearing aids before the procedure."

B. "This procedure will not cause any pain or discomfort." Dual-energy x-ray absorptiometry (DXA) is painless and measures the bone mass of spine, femur, forearm, and total body with minimal radiation exposure. A quantitative ultrasound (QUS) evaluates density, elasticity, and strength of bone using ultrasound of the calcaneus (heel). Magnetic resonance imaging would require removal of objects such as hearing aids that have metal parts.

Which information in a 67-year-old woman's health history will alert the nurse to the need for a more focused assessment of the musculoskeletal system? a. The patient sprained her ankle at age 13. b. The patient's mother became shorter with aging. c. The patient takes ibuprofen (Advil) for occasional headaches. d. The patient's father died of complications of miliary tuberculosis.

B. A family history of height loss with aging may indicate osteoporosis, and the nurse should perform a more thorough assessment of the patient's current height and other risk factors for osteoporosis. A sprained ankle during adolescence does not place the patient at increased current risk for musculoskeletal problems. A family history of tuberculosis is not a risk factor. Occasional nonsteroidal antiinflammatory drug (NSAID) use does not indicate any increased musculoskeletal risk.

A female patient with a long-standing history of rheumatoid arthritis has sought care because of increasing stiffness in her right knee that has culminated in complete fixation of the joint. The nurse would document the presence of which problem? A. Atrophy B. Ankylosis C. Crepitation D. Contracture

B. Ankylosis Ankylosis is stiffness or fixation of a joint, whereas contracture is reduced movement as a consequence of fibrosis of soft tissue (muscles, ligaments, or tendons). Atrophy is a flabby appearance of muscle leading to decreased function and tone. Crepitation is a grating or crackling sound that accompanies movement.

A female patient with a long-standing history of rheumatoid arthritis has sought care because of increasing stiffness in her right knee that has culminated in complete fixation of the joint. The nurse would document the presence of which problem? A. Atrophy B. Ankylosis C. Crepitation D. Contracture

B. Ankylosis Ankylosis is stiffness or fixation of a joint, whereas contracture is reduced movement as a consequence of fibrosis of soft tissue (muscles, ligaments, or tendons). Atrophy is a flabby appearance of muscle leading to decreased function and tone. Crepitation is a grating or crackling sound that accompanies movement.

Which action can the nurse delegate to unlicensed assistive personnel (UAP) who are working in the orthopedic clinic? a. Grade leg muscle strength for a patient with back pain. b. Obtain blood sample for uric acid from a patient with gout. c. Perform straight-leg-raise testing for a patient with sciatica. d. Check for knee joint crepitation before arthroscopic surgery.

B. Drawing blood specimens is a common skill performed by UAP in clinic settings. The other actions are assessments and require registered nurse (RN)-level judgment and critical thinking.

The home care nurse visits an 84-year-old woman with pneumonia after her discharge from the hospital. Which assessment finding would the nurse expect because of age-related changes in the musculoskeletal system? A. Positive straight-leg-raising test B. Muscle strength is scale grade 3/5 C. Lateral S-shaped curvature of the spine D. Fingers drift to the ulnar side of the forearm

B. Muscle strength is scale grade 3/5 Decreased muscle strength is an age-related change of the musculoskeletal system caused by decreased number and size of the muscle cells. The other assessment findings indicate musculoskeletal abnormalities. A positive straight-leg-raising test indicates nerve root irritation from intervertebral disk prolapse and herniation. An ulnar deviation or drift indicates rheumatoid arthritis due to tendon contracture. Scoliosis is a lateral curvature of the spine.

In reviewing bone remodeling, what should the nurse know about the involvement of bone cells? A. Osteoclasts add canaliculi. B. Osteoblasts deposit new bone. C. Osteocytes are mature bone cells. D. Osteons create a dense bone structure.

B. Osteoblasts deposit new bone. Bone remodeling is achieved when osteoclasts remove old bone and osteoblasts deposit new bone. Osteocytes are mature bone cells, and osteons or Haversian systems create a dense bone structure, but these are not involved with bone remodeling.

In reviewing bone remodeling, what should the nurse know about the involvement of bone cells? A. Osteoclasts add canaliculi. B. Osteoblasts deposit new bone. C. Osteocytes are mature bone cells. D. Osteons create a dense bone structure.

B. Osteoblasts deposit new bone. Bone remodeling is achieved when osteoclasts remove old bone and osteoblasts deposit new bone. Osteocytes are mature bone cells, and osteons or Haversian systems create a dense bone structure, but these are not involved with bone remodeling.

A 42-year-old male patient complains of shoulder pain when the nurse moves his arm behind the back. Which question should the nurse ask? a. "Are you able to feed yourself without difficulty?" b. "Do you have difficulty when you are putting on a shirt?" c. "Are you able to sleep through the night without waking?" d. "Do you ever have trouble lowering yourself to the toilet?"

B. The patient's pain will make it more difficult to accomplish tasks like putting on a shirt or jacket. This pain should not affect the patient's ability to feed himself or use the toilet because these tasks do not involve moving the arm behind the patient. The arm will not usually be positioned behind the patient during sleeping.

A 54-year-old patient admitted with cellulitis and probable osteomyelitis received an injection of radioisotope at 9:00 AM before a bone scan. The nurse should plan to send the patient for the bone scan at what time? A. 9:30 PM B. 10:00 AM C. 11:00 AM D. 1:00 PM

C. 11:00 AM A technician usually administers a calculated dose of a radioisotope 2 hours before a bone scan. If the patient was injected at 9:00 AM, the procedure should be done at 11:00 AM. 10:00 AM would be too early; 1:00 PM and 9:30 PM would be too late.

A 54-year-old patient admitted with cellulitis and probable osteomyelitis received an injection of radioisotope at 9:00 AM before a bone scan. The nurse should plan to send the patient for the bone scan at what time? A. 9:30 PM B. 10:00 AM C. 11:00 AM D. 1:00 PM

C. 11:00 AM A technician usually administers a calculated dose of a radioisotope 2 hours before a bone scan. If the patient was injected at 9:00 AM, the procedure should be done at 11:00 AM. 10:00 AM would be too early; 1:00 PM and 9:30 PM would be too late.

A 63-year-old woman has been taking prednisone (Deltasone) daily for several years after a kidney transplant to prevent organ rejection. What is most important for the nurse to assess? A. Staggering gait B. Ruptured tendon C. Back or neck pain D. Tardive dyskinesia

C. Back or neck pain Osteoporosis with resultant fractures is a frequent and serious complication of systemic corticosteroid therapy. The ribs and vertebrae are affected the most, and patients should be observed for signs of compression fractures (back and neck pain). Phenytoin (Dilantin) is an antiseizure medication. An adverse effect of phenytoin is an ataxic (or staggering) gait. A rare adverse effect of ciprofloxacin (Cipro) and other fluoroquinolones is tendon rupture, usually of the Achilles tendon. The highest risk is in people age 60 and older and in people taking corticosteroids. Antipsychotics and antidepressants may cause tardive dyskinesia, which is characterized by involuntary movements of the tongue and face.

After completing the health history, the nurse assessing the musculoskeletal system will begin by a. having the patient move the extremities against resistance. b. feeling for the presence of crepitus during joint movement. c. observing the patient's body build and muscle configuration. d. checking active and passive range of motion for the extremities.

C. The usual technique in the physical assessment is to begin with inspection. Abnormalities in muscle mass or configuration will allow the nurse to perform a more focused assessment of abnormal areas. The other assessments are also included in the assessment but are usually done after inspection.

An 82-year-old patient is frustrated by her flabby belly and rigid hips. What should the nurse tell the patient about these frustrations? A. "You should go on a diet and exercise more to feel better about yourself." B. "Something must be wrong with you because you should not have these problems." C. "You have arthritis and need to go on nonsteroidal antiinflammatory drugs (NSAIDs)." D. "Decreased muscle mass and strength and increased hip rigidity are normal changes of aging."

D. "Decreased muscle mass and strength and increased hip rigidity are normal changes of aging." The musculoskeletal system's normal changes of aging include decreased muscle mass and strength; increased rigidity in the hips, neck, shoulders, back, and knees; decreased fine motor dexterity; and slowed reaction times. Going on a diet and exercising will help but not stop these changes. Telling the patient "Something must be wrong with you..." will not be helpful to the patient's frustrations.

An 82-year-old patient is frustrated by her flabby belly and rigid hips. What should the nurse tell the patient about these frustrations? A. "You should go on a diet and exercise more to feel better about yourself." B. "Something must be wrong with you because you should not have these problems." C. "You have arthritis and need to go on nonsteroidal antiinflammatory drugs (NSAIDs)." D. "Decreased muscle mass and strength and increased hip rigidity are normal changes of aging."

D. "Decreased muscle mass and strength and increased hip rigidity are normal changes of aging." The musculoskeletal system's normal changes of aging include decreased muscle mass and strength; increased rigidity in the hips, neck, shoulders, back, and knees; decreased fine motor dexterity; and slowed reaction times. Going on a diet and exercising will help but not stop these changes. Telling the patient "Something must be wrong with you..." will not be helpful to the patient's frustrations.

The nurse finds that a patient can flex the arms when no resistance is applied but is unable to flex when the nurse applies light resistance. The nurse should document the patient's muscle strength as level a. 0. b. 1. c. 2. d. 3.

D. 3

Which finding is of highest priority when the nurse is planning care for a 77-year-old patient seen in the outpatient clinic? a. Symmetric joint swelling of fingers b. Decreased right knee range of motion c. Report of left hip aching when jogging d. History of recent loss of balance and fall

D. A history of falls requires further assessment and development of fall prevention strategies. The other changes are more typical of bone and joint changes associated with normal aging.

The nurse finds that a patient can flex the arms when no resistance is applied but is unable to flex when the nurse applies light resistance. The nurse should document the patient's muscle strength as level a. 0. b. 1. c. 2. d. 3.

D. A level 3 indicates that the patient is unable to move against resistance but can move against gravity. Level 1 indicates minimal muscle contraction, level 2 indicates that the arm can move when gravity is eliminated, and level 4 indicates active movement with some resistance.

The nurse who notes that a 59-year-old female patient has lost 1 inch in height over the past 2 years will plan to teach the patient about a. discography studies. b. myelographic testing. c. magnetic resonance imaging (MRI). d. dual-energy x-ray absorptiometry (DXA).

D. The decreased height and the patient's age suggest that the patient may have osteoporosis and that bone density testing is needed. Discography, MRI, and myelography are typically done for patients with current symptoms caused by musculoskeletal dysfunction and are not the initial diagnostic tests for osteoporosis.

A 54-year-old patient is about to have a bone scan. In teaching the patient about this procedure, the nurse should include what information? A. Two additional follow-up scans will be required. B. There will be only mild pain associated with the procedure. C. The procedure takes approximately 15 to 30 minutes to complete. D. The patient will be asked to drink increased fluids after the procedure.

D. The patient will be asked to drink increased fluids after the procedure. Patients are asked to drink increased fluids after a bone scan to aid in excretion of the radioisotope, if not contraindicated by another condition. No follow-up scans and no pain are associated with bone scans that take 1 hour of lying supine.

A 54-year-old patient is about to have a bone scan. In teaching the patient about this procedure, the nurse should include what information? A. Two additional follow-up scans will be required. B. There will be only mild pain associated with the procedure. C. The procedure takes approximately 15 to 30 minutes to complete. D. The patient will be asked to drink increased fluids after the procedure.

D. The patient will be asked to drink increased fluids after the procedure. Patients are asked to drink increased fluids after a bone scan to aid in excretion of the radioisotope, if not contraindicated by another condition. No follow-up scans and no pain are associated with bone scans that take 1 hour of lying supine.

While performing passive range of motion for a patient, the nurse puts the ankle joint through the movements of (select all that apply) a.flexion and extension. b.inversion and eversion. c.pronation and supination d.flexion, extension, abduction, and adduction. e.pronation, supination, rotation, and circumduction.

a.flexion and extension. b.inversion and eversion. Common movements that occur at the ankle include inversion, eversion, flexion, and extension.

A normal assessment finding of the musculoskeletal system is a.no deformity or crepitation. b.muscle and bone strength of 4. c.ulnar deviation and subluxation. d.angulation of bone toward midline.

a.no deformity or crepitation Normal physical assessment findings of the musculoskeletal system include normal spinal curvatures; no muscle atrophy or asymmetry; no joint swelling, deformity, or crepitation; no tenderness on palpation of muscles and joints; full range of motion of all joints without pain or laxity; and muscle strength score of 5.

A patient is scheduled for an electromyogram (EMG). The nurse explains that this diagnostic test involves a.incision or puncture of the joint capsule. b.insertion of small needles into certain muscles. c.administration of a radioisotope before the procedure. d.placement of skin electrodes to record muscle activity.

b.insertion of small needles into certain muscles Electromyography (EMG) is an evaluation of electrical potential associated with skeletal muscle contraction. Small-gauge needles are inserted into certain muscles and attached to leads that record electrical activity of muscle. Results provide information related to lower motor neuron dysfunction and primary muscle disease

While obtaining subjective assessment data related to the musculoskeletal system, it is particularly important to ask a patient about other medical problems such as a.hypertension. b.thyroid problems. c.diabetes mellitus. d.chronic bronchitis.

c.diabetes mellitus The nurse should question the patient about past medical problems because certain illnesses are known to affect the musculoskeletal system directly or indirectly. These diseases include tuberculosis, poliomyelitis, diabetes mellitus, parathyroid problems, hemophilia, rickets, soft tissue infection, and neuromuscular disabilities.

The bone cells that function in the resorption of bone tissue are called a.osteoids b.osteocytes c.osteoclasts d.osteoblasts

c.osteoclasts Osteoclasts participate in bone remodeling by assisting in the breakdown of bone tissue.

When grading muscle strength, the nurse records a score of 3, which indicates a.no detection of muscular contraction. b.a barely detectable flicker of contraction. c.active movement against full resistance without fatigue. d.active movement against gravity but not against resistance.

d. active movement against gravity but not against resistance Muscle strength score of 3 indicates active movement only against gravity and not against resistance

To prevent muscle atrophy, the nurse teaches the patient with a leg immobilized in traction to perform (select all that apply) a.flexion contractions. b.tetanic contractions. c.isotonic contractions. d.isometric contractions. e.extension contractions.

d.isometric contractions Isometric contractions increase the tension within a muscle but do not produce movement. Repeated isometric contractions make muscles grow larger and stronger. Muscular atrophy (i.e., decrease in size) occurs with the absence of contraction that results from immobility.


Related study sets

Data driven decision making Final review

View Set

Ch. 58: Assessment and Management of Patients With Breast Disorders

View Set

Driver's Education-Chapter 9 Natural Laws

View Set

Chapter 1 - Libby, Libby, and Short - Financial Accounting, Chapter 2 - Libby, Libby and Short - Financial Accounting, Chapter 3 - Libby, Libby & Short - Financial Accounting, Chapter 4 - Libby, Libby & Short - Financial Accounting

View Set

Chapter 20 - Nursing Informatics

View Set

Cardiac Care Week 3: Heart -- Electrical Conduction System

View Set